├── main.pdf ├── main - 5A.pdf ├── main - 5B.pdf ├── README.md ├── preface.tex ├── main.tex ├── main - 5A.tex ├── main - 5B.tex ├── 8.tex ├── 7.tex ├── 4.tex └── 6.tex /main.pdf: -------------------------------------------------------------------------------- https://raw.githubusercontent.com/jamesfang8499/math5/HEAD/main.pdf -------------------------------------------------------------------------------- /main - 5A.pdf: -------------------------------------------------------------------------------- https://raw.githubusercontent.com/jamesfang8499/math5/HEAD/main - 5A.pdf -------------------------------------------------------------------------------- /main - 5B.pdf: -------------------------------------------------------------------------------- https://raw.githubusercontent.com/jamesfang8499/math5/HEAD/main - 5B.pdf -------------------------------------------------------------------------------- /README.md: -------------------------------------------------------------------------------- 1 | # 中学数学实验教材 第五册(上、下册) 2 | 本项目对1980年代初出版发行的《中学数学实验教材 第五册》进行了重排。 3 | 4 | 这一套中学数学实验教材是教育部委托北京师范大学、中国科学院数学研究所、人民教育出版社、北京师范学院、北京景山学校等单位组成的领导小组组织“中学数学实验教材编写组”,根据美国加州大学伯克利分校数学系项武义教授的《关于中学实验数学教材的设想》编写的。 5 | 6 | 本重排本,将原教材的上下两册合排成一本,当中涉及的部分内容从现在看来已经超纲或过时,请阅读时仔细甄别。 7 | 8 | 书中的矢量图片使用Tikz和Tkz-euclide制作。其余的点阵图则是来自于扫描版电子文档(限于作者的能力和精力,无法将所有内容均以矢量图全部重绘)。 9 | 10 | **main - 5A.pdf和main - 5B.pdf分别是经过拆分后的上、下两册。** 11 | 12 | **注意:本项目的内容勿用于商业目的。** 13 | 14 | # 目录 15 | 16 | ## 第一章 直线和平面 17 | * 空间图形 18 | * 集合运算 19 | * 平面 20 | * 直线与直线的位置关系 21 | * 直线与平面的位置关系 22 | * 平面与平面的位置关系 23 | * 空间的对称性 24 | * 章末提示 25 | 26 | ## 第二章 柱、锥、台、球 27 | * 柱、锥、台、球的定义及性质 28 | * 柱、锥、台、球的全面积和部分面积 29 | * 柱、锥、台、球的体积和祖暅原理 30 | * 附录 圆锥曲线 31 | 32 | ## 第三章 向量与向量运算 33 | * 平面位移向量及其加法运算 34 | * 相似与向量的倍积 35 | * 长度、角度与内积运算 36 | * 空间向量及其运算 37 | 38 | 39 | ## 第四章 向量几何初步 40 | * 平行与相似 41 | * 垂直与度量问题 42 | * 圆 43 | 44 | 45 | ## 第五章 向量的坐标运算 直线与圆 46 | * 向量的坐标运算 47 | * 直线方程 48 | * 圆 49 | 50 | ## 第六章 圆锥曲线 51 | 52 | * 圆锥曲线的标准方程及其性质 53 | * 坐标变换 54 | * 一般二元二次方程的讨论 55 | 56 | ## 第七章 极坐标与参数方程 57 | * 极坐标系与曲线的极坐标方程 58 | * 参数方程 59 | 60 | 61 | ## 第八章 空间解析几何初步 62 | * 空间向量的坐标运算 63 | * 空间的平面 直线与球面方程 64 | 65 | 66 | 67 | 68 | -------------------------------------------------------------------------------- /preface.tex: -------------------------------------------------------------------------------- 1 | \chapter{前~~言} 2 | 3 | 这一套中学数学实验教材,内容的选取原则是精简实 4 | 用,教材的处理力求深入浅出,顺理成章,尽量作到使人人 5 | 能懂,到处有用。 6 | 7 | 本教材适用于重点中学,侧重在满足学生将来从事理工 8 | 方面学习和工作的需要。 9 | 10 | 本教材的教学目的是:使学生切实学好从事现代生产、 11 | 特别是学习现代科学技术所必需的数学基础知识;通过对数 12 | 学理论、应用、思想和方法的学习,培养学生运算能力,思 13 | 维能力,空间想象力,从而逐步培养运用数学的思想和方法 14 | 去分析和解决实际问题的能力;通过数学的教学和学习,培 15 | 养学生良好的学习习惯,严谨的治学态度和科学的思想方 16 | 法,逐步形成辩证唯物主义世界观。 17 | 18 | 根据上述教学目的,本教材精选了传统数学那些普遍实 19 | 用的最基础的部分,这就是在理论上、应用上和思想方法上 20 | 都是基本的、长远起作用的通性、通法。比如,代数中的数 21 | 系运算律,式的运算,解代数方程,待定系数法;几何中的 22 | 图形的基本概念和主要性质,向量,解析几何;分析中的函 23 | 数,极限,连续,微分,积分;概率统计以及逻辑、推理论 24 | 证等知识。对于那些理论和应用上虽有一定作用,但发展余 25 | 地不大,或没有普遍意义和实用价值,或不必要的重复和过 26 | 于繁琐的内容,如立体几何中的空间作图,几何体的体积、 27 | 表面积计算,几何难题,因式分解,对数计算等作了较大的 28 | 精简或删减。 29 | 30 | 全套教材共分六册。第一册是代数。在总结小学所学自 31 | 然数、小数、分数基础上,明确提出运算律,把数扩充到有 32 | 理数和实数系。灵活运用运算律解一元一次、二次方程,二 33 | 元、三元一次方程组,然后进一步系统化,引进多项式运 34 | 算,综合除法,辗转相除,余式定理及其推论,学到根式、 35 | 分式、部分分式。第二册是几何。由直观几何形象分析归纳 36 | 出几何基本概念和基本性质,通过集合术语、简易逻辑转入 37 | 欧氏推理几何,处理直线形,圆、基本轨迹与作图,三角比 38 | 与解三角形等基本内容。第三册是函数。数形结合引入坐 39 | 标,研究多项式函数,指数、对数、三角函数,不等式等。 40 | 第四册是代数。把数扩充到复数系,进一步加强多项式理论, 41 | 方程式论,讲线性方程组理论,概率(离散的)统计的初步 42 | 知识。第五册是几何。引进向量,用向量和初等几何方法综 43 | 合处理几何问题,坐标化处理直线、圆、锥线,坐标变换与 44 | 二次曲线讨论,然后讲立体几何,并引进空间向量研究空间 45 | 解析几何初步知识。第六册是微积分初步。突出逼近法,讲 46 | 实数完备性,函数,极限,连续,变率与微分,求和与积分。 47 | 48 | 本教材基本上采取代数、几何、分析分科,初中、高中 49 | 循环排列的安排体系。教学可按初一、初二代数、几何双科 50 | 并进,初三学分析,高一、高二代数(包括概率统计)、几 51 | 何双科并进,高三学微积分的程序来安排。 52 | 53 | 本教材的处理力求符合历史发展和认识发展的规律,深 54 | 入浅出,顺理成章。突出由算术到代数,由实验几何到论证 55 | 几何,由综合几何到解析几何,由常量数学到变量数学等四 56 | 个重大转折,着力采取措施引导学生合乎规律地实现这些转 57 | 折,为此,强调数系运算律,集合逻辑,向量和逼近法分别 58 | 在实现这四个转折中的作用。这样既遵循历史发展的规律, 59 | 又突出了几个转折关头,缩短了认识过程,有利于学生掌握 60 | 数学思想发展的脉络,提高数学教学的思想性。 61 | 62 | 这一套中学数学实验教材是教育部委托北京师范大学、 63 | 中国科学院数学研究所、人民教育出版社、北京师范学院、 64 | 北京景山学校等单位组成的领导小组组织“中学数学实验教 65 | 材编写组”,根据美国加州大学伯克利分校数学系项武义教 66 | 授的《关于中学实验数学教材的设想》编写的。第一版印出 67 | 后,由教育部实验研究组和有关省市实验研究组指导在北 68 | 京景山学校、北京师院附中、上海大同中学、天津南开中 69 | 学、天津十六中学、广东省实验中学、华南师院附中、长春 70 | 市实验中学等校试教过两遍,在这个基础上编写组吸收了实 71 | 验学校老师们的经验和意见,修改成这一版《中学数学实验 72 | 教材》,正式出版,内部发行,供中学选作实验教材,教师 73 | 参考书或学生课外读物。在编写和修订过程中,项武义教授 74 | 曾数次详细修改过原稿,提出过许多宝贵意见。 75 | 76 | 本教材虽然试用过两遍,但是实验基础仍然很不够,这 77 | 次修改出版,目的是通过更大范围的实验研究,逐步形成另 78 | 一套现代化而又适合我国国情的中学数学教科书。在实验过 79 | 程中,我们热忱希望大家多提意见,以便进一步把它修改好。 80 | 81 | \begin{flushright} 82 | 中学数学实验教材编写组\\ 83 | 一九八一年三月 84 | \end{flushright} 85 | 86 | 87 | 88 | 89 | 90 | 91 | 92 | -------------------------------------------------------------------------------- /main.tex: -------------------------------------------------------------------------------- 1 | \documentclass[b5paper, openany]{ctexbook} 2 | 3 | 4 | \usepackage[margin=2.5cm]{geometry} 5 | 6 | 7 | \usepackage{pifont} 8 | \usepackage[perpage,symbol*]{footmisc} 9 | \DefineFNsymbols{circled}{{\ding{192}}{\ding{193}}{\ding{194}} 10 | {\ding{195}}{\ding{196}}{\ding{197}}{\ding{198}}{\ding{199}}{\ding{200}}{\ding{201}}} 11 | \setfnsymbol{circled} 12 | 13 | \usepackage{ulem} 14 | 15 | \usepackage{amsmath,amsfonts,mathrsfs,amssymb} 16 | \usepackage{graphicx} 17 | 18 | \usepackage[font=bf,labelfont=bf,labelsep=quad]{caption} 19 | 20 | \usepackage{tikz} 21 | 22 | 23 | \usepackage{ntheorem} 24 | \theoremseparator{\;} 25 | 26 | 27 | 28 | \usepackage{blkarray} 29 | \usepackage{bm} 30 | \usepackage[colorlinks=true, linkcolor=black]{hyperref} 31 | 32 | \usepackage{enumerate} 33 | 34 | 35 | \theoremstyle{plain} 36 | \theoremheaderfont{\normalfont\bfseries} 37 | \theorembodyfont{\normalfont} 38 | 39 | 40 | \usepackage[framemethod=tikz]{mdframed} 41 | 42 | 43 | \newtheorem{example}{\bf 例}[chapter] 44 | \newenvironment{solution}{\noindent {\bf 解:}}{} 45 | \newenvironment{analyze}{\noindent {\bf 分析:}}{} 46 | \newenvironment{rmk}{\noindent {\bf 注意:}}{} 47 | \newenvironment{note}{\noindent {\bf 说明:}}{} 48 | 49 | 50 | 51 | \renewcommand{\proofname}{\bf 证明:} 52 | \newenvironment{proof}{{\noindent \bf 证明:}}{}%{\hfill $\square$\par} 53 | 54 | \newcommand{\E}{\mathbb{E}} 55 | \renewcommand{\Pr}{\mathbb{P}} 56 | \newcommand{\EP}{\mathbb{E}^{\mathbb{P}}} 57 | \newcommand{\EQ}{\mathbb{E}^{\mathbb{Q}}} 58 | \newcommand{\dif}{\,{\rm d}} 59 | \newcommand{\Var}{{\rm Var}} 60 | \newcommand{\Cov}{{\rm Cov}} 61 | \newcommand{\x}{\times} 62 | 63 | 64 | \usepackage{tcolorbox} 65 | \tcbuselibrary{breakable} 66 | \tcbuselibrary{most} 67 | 68 | 69 | 70 | \newtcolorbox{ex}[1][] 71 | {colback = white, colframe = cyan!75!black, fonttitle = \bfseries, 72 | colbacktitle = cyan!85!black, enhanced, 73 | attach boxed title to top center={yshift=-2mm},breakable, 74 | title=练习, #1} 75 | 76 | \newtcolorbox{blk}[2][] 77 | {colback = white, colframe = magenta!75!black, fonttitle = \bfseries, 78 | colbacktitle = magenta!85!black, enhanced, 79 | attach boxed title to top left={xshift=5mm, yshift=-2mm},breakable, 80 | title=#2, #1} 81 | 82 | 83 | \setcounter{tocdepth}{2} 84 | 85 | \setcounter{secnumdepth}{3} 86 | 87 | 88 | 89 | \ctexset { 90 | section = { 91 | name = {第,节}, 92 | number = \chinese{section}}, 93 | subsection = { 94 | name = {,、\hspace{-1em}}, 95 | number = \chinese{subsection} 96 | }, 97 | subsubsection = { 98 | name = {(,)\hspace{-1em}}, 99 | number = \chinese{subsubsection}, 100 | } 101 | } 102 | 103 | 104 | 105 | \renewcommand{\contentsname}{目~~录} 106 | 107 | \newcommand{\poly}{\polynomial[reciprocal]} 108 | 109 | 110 | 111 | \usepackage{mathtools} 112 | 113 | \setlength{\abovecaptionskip}{0.cm} 114 | \setlength{\belowcaptionskip}{-0.cm} 115 | 116 | \usetikzlibrary{decorations.pathmorphing, patterns} 117 | \usetikzlibrary{calc, patterns, decorations.markings} 118 | \usetikzlibrary{positioning, snakes} 119 | 120 | 121 | \usepackage{tkz-base, tkz-euclide} 122 | \usepackage{yhmath} % \wideparen 123 | \usepackage{longdivision} 124 | \usepackage{polynom} 125 | \usepackage{polynomial} 126 | \usepackage{cancel} 127 | 128 | \renewcommand{\frac}{\dfrac} 129 | \newcommand{\oc}{$^{\circ}{\rm C}$} 130 | \renewcommand{\Vec}{\overrightarrow} 131 | 132 | \usepackage{multicol} 133 | \usepackage{cases} 134 | 135 | \newcommand\parallelogram{% 136 | \mathord{\text{% 137 | \tikz[baseline] 138 | \draw(0em,.1ex)--++(.9em,0ex)--++(.2em,1.2ex)--++(-.9em,0ex)--cycle; 139 | }} 140 | } 141 | 142 | \renewcommand\parallel{\mathrel{/\mskip-2.5mu/}} 143 | 144 | 145 | \newcommand{\eY}{\vec{e}_y} 146 | \newcommand{\eX}{\vec{e}_x} 147 | \newcommand{\eZ}{\vec{e}_z} 148 | \newcommand{\va}{\vec{a}} 149 | \newcommand{\expval}[1]{\left\langle #1 \right\rangle} 150 | 151 | 152 | 153 | 154 | 155 | \begin{document} 156 | 157 | 158 | 159 | 160 | \title{\Huge\bfseries 中学数学实验教材\\第五册} 161 | 162 | 163 | 164 | \author{\Large 中学数学实验教材编写组编} 165 | \date{\Large 1984年6月} 166 | 167 | \maketitle 168 | 169 | 170 | 171 | 172 | \frontmatter 173 | 174 | \input{preface.tex} 175 | \tableofcontents 176 | 177 | 178 | \mainmatter 179 | 180 | \input{1.tex} 181 | 182 | \input{2.tex} 183 | \input{3.tex} 184 | \input{4.tex} 185 | \input{5.tex} 186 | 187 | \input{6.tex} 188 | \input{7.tex} 189 | \input{8.tex} 190 | 191 | 192 | 193 | \end{document} 194 | 195 | 196 | \begin{figure}[htp]\centering 197 | \begin{minipage}[t]{0.48\textwidth} 198 | \centering 199 | \begin{tikzpicture}[>=latex, scale=1] 200 | 201 | \end{tikzpicture} 202 | \caption{} 203 | \end{minipage} 204 | \begin{minipage}[t]{0.48\textwidth} 205 | \centering 206 | \begin{tikzpicture}[>=latex, scale=1] 207 | 208 | \end{tikzpicture} 209 | \caption{} 210 | \end{minipage} 211 | \end{figure} 212 | 213 | -------------------------------------------------------------------------------- /main - 5A.tex: -------------------------------------------------------------------------------- 1 | \documentclass[b5paper, openany]{ctexbook} 2 | 3 | 4 | \usepackage[margin=2.5cm]{geometry} 5 | 6 | 7 | \usepackage{pifont} 8 | \usepackage[perpage,symbol*]{footmisc} 9 | \DefineFNsymbols{circled}{{\ding{192}}{\ding{193}}{\ding{194}} 10 | {\ding{195}}{\ding{196}}{\ding{197}}{\ding{198}}{\ding{199}}{\ding{200}}{\ding{201}}} 11 | \setfnsymbol{circled} 12 | 13 | \usepackage{ulem} 14 | 15 | \usepackage{amsmath,amsfonts,mathrsfs,amssymb} 16 | \usepackage{graphicx} 17 | 18 | \usepackage[font=bf,labelfont=bf,labelsep=quad]{caption} 19 | 20 | \usepackage{tikz} 21 | 22 | 23 | \usepackage{ntheorem} 24 | \theoremseparator{\;} 25 | 26 | 27 | 28 | \usepackage{blkarray} 29 | \usepackage{bm} 30 | \usepackage[colorlinks=true, linkcolor=black]{hyperref} 31 | 32 | \usepackage{enumerate} 33 | 34 | 35 | \theoremstyle{plain} 36 | \theoremheaderfont{\normalfont\bfseries} 37 | \theorembodyfont{\normalfont} 38 | 39 | 40 | \usepackage[framemethod=tikz]{mdframed} 41 | 42 | 43 | \newtheorem{example}{\bf 例}[chapter] 44 | \newenvironment{solution}{\noindent {\bf 解:}}{} 45 | \newenvironment{analyze}{\noindent {\bf 分析:}}{} 46 | \newenvironment{rmk}{\noindent {\bf 注意:}}{} 47 | \newenvironment{note}{\noindent {\bf 说明:}}{} 48 | 49 | 50 | 51 | \renewcommand{\proofname}{\bf 证明:} 52 | \newenvironment{proof}{{\noindent \bf 证明:}}{}%{\hfill $\square$\par} 53 | 54 | \newcommand{\E}{\mathbb{E}} 55 | \renewcommand{\Pr}{\mathbb{P}} 56 | \newcommand{\EP}{\mathbb{E}^{\mathbb{P}}} 57 | \newcommand{\EQ}{\mathbb{E}^{\mathbb{Q}}} 58 | \newcommand{\dif}{\,{\rm d}} 59 | \newcommand{\Var}{{\rm Var}} 60 | \newcommand{\Cov}{{\rm Cov}} 61 | \newcommand{\x}{\times} 62 | 63 | 64 | \usepackage{tcolorbox} 65 | \tcbuselibrary{breakable} 66 | \tcbuselibrary{most} 67 | 68 | 69 | 70 | \newtcolorbox{ex}[1][] 71 | {colback = white, colframe = cyan!75!black, fonttitle = \bfseries, 72 | colbacktitle = cyan!85!black, enhanced, 73 | attach boxed title to top center={yshift=-2mm},breakable, 74 | title=练习, #1} 75 | 76 | \newtcolorbox{blk}[2][] 77 | {colback = white, colframe = magenta!75!black, fonttitle = \bfseries, 78 | colbacktitle = magenta!85!black, enhanced, 79 | attach boxed title to top left={xshift=5mm, yshift=-2mm},breakable, 80 | title=#2, #1} 81 | 82 | 83 | \setcounter{tocdepth}{2} 84 | 85 | \setcounter{secnumdepth}{3} 86 | 87 | 88 | 89 | \ctexset { 90 | section = { 91 | name = {第,节}, 92 | number = \chinese{section}}, 93 | subsection = { 94 | name = {,、\hspace{-1em}}, 95 | number = \chinese{subsection} 96 | }, 97 | subsubsection = { 98 | name = {(,)\hspace{-1em}}, 99 | number = \chinese{subsubsection}, 100 | } 101 | } 102 | 103 | 104 | 105 | \renewcommand{\contentsname}{目~~录} 106 | 107 | \newcommand{\poly}{\polynomial[reciprocal]} 108 | 109 | 110 | 111 | \usepackage{mathtools} 112 | 113 | \setlength{\abovecaptionskip}{0.cm} 114 | \setlength{\belowcaptionskip}{-0.cm} 115 | 116 | \usetikzlibrary{decorations.pathmorphing, patterns} 117 | \usetikzlibrary{calc, patterns, decorations.markings} 118 | \usetikzlibrary{positioning, snakes} 119 | 120 | 121 | \usepackage{tkz-base, tkz-euclide} 122 | \usepackage{yhmath} % \wideparen 123 | \usepackage{longdivision} 124 | \usepackage{polynom} 125 | \usepackage{polynomial} 126 | \usepackage{cancel} 127 | 128 | \renewcommand{\frac}{\dfrac} 129 | \newcommand{\oc}{$^{\circ}{\rm C}$} 130 | \renewcommand{\Vec}{\overrightarrow} 131 | 132 | \usepackage{multicol} 133 | \usepackage{cases} 134 | 135 | \newcommand\parallelogram{% 136 | \mathord{\text{% 137 | \tikz[baseline] 138 | \draw(0em,.1ex)--++(.9em,0ex)--++(.2em,1.2ex)--++(-.9em,0ex)--cycle; 139 | }} 140 | } 141 | 142 | \renewcommand\parallel{\mathrel{/\mskip-2.5mu/}} 143 | 144 | 145 | \newcommand{\eY}{\vec{e}_y} 146 | \newcommand{\eX}{\vec{e}_x} 147 | \newcommand{\eZ}{\vec{e}_z} 148 | \newcommand{\va}{\vec{a}} 149 | \newcommand{\expval}[1]{\left\langle #1 \right\rangle} 150 | 151 | 152 | 153 | 154 | 155 | \begin{document} 156 | 157 | 158 | 159 | 160 | \title{\Huge\bfseries 中学数学实验教材\\第五册(上)} 161 | 162 | 163 | 164 | \author{\Large 中学数学实验教材编写组编} 165 | \date{\Large 1984年6月} 166 | 167 | \maketitle 168 | 169 | 170 | 171 | 172 | \frontmatter 173 | 174 | \input{preface.tex} 175 | \tableofcontents 176 | 177 | 178 | \mainmatter 179 | 180 | \input{1.tex} 181 | 182 | \input{2.tex} 183 | \input{3.tex} 184 | \input{4.tex} 185 | % \input{5.tex} 186 | 187 | % \input{6.tex} 188 | % \input{7.tex} 189 | % \input{8.tex} 190 | 191 | 192 | 193 | \end{document} 194 | 195 | 196 | \begin{figure}[htp]\centering 197 | \begin{minipage}[t]{0.48\textwidth} 198 | \centering 199 | \begin{tikzpicture}[>=latex, scale=1] 200 | 201 | \end{tikzpicture} 202 | \caption{} 203 | \end{minipage} 204 | \begin{minipage}[t]{0.48\textwidth} 205 | \centering 206 | \begin{tikzpicture}[>=latex, scale=1] 207 | 208 | \end{tikzpicture} 209 | \caption{} 210 | \end{minipage} 211 | \end{figure} 212 | 213 | -------------------------------------------------------------------------------- /main - 5B.tex: -------------------------------------------------------------------------------- 1 | \documentclass[b5paper, openany]{ctexbook} 2 | 3 | 4 | \usepackage[margin=2.5cm]{geometry} 5 | 6 | 7 | \usepackage{pifont} 8 | \usepackage[perpage,symbol*]{footmisc} 9 | \DefineFNsymbols{circled}{{\ding{192}}{\ding{193}}{\ding{194}} 10 | {\ding{195}}{\ding{196}}{\ding{197}}{\ding{198}}{\ding{199}}{\ding{200}}{\ding{201}}} 11 | \setfnsymbol{circled} 12 | 13 | \usepackage{ulem} 14 | 15 | \usepackage{amsmath,amsfonts,mathrsfs,amssymb} 16 | \usepackage{graphicx} 17 | 18 | \usepackage[font=bf,labelfont=bf,labelsep=quad]{caption} 19 | 20 | \usepackage{tikz} 21 | 22 | 23 | \usepackage{ntheorem} 24 | \theoremseparator{\;} 25 | 26 | 27 | 28 | \usepackage{blkarray} 29 | \usepackage{bm} 30 | \usepackage[colorlinks=true, linkcolor=black]{hyperref} 31 | 32 | \usepackage{enumerate} 33 | 34 | 35 | \theoremstyle{plain} 36 | \theoremheaderfont{\normalfont\bfseries} 37 | \theorembodyfont{\normalfont} 38 | 39 | 40 | \usepackage[framemethod=tikz]{mdframed} 41 | 42 | 43 | \newtheorem{example}{\bf 例}[chapter] 44 | \newenvironment{solution}{\noindent {\bf 解:}}{} 45 | \newenvironment{analyze}{\noindent {\bf 分析:}}{} 46 | \newenvironment{rmk}{\noindent {\bf 注意:}}{} 47 | \newenvironment{note}{\noindent {\bf 说明:}}{} 48 | 49 | 50 | 51 | \renewcommand{\proofname}{\bf 证明:} 52 | \newenvironment{proof}{{\noindent \bf 证明:}}{}%{\hfill $\square$\par} 53 | 54 | \newcommand{\E}{\mathbb{E}} 55 | \renewcommand{\Pr}{\mathbb{P}} 56 | \newcommand{\EP}{\mathbb{E}^{\mathbb{P}}} 57 | \newcommand{\EQ}{\mathbb{E}^{\mathbb{Q}}} 58 | \newcommand{\dif}{\,{\rm d}} 59 | \newcommand{\Var}{{\rm Var}} 60 | \newcommand{\Cov}{{\rm Cov}} 61 | \newcommand{\x}{\times} 62 | 63 | 64 | \usepackage{tcolorbox} 65 | \tcbuselibrary{breakable} 66 | \tcbuselibrary{most} 67 | 68 | 69 | 70 | \newtcolorbox{ex}[1][] 71 | {colback = white, colframe = cyan!75!black, fonttitle = \bfseries, 72 | colbacktitle = cyan!85!black, enhanced, 73 | attach boxed title to top center={yshift=-2mm},breakable, 74 | title=练习, #1} 75 | 76 | \newtcolorbox{blk}[2][] 77 | {colback = white, colframe = magenta!75!black, fonttitle = \bfseries, 78 | colbacktitle = magenta!85!black, enhanced, 79 | attach boxed title to top left={xshift=5mm, yshift=-2mm},breakable, 80 | title=#2, #1} 81 | 82 | 83 | \setcounter{tocdepth}{2} 84 | 85 | \setcounter{secnumdepth}{3} 86 | 87 | 88 | 89 | \ctexset { 90 | section = { 91 | name = {第,节}, 92 | number = \chinese{section}}, 93 | subsection = { 94 | name = {,、\hspace{-1em}}, 95 | number = \chinese{subsection} 96 | }, 97 | subsubsection = { 98 | name = {(,)\hspace{-1em}}, 99 | number = \chinese{subsubsection}, 100 | } 101 | } 102 | 103 | 104 | 105 | \renewcommand{\contentsname}{目~~录} 106 | 107 | \newcommand{\poly}{\polynomial[reciprocal]} 108 | 109 | 110 | 111 | \usepackage{mathtools} 112 | 113 | \setlength{\abovecaptionskip}{0.cm} 114 | \setlength{\belowcaptionskip}{-0.cm} 115 | 116 | \usetikzlibrary{decorations.pathmorphing, patterns} 117 | \usetikzlibrary{calc, patterns, decorations.markings} 118 | \usetikzlibrary{positioning, snakes} 119 | 120 | 121 | \usepackage{tkz-base, tkz-euclide} 122 | \usepackage{yhmath} % \wideparen 123 | \usepackage{longdivision} 124 | \usepackage{polynom} 125 | \usepackage{polynomial} 126 | \usepackage{cancel} 127 | 128 | \renewcommand{\frac}{\dfrac} 129 | \newcommand{\oc}{$^{\circ}{\rm C}$} 130 | \renewcommand{\Vec}{\overrightarrow} 131 | 132 | \usepackage{multicol} 133 | \usepackage{cases} 134 | 135 | \newcommand\parallelogram{% 136 | \mathord{\text{% 137 | \tikz[baseline] 138 | \draw(0em,.1ex)--++(.9em,0ex)--++(.2em,1.2ex)--++(-.9em,0ex)--cycle; 139 | }} 140 | } 141 | 142 | \renewcommand\parallel{\mathrel{/\mskip-2.5mu/}} 143 | 144 | 145 | \newcommand{\eY}{\vec{e}_y} 146 | \newcommand{\eX}{\vec{e}_x} 147 | \newcommand{\eZ}{\vec{e}_z} 148 | \newcommand{\va}{\vec{a}} 149 | \newcommand{\expval}[1]{\left\langle #1 \right\rangle} 150 | 151 | 152 | 153 | 154 | 155 | \begin{document} 156 | 157 | 158 | 159 | 160 | \title{\Huge\bfseries 中学数学实验教材\\第五册(下)} 161 | 162 | 163 | 164 | \author{\Large 中学数学实验教材编写组编} 165 | \date{\Large 1985年5月} 166 | 167 | \maketitle 168 | 169 | 170 | 171 | 172 | \frontmatter 173 | 174 | \input{preface.tex} 175 | \tableofcontents 176 | 177 | 178 | \mainmatter 179 | 180 | % \input{1.tex} 181 | % 182 | % \input{2.tex} 183 | % \input{3.tex} 184 | % \input{4.tex} 185 | 186 | \setcounter{chapter}{4} 187 | \input{5.tex} 188 | 189 | \input{6.tex} 190 | \input{7.tex} 191 | \input{8.tex} 192 | 193 | 194 | 195 | \end{document} 196 | 197 | 198 | \begin{figure}[htp]\centering 199 | \begin{minipage}[t]{0.48\textwidth} 200 | \centering 201 | \begin{tikzpicture}[>=latex, scale=1] 202 | 203 | \end{tikzpicture} 204 | \caption{} 205 | \end{minipage} 206 | \begin{minipage}[t]{0.48\textwidth} 207 | \centering 208 | \begin{tikzpicture}[>=latex, scale=1] 209 | 210 | \end{tikzpicture} 211 | \caption{} 212 | \end{minipage} 213 | \end{figure} 214 | 215 | -------------------------------------------------------------------------------- /8.tex: -------------------------------------------------------------------------------- 1 | 2 | \chapter{空间解析几何初步} 3 | \section{空间向量的坐标运算} 4 | \subsection{空间直角坐标系与向量运算} 5 | 任取一点$O$(图8.1), 一个单位长,通过$O$点建立 6 | 三条互相垂直的数轴,$X$轴、$Y$轴、$Z$轴,并且使这三个数 7 | 轴的正方向构成右手系.这样我们 8 | 就说在空间建立了一个空间右手坐 9 | 标系,并用$OXYZ$来表示.$O$点 10 | 叫做坐标系的原点.$X$轴、$Y$轴、 11 | $Z$轴总称为坐标轴.三个坐标轴每 12 | 两个决定一平面叫做坐标平面.坐标平面共有三个$OXY$、$OYZ$、 13 | $OZX$,它们互相垂直并且把空间分为八个区域,每个区域叫做一个\textbf{卦限}. 14 | 15 | \begin{figure}[htp]\centering 16 | \begin{minipage}[t]{0.48\textwidth} 17 | \centering 18 | \begin{tikzpicture}[>=latex, scale=1] 19 | \draw[<->](0,3.5)node[right]{$Z$}--(0,0)node [below right]{$O$}--(3,0)node[right]{$Y$}; 20 | \draw[dashed](-2,0)--(0,0)--(1.5,1.5); 21 | \draw[dashed](0,0)--(0,-1); 22 | \draw[->](0,0)--(-1.5,-1.5)node[right]{$X$}; 23 | \end{tikzpicture} 24 | \caption{} 25 | \end{minipage} 26 | \begin{minipage}[t]{0.48\textwidth} 27 | \centering 28 | \begin{tikzpicture}[>=latex, scale=1] 29 | \draw[<->](0,3.5)node[right]{$Z$}--(0,0)--(3,0)node[right]{$Y$}; 30 | \draw[->](0,0)--(-1.5,-1.5)node[left]{$X$}; 31 | \tkzDefPoints{0/0/O, 2/0/B, 2/2.5/P', 0/2.5/C, -1/-1/A} 32 | \tkzDefPointsBy[translation= from O to A](B,P',C){B',P,C'} 33 | \tkzDrawPolygon(B',P,C',A) 34 | \tkzDrawPolygon(B,P',C,O) 35 | \tkzDrawSegments(P,P' C,C' B,B') 36 | \tkzLabelPoints[below](A,O,B) 37 | \tkzLabelPoints[right](P) 38 | \tkzLabelPoints[left](C) 39 | \end{tikzpicture} 40 | \caption{} 41 | \end{minipage} 42 | \end{figure} 43 | 44 | 设$P$是空间中任一点,通过$P$点作平面分别与坐标平面 45 | $OYZ$、$OZX$、$OXY$平行(图8.2),并且分别与$X$ 46 | 轴、$Y$轴、$Z$轴相交于$A$、$B$、$C$三点,如果$A$、$B$、$C$在 47 | 各坐标轴上的坐标分别为$x$、$y$、$z$, 则这三个有序实数组 48 | $(x,y,z)$叫$P$点的\textbf{空间坐标}.简称坐标.$P$ 49 | 点的坐标是$(x,y,z)$, 50 | 记作$P(x,y,z)$. $x$、$y$、$z$分别叫做$P$点 51 | 的$X$坐标,$Y$坐标,$Z$坐标. 52 | 53 | \begin{figure}[htp] 54 | \centering 55 | \begin{tikzpicture}[>=latex] 56 | \draw[->](0,2.6)--(0,4)node[right]{$Z$}; 57 | \draw[->](0,0)--(4.5,0)node[right]{$Y$}; 58 | \draw[->](0,0)--(-1.5,-1.5)node[left]{$X$}; 59 | \draw[->, very thick](0,0)--node[right]{$\eZ$}(0,1); 60 | \draw[->, very thick](0,0)--(1,0)node[below]{$\eY$}; 61 | \draw[->, very thick](0,0)--(-.5,-.5)node[right]{$\eX$}; 62 | \node at (0,0)[below right]{$O$}; 63 | \tkzDefPoints{-1/1/A, 2/1/B, 3.3/1.75/C, .3/1.75/D, -1/2/A'} 64 | \tkzDefPointsBy[translation = from A to A'](B,C,D){B',C',D'} 65 | \tkzDrawPolygon[thick](A',B',C',D') 66 | \tkzDrawPolygon[dashed](A,B,C,D) 67 | \tkzDrawSegments[dashed](D,D') 68 | \tkzDrawSegments[thick](A,A' B,B' A,B) 69 | \draw[dashed](0,1)--(0,2.6); 70 | \draw[very thick, ->](B)--node[below]{$a_x\eX$}(C); 71 | \draw[very thick, ->](C)--node[right]{$a_z\eZ$}(C'); 72 | \draw[very thick, ->](C')--node[above]{$a_y\eY$}(D'); 73 | \draw[thick, ->, dashed](B)--node[below]{$\vec{a}$}(D'); 74 | \end{tikzpicture} 75 | \caption{} 76 | \end{figure} 77 | 78 | 79 | 如果沿$X$轴、$Y$轴、$Z$轴的正方向分别引单位向量$\eX$、$\eY$、$\eZ$(图8.3), 那么对空间任一向量$\vec{a}$, 存在唯一的有序数组 80 | $(a_x,a_y,a_z)$使 81 | \[\vec{a}=a_x\eX+a_y\eY+a_z\eZ\] 82 | $(a_x,a_y,a_z)$就叫做$\vec{a}$在 83 | $OXYZ$中的坐标.并简记作 84 | \[\vec{a}=(a_x,a_y,a_z)\] 85 | 其中$a$叫做$\vec{a}$在$X$轴上的坐标分量. 86 | $a_y$叫做$\vec{a}$在$Y$轴上的坐标分量.$a_z$叫做$\vec{a}$在$Z$轴上的坐标分量. 87 | 88 | 如果$\vec{a}=a_x\eX+a_y\eY+a_z\eZ$,那么分别对这个表示式两 89 | 边对$\eX,\eY,\eZ$取内积运算,就可得到 90 | \[\begin{split} 91 | a_x&=\eX\cdot \vec{a}=|\vec{a}|\cos\langle \eX,\vec{a} \rangle \\ 92 | a_y&=\eY\cdot \vec{a}=|\vec{a}|\cos\langle \eY,\vec{a} \rangle \\ 93 | a_z&=\eZ\cdot \vec{a}=|\vec{a}|\cos\langle \eZ,\vec{a} \rangle \\ 94 | \end{split}\] 95 | 96 | 如果$\langle \eX,\vec{a} \rangle=\alpha$, $\langle \eY,\vec{a} \rangle=\beta$, $\langle \eZ,\vec{a} \rangle=\gamma$, 那 97 | 么$\alpha$、$\beta$、$\gamma$确定了$\vec{a}$在空间中的方向.$\alpha$、$\beta$、$\gamma$叫做 98 | $\vec{a}$的方向角,$\cos\alpha$、$\cos\beta$、$\cos\gamma$叫做$\vec{a}$的方向余弦,于是 99 | $\vec{a}$的单位向量 100 | \[\vec{a}_0=(\cos\alpha, \cos\beta, \cos\gamma)\] 101 | 102 | 对空间任一点$P$, 它被相对于$O$点的位置向量所唯一确 103 | 定(图8.4).设 104 | \[\Vec{OP}=x\eX+y\eY+z\eZ\] 105 | 由上述点的坐标和向量坐标的定 106 | 义,$\Vec{OP}$的坐标$(x,y,z)$ 107 | 也就是$P$点的坐标;反之$P$点的 108 | 坐标也是$\Vec{OP}$的坐标.由此可 109 | 见,给定了原点$O$和三个互相垂 110 | 直且构成右手系的单位向量$\eX,\eY,\eZ$,坐标系$OXYZ$也就完全确定了.因此,坐标系 111 | $OXYZ$也可用$[O:\eX,\eY,\eZ]$来表示,$\eX,\eY,\eZ$叫 112 | 做$OXYZ$的基向量. 113 | 114 | \begin{figure}[htp] 115 | \centering 116 | \begin{tikzpicture}[>=latex] 117 | \tkzDefPoints{0/0/A, 2/0/B, 2/2.5/C, 0/2.5/D, -.8/-.8/A'} 118 | \tkzDefPointsBy[translation= from A to A'](B,C,D){B',C',D'} 119 | \tkzDrawPolygon(A',B',C',D') 120 | \tkzDrawPolygon[dashed](A,B,C,D) 121 | \tkzDrawSegments(B,C C,D B,B' C,C' D,D') 122 | \tkzDrawSegments[dashed](A,B A,D A,A') 123 | \draw[->](A')--(-1.5,-1.5)node[left]{$X$}; 124 | \draw[->](B)--(3,0)node[right]{$Y$}; 125 | \draw[->](D)--(0,3.5)node[right]{$Z$}; 126 | \draw[->, dashed](A)--(C'); 127 | \draw[->, very thick](0,0)--(0,1)node[left]{$\eZ$}; 128 | \draw[->, very thick](0,0)--node[above]{$\eY$}(1,0); 129 | \draw[->, very thick](0,0)--(-.5,-.5)node[above]{$\eX$}; 130 | \node at (0,0)[below right]{$O$}; 131 | \node at (C')[right]{$P$}; 132 | \end{tikzpicture} 133 | \caption{} 134 | \end{figure} 135 | 136 | 已知$A(x_1,y_1,z_1)$, $B(x_2,y_2,z_2)$, 则: 137 | \[\begin{split} 138 | \Vec{AB}&=\Vec{OB}-\Vec{OA}\\ 139 | &=x_2\eX+y_2\eY+z_2\eZ-(x_1\eX+y_1\eY+z_1\eZ)\\ 140 | &=(x_2-x_1)\eX+(y_2-y_1)\eY+(z_2-z_1)\eZ\\ 141 | &=(x_2-x_1, y_2-y_1, z_2-z_1) 142 | \end{split}\] 143 | 这就是说\textbf{一个向量的坐标,等于表示它的有向线段终点的坐 144 | 标减去起点的坐标}.例如,已知$A(2,-1,5)$、$B(3, 145 | 2,-7)$, 则 146 | \[\Vec{AB}=[3-2,\; 2-(-1),\; -7-5]=(1,3,-12)\] 147 | 148 | \begin{ex} 149 | \begin{enumerate} 150 | \item 问在$OXYZ$中,哪个坐标平面与$X$轴垂直,哪个坐标 151 | 平面与$Y$轴垂直,哪个坐标平面与$Z$轴垂直? 152 | \item 写出点$P(2,4,3)$在$OXYZ$的三个坐标平面上投 153 | 影点的坐标. 154 | \item 求点$P(3,5,4)$关于坐标平面$OXY$的对称点的坐 155 | 标. 156 | \item 点$P$在$OXYZ$中的坐标平面$OXY$上,若$P$点在平面 157 | 直角坐标系$OXY$中的坐标是$(2,3)$, 求它在 158 | $OXYZ$中的坐标. 159 | \item 写出基向量$\eX,\eY,\eZ$的坐标. 160 | \item 已知$\vec{a}=12$, $\langle\eX ,\vec{a}\rangle=30^{\circ}$, $\langle\eY ,\vec{a}\rangle=45^{\circ}$, $\langle\eZ ,\vec{a}\rangle=60^{\circ}$, 161 | 求$\vec{a}$的坐标. 162 | \item 已知$P(-3,2,4)$, $Q(5,7,-2)$, 求$\Vec{PQ}$与$\Vec{QP}$的坐标. 163 | \item 已知$A(2,-1,5)$, $B(3,2,-1)$用基向量$\eX,\eY,\eZ$表示向量$\Vec{AB}$. 164 | \end{enumerate} 165 | \end{ex} 166 | 167 | \subsection{向量的坐标运算} 168 | 169 | \begin{blk}{定理} 170 | 如果$\vec{a}=(a_x,a_y,a_z)$, $\vec{b}=(b_x, b_y,b_z)$, 171 | $\vec{c}=(c_x,c_y,c_z)$, 那么 172 | \[\begin{split} 173 | \vec{a}\pm \vec{b}&=(a_x,a_y,a_z)\pm (b_x,b_y,b_z) 174 | =(a_x\pm b_x,a_y\pm b_y,a_z\pm b_z)\\ 175 | \lambda\vec{a}&=\lambda(a_x, a_y,a_z)=(\lambda a_x,\lambda 176 | a_y,\lambda a_z)\\ 177 | \vec{a}\cdot \vec{b}&=(a_x, a_y, a_z)\cdot (b_x,b_y,b_z) 178 | =a_xb_x+a_yb_y+a_zb_z\\ 179 | \end{split}\] 180 | \[\vec{a}\x \vec{b}=\begin{vmatrix} 181 | \eX&\eY&\eZ\\ 182 | a_x&a_y&a_z\\ 183 | b_x&b_y&b_z 184 | \end{vmatrix},\qquad \left(\vec{a},\vec{b},\vec{c}\right)=\begin{vmatrix} 185 | a_x&a_y&a_z\\ 186 | b_x&b_y&b_z\\ 187 | c_x&c_y&c_z 188 | \end{vmatrix}\] 189 | \end{blk} 190 | 191 | 192 | 证明留给同学作为练习. 193 | 194 | 下面我们研究如何用向量的坐标来表示向量垂直、平行 195 | 与共面的条件. 196 | 197 | 已知$\vec{a}\parallel \vec{b}\quad (\vec{b}\ne 0)$的充要条件是存在一实数$\lambda$,使 198 | $$\vec{a}=\lambda\vec{b}$$ 199 | 如果 200 | $\vec{a}=(a_x,a_y,a_z)$, $\vec{b}=(b_x,b_y,b_z)$, 那么上面 201 | 条件用坐标表示,即为 202 | \begin{equation} 203 | a_x=\lambda b_x,\qquad a_y=\lambda b_y,\qquad a_z=\lambda b_z 204 | \end{equation} 205 | 或 206 | \begin{equation} 207 | a_x:b_x=a_y:b_y=a_z:b_z 208 | \end{equation} 209 | 这就是说\textbf{两个向量平行的充要条件是它们的坐标成比例}. 210 | 211 | 已知$\vec{a}\bot \vec{b}\quad \Longleftrightarrow \quad \vec{a}\cdot \vec{b}=0$ 212 | 用坐标表示,即为 213 | \begin{equation} 214 | \vec{a}\bot \vec{b}\quad \Longleftrightarrow \quad a_xb_x+a_yb_y+a_zb_z=0 215 | \end{equation} 216 | 217 | 已知$\vec{a}=(a_x,a_y,a_z)$, $\vec{b}=(b_x,b_y,b_z)$, $\vec{c}=(c_x,c_y,c_z)$, 则 218 | \[\vec{a}, \vec{b}, \vec{c}\text{ 共面} \quad \Longleftrightarrow \quad (\vec{a}, \vec{b}, \vec{c})=0\] 219 | 即 220 | \[\vec{a}, \vec{b}, \vec{c}\text{ 共面} \quad \Longleftrightarrow \quad \begin{vmatrix} 221 | a_x&a_y&a_z\\b_x&b_y&b_z\\c_x&c_y&c_z 222 | \end{vmatrix}=0\] 223 | 224 | \begin{example} 225 | 已知 $\vec{a}=(1,1,1)$, $\vec{b}=(3,-1,2)$, 226 | $\vec{c}=(1,-3,0)$. 227 | 求证:$\vec{a},\vec{b},\vec{c}$共面. 228 | \end{example} 229 | 230 | \begin{solution} 231 | \[\because\quad (\vec{a},\vec{b},\vec{c})=\begin{vmatrix} 232 | 1&1&1\\3&-1&2\\1&-3&0 233 | \end{vmatrix}=0\] 234 | 235 | $\therefore\quad \vec{a},\vec{b},\vec{c}$共面. 236 | \end{solution} 237 | 238 | 239 | \begin{ex} 240 | \begin{enumerate} 241 | \item 已知$\vec{a}=(-1,2,3)$, $\vec{b}=(2,-4,-6)$, 求证: 242 | $\vec{a}\parallel \vec{b}$. 243 | \item 试证下面各对向量线性相关. 244 | \begin{enumerate} 245 | \item $\vec{a}=(2,-1,-2),\qquad \vec{b}=(6,-3,-6)$ 246 | \item $\vec{a}=(-3,-5,4),\qquad \vec{b}=(6,10,-8)$ 247 | \end{enumerate} 248 | 249 | \item 已知$\vec{a}=(2,3,4)$, $\vec{b}=(-3,-6,6)$, 求证$\vec{a}\bot \vec{b}$. 250 | 251 | \item 设$\vec{a}=(2,-1,4)$, $\vec{b}=(-4,-5.-1)$, 求使 252 | $\vec{a}-k\vec{b}$垂直于$\vec{b}$的实数$k$的值. 253 | 254 | \item 已知$\vec{a}=(-5,2,3)$, $\vec{b}=(0,-3,2)$, $\vec{c}=(5, 255 | -2,-3)$, 求证:$\vec{a},\vec{b},\vec{c}$三个向量共面. 256 | \item 已知$P(x,y,z)$, $P_1(x_1,y_1,z_1)$, $P_2(x_2,y_2, 257 | z_2)$, $P_3(x_3, У_3,z_3)$. 求证这四点共面的充要条件是 258 | \[\begin{vmatrix} 259 | x-x_1 &y-y_1&z-z_1\\ 260 | x_2-x_1 &y_2-y_1&z_2-z_1\\ 261 | x_3-x_1 &y_3-y_1&z_3-z_1\\ 262 | \end{vmatrix}=0\quad \text{或}\quad \begin{vmatrix} 263 | x&y&z&1\\x_1 & y_1&z_1& 1\\ 264 | x_2 & y_2&z_2& 1\\x_3 & y_3&z_3& 1 265 | \end{vmatrix}=0\] 266 | \end{enumerate} 267 | \end{ex} 268 | 269 | \subsection{空间解析几何的基本问题} 270 | \begin{blk}{问题1} 271 | 求有向线段定比分点的坐标. 272 | \end{blk} 273 | 274 | 已知$P_1(x_1,y_1,z_1)$、$P_2(x_2,y_2,z_2)$, 如果 275 | $P(x,y,z)$, 按定比$\mu$分割$\Vec{P_1P_2}$, 那么 276 | \[\Vec{OP}=\frac{\Vec{OP_1}+\mu \Vec{OP_2}}{1+\mu}\] 277 | 换用坐标表示,即为 278 | \begin{equation} 279 | \begin{cases} 280 | x=\frac{x_1+\mu x_2}{1+\mu}\\ 281 | y=\frac{y_1+\mu y_2}{1+\mu}\\ 282 | z=\frac{z_1+\mu z_2}{1+\mu}\\ 283 | \end{cases} 284 | \end{equation} 285 | (8.4)式就是求$\Vec{P_1P_2}$的\textbf{定比分点坐标的计算公式}.当 286 | $\mu=1$时,$P$点是$\overline{P_1P_2}$的中点,$P$点的坐标是 287 | \begin{equation} 288 | \begin{cases} 289 | x=\frac{x_1+ x_2}{2}\\ 290 | y=\frac{y_1+ y_2}{2}\\ 291 | z=\frac{z_1+ z_2}{2}\\ 292 | \end{cases} 293 | \end{equation} 294 | 公式(8.5)又叫做中点公式. 295 | 296 | 297 | \begin{example} 298 | 已知$A(-1,2,2)$, $B(-4,2,5)$, 点$P$按定比 299 | $\mu=2$分割$\Vec{AB}$, 求$P(x,y,z)$. 300 | \end{example} 301 | 302 | \begin{solution} 303 | 由于$\mu=2$,因此: 304 | \[\begin{split} 305 | x&=\frac{-1+2\x (-4)}{1+2}=-3\\ 306 | y&=\frac{2+2\x 2}{1+2}=2\\ 307 | z&=\frac{2+2\x 5}{1+2}=4\\ 308 | \end{split}\] 309 | 即:$P(-3,2,4)$ 310 | \end{solution} 311 | 312 | \begin{blk}{问题2} 313 | 求向量长度和两点间距离公式. 314 | \end{blk} 315 | 316 | 若$\vec{a}=(a_x,a_y,a_z)$, 则 317 | \begin{align} 318 | |\vec{a}|^2&=\vec{a}\cdot \vec{a} =a^2_x+a^2_y+a^2_z\nonumber\\ 319 | |\vec{a}|&=\sqrt{a^2_x+a^2_y+a^2_z} 320 | \end{align} 321 | (8.6)式就是求\textbf{向量$\vec{a}$的长度的计算公式}. 322 | 323 | 若$A(x_1,y_1,z_1)$、$B(x_2,y_2,z_2)$,则: 324 | \begin{equation} 325 | |\Vec{AB}|=\sqrt{(x_2-x_1)^2+(y_2-y_1)^2+(z_2-z_1)^2} 326 | \end{equation} 327 | (8.7)式就是求空间任意\textbf{两点间的距离公式}. 328 | 329 | 330 | \begin{blk} 331 | {问题3} 求一向量的方向余弦. 332 | \end{blk} 333 | 334 | 若$\vec{a}=(a_x,a_y,a_z)$, $\alpha,\beta,\gamma$为$\vec{a}$的方向角,则: 335 | \[a_x=|\vec{a}|\cos\alpha,\qquad a_y=|\vec{a}|\cos\beta,\qquad a_z=|\vec{a}|\cos\gamma\] 336 | 于是得: 337 | \begin{equation} 338 | \begin{cases} 339 | \cos\alpha=\frac{a_x}{|\vec{a}|}=\frac{a_x}{\sqrt{a^2_x+a^2_y+a^2_z}}\\ 340 | \cos\beta=\frac{a_y}{|\vec{a}|}=\frac{a_y}{\sqrt{a^2_x+a^2_y+a^2_z}}\\ 341 | \cos\gamma=\frac{a_z}{|\vec{a}|}=\frac{a_z}{\sqrt{a^2_x+a^2_y+a^2_z}}\\ 342 | \end{cases} 343 | \end{equation} 344 | (8.8)式就是\textbf{向量$\vec{a}$的方向余弦的计算公式}. 345 | 346 | 把(8.7)式两边平方加起来,得 347 | \[\cos^2\alpha+\cos^2\beta+\cos^2\gamma=1\] 348 | 这就是说,\textbf{任何一个向量的方向余弦的平方和恒等于1}. 349 | 350 | 若$\vec{a}=(a_x,a_y,a_z)$,则$\vec{a}$的单位向量 351 | \[\begin{split} 352 | \vec{a}_0=\frac{\vec{a}}{|\vec{a}|}&=\left(\frac{a_x}{|\vec{a}|},\frac{a_y}{|\vec{a}|},\frac{a_z}{|\vec{a}|}\right)\\ 353 | &=(\cos\alpha,\cos\beta,\cos\gamma) 354 | \end{split}\] 355 | 这就是说,\textbf{空间任一向量的单位向量的坐标分量正好等于它 356 | 的方向余弦}. 357 | 358 | \begin{example} 359 | 求$\vec{a}=(2,-3,1)$的方向余弦和它的单位向 360 | 量$\vec{a}_0$的坐标. 361 | \end{example} 362 | 363 | \begin{solution} 364 | 由于:$|\vec{a}|=\sqrt{2^2+(-3)^2+1^2}=\sqrt{14}$ 365 | 366 | $\therefore\quad \cos\alpha=\frac{2}{\sqrt{14}},\qquad \cos\beta=\frac{-3}{\sqrt{14}},\qquad \cos\gamma=\frac{1}{\sqrt{14}}$ 367 | \[\vec{a}_0=\left(\frac{2}{\sqrt{14}},\frac{-3}{\sqrt{14}},\frac{1}{\sqrt{14}}\right)\] 368 | \end{solution} 369 | 370 | \begin{blk} 371 | {问题4} 372 | 求两个向量的夹角. 373 | \end{blk} 374 | 375 | 如果$\vec{a}=(a_x,a_y,a_z)$, $\vec{b}=(b_x,b_y,b_z)$,那么 376 | \begin{equation} 377 | \begin{split} 378 | \cos\langle \vec{a},\vec{b}\rangle &=\frac{\vec{a}\cdot \vec{b}}{|\vec{a}||\vec{b}|}\\ 379 | &=\frac{a_xb_x+a_yb_y+a_zb_z}{\sqrt{a^2_x+a^2_y+a^2_z}\cdot \sqrt{b^2_x+b^2_y+b^2_z}} 380 | \end{split} 381 | \end{equation} 382 | 公式(8.9)就是求向量夹角的计算公式. 383 | 384 | 385 | 386 | \begin{example} 387 | 已知$\vec{a}=(1,1,0)$, $\vec{b}=(1,0,1)$, 388 | 求$\langle \vec{a},\vec{b}\rangle$. 389 | \end{example} 390 | 391 | \begin{solution} 392 | \[\cos\langle \vec{a}\cdot \vec{b} \rangle=\frac{1\x 1+1\x 0+0\x 1}{\sqrt{1^2+1^2+0^2}\cdot \sqrt{1^2+1^2+0^2}}=\frac{1}{2} \] 393 | $\therefore\quad \langle \vec{a},\vec{b}\rangle=\frac{\pi}{3}$ 394 | \end{solution} 395 | 396 | \begin{ex} 397 | \begin{enumerate} 398 | \item 已知$A(3,5,-7)$, $B(-2,4,3)$, 点$P$按定 399 | 比$\mu=-2$分割$\Vec{AB}$, 求$P$点的坐标. 400 | \item 已知$P(3,-4,1)$, $Q(0,2,-3)$, 点$A$按定 401 | 比$\mu=2$分割$\Vec{QP}$, 求$A$点的坐标. 402 | \item 已知$A(0,-1,1)$, $B(2,1,-3)$, 求$\overline{AB}$中点的坐 403 | 标. 404 | \item 已知$\triangle ABC$的两个顶点$A(-4,-1,2)$, $B(3, 405 | 5,-16)$, $\overline{AC}$边的中点在$Y$轴上, 406 | $\overline{BC}$边的中点在 407 | $OZX$平面上,求第三顶点$C$的坐标. 408 | \item 已知$A(3,-1,0)$、$B(2,1,-3)$, 求$A$、$B$ 409 | 两点间的距离. 410 | \item 已知$|\vec{a}|=10$, $\langle \eX,\vec{a} \rangle=60^{\circ}$, $\langle \eY,\vec{a} \rangle=60^{\circ}$ 411 | 412 | 求$\langle \eZ,\vec{a} \rangle=60^{\circ}$和$\vec{a}$的坐标. 413 | \item 已知$\vec{a}\parallel \vec{b}$, $|\vec{a}|=10$, $\vec{b}=(3,-3,3)$, 求$\vec{a}$的坐标. 414 | 415 | \item 已知$\vec{a}=(-1,2,3)$, $\vec{b}=(2,5,4)$, 求$\langle \vec{a},\vec{b} \rangle$ 416 | 417 | \item 已知$\vec{a}=(2,-3,5)$, $\vec{b}=(-4,2,6)$, 求证 418 | $\vec{a}\nparallel \vec{b}$. 419 | \end{enumerate} 420 | \end{ex} 421 | 422 | \section*{习题8.1} 423 | \addcontentsline{toc}{subsection}{习题8.1} 424 | 425 | \begin{enumerate} 426 | \item 如果向量$\vec{a}$、$\vec{b}$、$\vec{c}$分别平行于$X$轴、$Y$轴、$Z$轴,问 427 | 它们的坐标各有什么特点? 428 | \item 429 | 如果$\vec{a}$的$x$坐标是0, 那么$\vec{a}$与哪个平面平行. 430 | \item 已知$\vec{a}=(2,-1,3)$、$\vec{b}=(-3,0,4)$, 求满足下 431 | 列关系的向量$\vec{c}$的坐标. 432 | \begin{multicols}{2} 433 | \begin{enumerate} 434 | \item $3\vec{a}+2\vec{c}=\vec{b}$ 435 | \item $\vec{a}-3\vec{c}=2\vec{b}$ 436 | \item $\vec{a}-2\vec{c}=3\vec{b}-\vec{c}$ 437 | \item $2(3\vec{a}-\vec{c})+\vec{b}=0$ 438 | \end{enumerate} 439 | \end{multicols} 440 | \item 已知$A(2,-1,7)$, $B(4,5,-2)$, 求每个坐标平 441 | 面分割$\Vec{AB}$的比值. 442 | \item 已知$A(2,3,6)$, $B(5,2,8)$, 直线$AB$上有$C$点 443 | 使$B$点为$\overline{AC}$的中点,求$C(x,y,z)$. 444 | \item 已知$A=(x_1,y_1,z_1)$、$B=(x_2,y_2,z_2)$、 445 | $C=(x_3,y_3,z_3)$, 求$\triangle ABC$的重心. 446 | 447 | \item 已知$\vec{a}=(1,2,-2)$, $\vec{b}=(3,4,2)$, 448 | $\vec{c}=(-2,-4,4)$, 求证:$\vec{a}$、$\vec{b}$、$\vec{c}$线性相关. 449 | 450 | \item 已知$A(4,1,3)$, $B(2,-5,1)$, $C(3,7,-5)$. 求向量$\Vec{AB},\Vec{BA},\Vec{AC},\Vec{BC}$的坐标和长度 451 | (精确到0.01). 452 | \item 已知$A(1,1,\sqrt{2})$, 求$\Vec{OA}$与三个坐标轴的夹角. 453 | \item 已知$\vec{a}=(-1,1,0)$, $\vec{b}=(1,-2,2)$, 454 | 求$\langle \vec{a},\vec{b}\rangle$. 455 | \end{enumerate} 456 | 457 | \section{空间的平面~~直线与球面方程} 458 | 459 | \subsection{空间的平面方程} 460 | 461 | 已知非零向量$\vec{n}=(a,b,c)$和定点$P_0(x_0,y_0,z_0)$, 462 | 过$P_0$点作平面$\pi$与$\vec{n}$垂直,求平面 463 | $\pi$的方程. 464 | 465 | \begin{figure}[htp] 466 | \centering 467 | \begin{tikzpicture}[>=latex] 468 | \draw[->](0,0)node[below]{$O$}--(4,0)node[right]{$Y$}; 469 | \draw[->](0,0)--(0,4)node[right]{$Z$}; 470 | \draw[->](0,0)--(-1,-1)node[right]{$X$}; 471 | \tkzDefPoints{.5/2.5/A, 3/2/B, -.5/1/A', 0/0/O} 472 | \tkzDefPointsBy[translation = from A to A'](B){B'} 473 | \tkzDrawPolygon[fill=white](A,B,B',A') 474 | \tkzDefPoint(30:2.6){P} 475 | \tkzDefPoint(60:2){P_0} 476 | \tkzDrawSegments[dashed, ->](O,P O,P_0) 477 | \tkzDrawSegments[->](P_0,P) 478 | \tkzLabelPoints[above](P_0,P) 479 | \tkzDefPointWith[linear, K=.5](O,P) \tkzGetPoint{P1} 480 | \tkzDefPointWith[linear, K=.43](O,P_0) \tkzGetPoint{P2} 481 | 482 | 483 | \draw[->, thick](.65,2.2)--node[left]{$\vec{n}$}+(72:1); 484 | 485 | \tkzDrawSegments (O,P1 O,P2) 486 | \end{tikzpicture} 487 | \caption{} 488 | \end{figure} 489 | 490 | 设$P(x,y,z)$为平面$\pi$上一动点,因为 491 | $\Vec{P_0P}\bot\vec{n}$, 所以 492 | $\Vec{P_0P}\cdot \vec{n}=0$, 493 | 即: 494 | \begin{equation} 495 | \left(\Vec{OP}-\Vec{OP_0}\right)\cdot\vec{n}=0 496 | \end{equation} 497 | 反之,如果$P(x,y,z)$满足(8.10)式,那么$P$点一定在 498 | 平面$\pi$上,所以(8.10)式就是\textbf{平面$\pi$的向量方程}. 499 | 500 | (8.10)式用坐标表示即可写为 501 | \begin{equation} 502 | a(x-x_0)+b(y-y_0)+c(z-z_0)=0 503 | \end{equation} 504 | (8.11)式就叫做\textbf{平面的点法向式方程}.其中$\vec{n}=(a,b,c)$, 505 | 叫做平面$\pi$的一条法线向量. 506 | 507 | 如果令$d=-(ax_0+by_0+cz_0)$, 那么(8.11)式又可 508 | 写为 509 | \begin{equation} 510 | ax+by+cz+d=0 511 | \end{equation} 512 | 方程(8.12)又叫做\textbf{平面的普通方程},其中$a,b,c$至少有 513 | 一个不为零. 514 | 515 | 显然,如果$\vec{n}=(a,b,c)$是平面$\pi$的一个法线向 516 | 量,那么对任何非零常数$k$, $k\vec{n}$也是$\pi$的法线向量.这 517 | 样,若取$k\vec{n}$作为平面的法线向量,则$\pi$的方程还可写为 518 | \[k(ax+by+cz+d)=0\] 519 | 因此,同一个平面方程,仅仅相差一个常数因子. 520 | 521 | 由方程(8.12)可以看出,平面的方程是$x,y,z$的一 522 | 次方程;反之,如果设$(x_0,y_0,z_0)$是三元一次方程 523 | $ax+by+cz+d=0$ 524 | 的一个解,则 525 | \[ax_0+by_0+cz_0+d=0\] 526 | 两式相减,得 527 | \begin{equation} 528 | a(x-x_0)+b(y-y_0)+c(z-z_0)=0 529 | \end{equation} 530 | 如果建立空间直角坐标系,作$\Vec{OP_0}=(x_0,y_0,z_0)$, 531 | $\vec{n}=(a,b,c)$, 那么(8.13)式就是通过$P_0$且垂直于$\vec{n}$ 532 | 的一个平面方程,这就是说,\textbf{任何一个三元一次方程都表示 533 | 一个平面}.这样,在空间解析几何中,一个平面和一个三元 534 | 一次方程是同一码事. 535 | 536 | 由以上分析,我们还可得到一个结论,即,\textbf{任给一个平 537 | 面$\pi:\; ax+by+cz+d=0$, 其中$x,y,z$的系数向量 538 | $\vec{n}=(a,b,c)$是平面$\pi$的一个法线向量. 539 | } 540 | 541 | \begin{example} 542 | 求通过点$P(2,-1,3)$且垂直于$\vec{n}=(2,-1,5)$ 543 | 的平面方程. 544 | \end{example} 545 | 546 | \begin{solution} 547 | 由平面的点法式方程,得所求平面方程为 548 | \[2(x-2)+(-1)[y-(-1)]+5(z-3)=0\] 549 | 整理得 550 | \[2x-y+5z-20=0\] 551 | \end{solution} 552 | 553 | 554 | 555 | 556 | \begin{example} 557 | 已知$A(x_1,y_1,z_1)$, $B(x_2,y_2,z_2)$, 558 | $C(x_3,y_3,z_3)$三点不共线.求通过$A$、$B$、$C$的平面方 559 | 程. 560 | \end{example} 561 | 562 | \begin{solution} 563 | 设$P(x,y,z)$为所求平面的一个动点,则$P$点 564 | 与$A$、$B$、$C$三点共面的充要条件是 565 | \[\begin{vmatrix} 566 | x-x_1&y-y_1&z-z_1\\ 567 | x_2-x_1&y_2-y_1&z_2-z_1\\ 568 | x_3-x_1&y_3-y_1&z_3-z_1\\ 569 | \end{vmatrix}=0\] 570 | 这就是\textbf{通过$A$、$B$、$C$三点的平面方程},叫做平面方程的三 571 | 点式. 572 | \end{solution} 573 | 574 | 575 | 576 | 577 | \begin{example} 578 | 求通过原点和两点$(2,0,1)$, $(0,1,3)$ 579 | 的平面方程. 580 | \end{example} 581 | 582 | \begin{solution} 583 | \textbf{方法1:} 由平面方程的三点式,得 584 | \[\begin{vmatrix} 585 | x-0&y-0&z-0\\ 586 | 2-0&0-0&1-0\\ 587 | 0-0&1-0&3-0\\ 588 | \end{vmatrix}=0\] 589 | 展开化简,得 590 | \[x+6y-2z=0\] 591 | \textbf{方法2:} 592 | 设所求的平面方程为$ax+by+cz+d=0$, 593 | 把已知三点的坐标,代入上面方程,得 594 | \[\begin{cases} 595 | d=0\\ 596 | 2a+c=0\\ 597 | b+3c=0 598 | \end{cases}\] 599 | 解此方程组,得 600 | \[a=-\frac{1}{2}c,\qquad b=-3c,\qquad d=0\] 601 | 所以,所求的平面方程为 602 | \[\frac{1}{2}cx-3cy+cz=0\] 603 | 即:$x+6y-2z=0$. 604 | \end{solution} 605 | 606 | \begin{example} 607 | 求通过点$(1,2,3)$且平行于平面 608 | $2x+y-z+3=0$的平面方程. 609 | \end{example} 610 | 611 | \begin{solution} 612 | 已知平面的一个法线向量是$\vec{n}=(2,1,-1)$, 613 | 它与所求平面垂直,由平面的点法向式方程,得所求方程为 614 | \[2(x-1)+1(y-2)+(-1)(z-3)=0\] 615 | 整理,得 616 | \[2x+y-z-1=0\] 617 | \end{solution} 618 | 619 | \begin{example} 620 | 求点$P_1(x_1,y_1,z_1)$到平面$\pi:\; ax+by+cz+d=0$的距离$d$(图8.6). 621 | \end{example} 622 | 623 | \begin{figure}[htp] 624 | \centering 625 | \begin{tikzpicture}[>=latex, scale=1.2] 626 | \draw(0,0)--(3,0)--(3.5,1)--(.5,1)--(0,0); 627 | \draw[->, thick](3,.5)--(3,1.5)node[right]{$\vec{n}$}; 628 | \tkzDefPoints{1/0/A, 1.5/1/B, 1.9/.5/P_0} 629 | \tkzDefPointWith[linear, K=1.6](A,B) \tkzGetPoint{P_1} 630 | \tkzDefPointWith[linear, K=1.6](B,A) \tkzGetPoint{S} 631 | \tkzDefPointWith[linear, K=0.6](S,P_0) \tkzGetPoint{S1} 632 | \tkzDrawSegments[dashed](A,B P_0,S1) 633 | \tkzDrawSegments(P_1,B S,A P_1,P_0 S,S1) 634 | \tkzLabelPoints[right](P_1,P_0) 635 | \end{tikzpicture} 636 | \caption{} 637 | \end{figure} 638 | 639 | \begin{solution} 640 | 过$P_1$作$P_1P_0$垂直平面$\pi$ 641 | 于$P_0$点,则 642 | \[d=|\Vec{P_0P_1}|\] 643 | 设$P_0$的坐标为$(x_0,y_0,z_0)$. 则 644 | \[|\Vec{P_0P_2}|=|\Vec{P_0P_1} \cdot \vec{n}_0|\] 645 | 其中$\vec{n}_0$是$\pi$的单位法向量.换用坐标表示,即为 646 | \[|\Vec{P_0P_1}|=\frac{a(x_1-x_0)+b(y_1-y_0)+c(z_1-z_0)}{\sqrt{a^2+b^2+c^2}}\] 647 | 因为$P_0\in \pi$, 所以 648 | \[ax_0+by_0+cz_0+d=0\] 649 | 其中: $d=-(ax_0+by_0+cz_0)$, 代入上式,得 650 | \[|\Vec{P_0P_1}|=\frac{ax_1+by_1+cz_1+d}{\sqrt{a^2+b^2+c^2}}\] 651 | \[d=\frac{|ax_1+by_1+cz_1+d|}{\sqrt{a^2+b^2+c^2}}\] 652 | \end{solution} 653 | 654 | 例8.9说明,如果要求一点到一平面的距离,只要把这 655 | 一点的坐标代入平面方程.取绝对值,再除以系数向量的长 656 | 度就可求出. 657 | 658 | \begin{ex} 659 | \begin{enumerate} 660 | \item 求三个坐标平面的方程. 661 | \item 求过点$A(1,2,-3)$, 以$\vec{n}=(1,-3,2)$为法 662 | 线向量的方程. 663 | \item 求过点$P_0(x_0,y_0,z_0)$且垂直于$X$轴的平面方程. 664 | \item 已知两点$A(2,3,4)$, $B(-2,4,3)$, 求$\overline{AB}$ 665 | 的垂直平分面的方程. 666 | \item 求通过点$P_0(x_0,y_0,z_0)$且平行于$OXY$平面的方程. 667 | \item 证明方程$ax+by+cz=0$, 是通过原点的平面,其中 668 | $a$、$b$、$c$至少有一个不为零 669 | \item 求过原点和两点$(1,0,-1)$, $(0,2,3)$的平 670 | 面方程. 671 | \item 求过点$(3,5,-2)$且平行于平面$2x-y+3z=0$ 672 | 的平面方程. 673 | \item 求点$(3,-2,5)$到平面$3x-4y-z+3=0$的距 674 | 离. 675 | \end{enumerate} 676 | \end{ex} 677 | 678 | 679 | \subsection{空间的直线方程} 680 | 已知,一定点$P_0(x_0,y_0,z_0)$ 681 | 和一向量$\vec{a}=(a_1,a_2,a_3)$, 求过 682 | $P_0$且平行于向量$\vec{a}$的直线方程. 683 | 684 | \begin{figure}[htp] 685 | \centering 686 | \begin{tikzpicture}[>=latex] 687 | \draw[->](0,0)node[below]{$O$}--(3.5,0)node[right]{$Y$}; 688 | \draw[->](0,0)--(0,3)node[right]{$Z$}; 689 | \draw[->](0,0)--(-1.25,-1.25)node[right]{$X$}; 690 | \draw[domain=-1:2.5, samples=10, thick]plot(\x, {1.5-\x})node[right]{$\ell$}; 691 | 692 | \tkzDefPoints{0/0/O, 0.6/0.9/P_0, -.5/2/P} 693 | \tkzDrawSegments[->, thick](O,P O,P_0) 694 | \draw[thick,->](0,0)--node[below]{$\vec{a}$}(135:1); 695 | \tkzLabelPoints[above](P,P_0) 696 | \end{tikzpicture} 697 | \caption{} 698 | \end{figure} 699 | 700 | 设$P(x,y,z)$是所求直线 701 | $\ell$上一动点,则存在一实数$t$使 702 | \[\Vec{P_0P}=t\vec{a},\qquad \Vec{OP}=\Vec{OP_0}+t\vec{a}\] 703 | 换用坐标表示,即为 704 | \begin{equation} 705 | \begin{cases} 706 | x=x_0+a_1t\\ 707 | y = y_0+a_2t\\ 708 | z=z_0+a_3t 709 | \end{cases} 710 | \end{equation} 711 | (8.14)式叫做直线$\ell$的\textbf{参数方程}.$t$叫做\textbf{参数}. 712 | 713 | 如果$a_1,a_2,a_3$都不为零,从(8.14)式消去参数$t$, 得 714 | \[\frac{x-x_0}{a_1}=\frac{y-y_0}{a_2}=\frac{z-z_0}{a_3}\] 715 | (8.15)式叫做$\ell$的\textbf{点、方向式方程}又叫\textbf{对称式方程}.其中 716 | $\vec{a}=(a_1,a_2,a_3)$叫做$\ell$的方向向量.如果取$\vec{a}$的单位向量 717 | \[\vec{a}_0=(\cos\alpha, \cos\beta,\cos\gamma)\] 718 | 作为方向向量,则$\ell$的方程为 719 | \[\frac{x-x_0}{\cos\alpha}=\frac{y-y_0}{\cos\beta}=\frac{z-z_0}{\cos\gamma}\] 720 | $\cos\alpha, \cos\beta,\cos\gamma$又叫做有向\textbf{直线$\ell$的方向余弦}. 721 | 722 | 如果直线$\ell$通过两点$P_1(x_1,y_1,z_1)$, $P_2(x_2,y_2,z_2)$, 723 | 则直线$\ell$的方向向量可取 724 | \[\Vec{P_1P_2}=(x_2-x_1,y_2-y_1,z_2-z_1)\] 725 | 这时直线$\ell$的方程可写为 726 | \begin{equation} 727 | \frac{x-x_1}{x_2-x_1}=\frac{y-y_1}{y_2-y_1}=\frac{z-z_1}{z_2-z_1} 728 | \end{equation} 729 | 方程(8.16)一般叫做直线的\textbf{两点式方程}. 730 | 731 | 732 | 733 | \begin{example} 734 | 求通过$P_0(1,-1,2)$, 且和向量$\vec{a}=(2,3,1)$平 735 | 行的直线$\ell$的方程. 736 | \end{example} 737 | 738 | 739 | \begin{solution} 740 | 由直线的对称式方程可得直线$\ell$的方程为 741 | \[\frac{x-1}{2}=\frac{y+1}{3}=z-2\] 742 | \end{solution} 743 | 744 | \begin{example} 745 | 求通过两个不同点$P_1(x_1,y_1,z_1)$, $P_2(x_2, 746 | y_2,z_2)$的直线的参数方程. 747 | \end{example} 748 | 749 | 750 | \begin{solution} 751 | 取直线$P_1P_2$的方向向量 752 | \[ \Vec{P_1P_2}=(x_2-x_1, y_2-У_1,z_2-z_1)\] 753 | 由直线的参数方程.可得直线$P_1P_2$的参数方程为 754 | \[\begin{cases} 755 | x=x_1+(x_2-x_1)t\\ 756 | y=y_1+(y_2-y_1)t\\ 757 | z=z_1+(z_2-z_1)t 758 | \end{cases}\] 759 | \end{solution} 760 | 761 | \begin{example} 762 | 已知$P_1(5,0,1)$, $P_2(5,6,4)$, 求直线 763 | $P_1P_2$的参数方程. 764 | \end{example} 765 | 766 | 767 | \begin{solution} 768 | 由例8.11, 可知直线$P_1P_2$的参数方程为 769 | \[\begin{cases} 770 | x=5\\ 771 | y=6t\\ 772 | z=1+3t\\ 773 | \end{cases}\] 774 | \end{solution} 775 | 776 | 在例8.12中,由于$\Vec{P_1P_2}=(0,6,3)$, 其中$x$坐标为 777 | 零,因此直线$P_1P_2$不能写为对称式方程,但确能用参数方 778 | 程来表达.由此可看到,直线的参数方程比较优越. 779 | 780 | 781 | \begin{ex} 782 | \begin{enumerate} 783 | \item 求通过点$P_0(-1,2,-3)$且平行于向量$\vec{s}=(2,3,-5)$ 784 | 的直线方程. 785 | \item 求通过$P_0(2,3,1)$, $P_1(-1,-2,3)$的直线方程. 786 | \item 求通过点$(2,3,1)$且和$X$轴平行的直线方程. 787 | \item 求过点$(2,-3,7)$, 其方向向量为$(2,0,3)$的直 788 | 线方程. 789 | \item 求直线$2x-6=4-y=2-5$的方向向量. 790 | \item 求平行于两平面$x-2y+5z+2=0$和$3x+y-z 791 | +5=0$的交线,且通过原点的直线方程. 792 | \end{enumerate} 793 | \end{ex} 794 | 795 | 796 | \subsection{球面方程} 797 | 空间一动点$P(x,y,z)$在以$A(a,b,c)$为球 798 | 心,$R$为半径的球面上的充要条件是 799 | \[|\Vec{OP}-\Vec{OA}|=R\] 800 | 或 801 | \[(\Vec{OP}-\Vec{OA})\cdot (\Vec{OP}-\Vec{OA})=R^2\] 802 | 换用坐标表示,条件可写为 803 | \begin{equation} 804 | (x-a)^2+(y-b)^2+(z-c)^2=R^2 805 | \end{equation} 806 | 807 | \begin{figure}[htp] 808 | \centering 809 | \begin{tikzpicture}[>=latex] 810 | \draw[<->](0,4)node[right]{$Z$}--(0,0)node[below right]{$O$}--(3.5,0)node[right]{$Y$}; 811 | \draw[->](0,0)--(-1,-1)node[right]{$X$}; 812 | \tkzDefPoints{1.5/1.75/A, 0/0/O, 1/2.62/P} 813 | 814 | \draw[thick](A) circle (1); 815 | \draw[dashed](A) ellipse[x radius=1, y radius=.4]; 816 | \draw[thick](2.5,1.75) arc [x radius=1, y radius=.4,start angle =0, end angle =-180]; 817 | \tkzDrawSegments[dashed, ->](O,P A,P O,A) 818 | \tkzLabelPoints[right](A) 819 | \tkzLabelPoints[above](P) 820 | \end{tikzpicture} 821 | \caption{} 822 | \end{figure} 823 | 824 | 825 | 方程(8.16)就是以$A(a,b,c)$为球心,以$R$为半径的\textbf{球面 826 | 方程}.当$A$点在原点,球面方程变为 827 | \[ x^2+y^2+z^2=R^2\] 828 | 829 | 830 | 831 | \begin{ex} 832 | \begin{enumerate} 833 | \item 求以$A(1,2,-2)$为球心,3为半径的球面方程. 834 | \item 求球心在原点,半径等于5的球面方程. 835 | \item 设一动点$Q$在以$A(0,4,0)$为球心,2为半径的球面 836 | 上变动,求$\overline{OQ}$中点的轨迹. 837 | \end{enumerate} 838 | \end{ex} 839 | 840 | \section*{习题8.2} 841 | \addcontentsline{toc}{subsection}{习题8.2} 842 | 843 | \begin{enumerate} 844 | \item 求满足以下条件的平面方程. 845 | \begin{enumerate} 846 | \item 通过点$P_0(5,3,4)$且垂直于向量$\vec{n}=(1,1,1)$; 847 | \item 通过坐标原点且垂直于$\vec{n}=\left(-\frac{1}{3},\frac{2}{3},-\frac{2}{3}\right)$; 848 | \item 垂直于$\vec{n}=\left(\frac{1}{2},\frac{\sqrt{3}}{2},0\right)$且与原点的距离等于5. 849 | \end{enumerate} 850 | 851 | \item 说出如下方程表示的平面的几何特征. 852 | \begin{multicols}{3} 853 | \begin{enumerate} 854 | \item $x=2$ \item $x=y$ \item $x+y+z=1$ 855 | \end{enumerate} 856 | \end{multicols} 857 | 858 | \item 求证通过三点$A(a,0,0)$, $B(0,b,0)$, $C(0, 859 | 0,c)$的平面方程为 860 | \[\frac{x}{a}+\frac{y}{b}+\frac{z}{c}=1\] 861 | 862 | \item 如图,试写出长方体$ABC 863 | D-A'B'C'D'$的各侧面,底面的平面方程以及各 864 | 棱所在的直线方程.已知 865 | $\overline{AB}=a$, $\overline{AD}=b$, 866 | $\overline{AA'}=c$. 867 | 868 | \begin{figure}[htp] 869 | \centering 870 | \begin{tikzpicture}[>=latex, scale=1.3] 871 | \tkzDefPoints{0/0/A, -.5/-.5/B, 1/-.5/C, 1.5/0/D, 0/2/A'} 872 | \tkzDefPointsBy[translation= from A to A'](B,C,D){B',C',D'} 873 | \tkzDrawPolygon[thick](A',B',C',D') 874 | \tkzDrawPolygon[dashed](A,B,C,D) 875 | \tkzDrawSegments[thick](B,C C,D B,B' C,C' D,D') 876 | \tkzDrawSegments[dashed](A,B A,D A,A') 877 | \draw[->](B)--(-1,-1)node[left]{$X$}; 878 | \draw[->](D)--(2.5,0)node[right]{$Y$}; 879 | \draw[->](A')--(0,3)node[right]{$Z$}; 880 | \node at (0,0)[left]{$O$}; 881 | \tkzLabelPoints[below](B,C,A,D) 882 | \tkzLabelPoints[left](B',A') 883 | \tkzLabelPoints[right](C',D') 884 | \end{tikzpicture} 885 | \caption*{第4题} 886 | \end{figure} 887 | 888 | 889 | \item 分别求两点$P_1(3,9,1)$, $P_2(4,1,5)$到平面$S:\; 890 | x-2y+2z-3=0$的距离. 891 | \item 求两条直线 892 | \[\begin{split} 893 | \ell_1:&\quad \frac{x-1}{3}=\frac{y+2}{6}=\frac{z-5}{2}\\ 894 | \ell_2:&\quad \frac{x}{2}=\frac{y-3}{9}=\frac{z+1}{6}\\ 895 | \end{split}\] 896 | 的夹角. 897 | \item 求通过点$(1,-1,2)$并与已知平面:$x+y+z=1$ 898 | 垂直的直线方程.并求这条直线与平面交点的坐标. 899 | \item 在直线$\ell:\; x=1+2t,\; y=8+t,\; z=8+3t$上求 900 | 一点使它和原点的距离等于35. 901 | \item 求满足下列条件的球面方程. 902 | \begin{multicols}{2} 903 | \begin{enumerate} 904 | \item 球心在$(-2,3,-6)$, 半径是7 905 | \item 球心在$(4,0,0)$, 半径是2 906 | \item 球心在$(0,-4,3)$, 半径是5 907 | \item 球心在$(0,-5,0)$, 半径是2 908 | \item 球心在$\left(\frac{2}{3},-\frac{1}{3},0\right)$, 909 | 半径是1 910 | \end{enumerate} 911 | \end{multicols} 912 | 913 | \item 求球面:$x^2+y^2+z^2+4x-6y-2z+5=0$的球心和 914 | 半径. 915 | \end{enumerate} 916 | 917 | \section*{复习题八} 918 | \addcontentsline{toc}{section}{复习题八} 919 | 920 | \begin{enumerate} 921 | \item 已知点$P(3,-1,2)$和$M(a,b,c)$, 求$P$、$M$两点 922 | 分别关于坐标平面、坐标轴以及原点的对称点的坐标. 923 | \item 求点$P(2,5,6)$到坐标原点以及三条坐标轴的距离. 924 | \item 已知$\Vec{OA}=(6,2,9)$,求$\Vec{OA}$与三个坐标平面的夹角. 925 | \item 已知$A(-2,1,3)$, $B(0,-1,2)$,求与$A$、$B$两点 926 | 距离相等点的轨迹方程. 927 | \item 已知$A(a,0,0)$, $B(0,b,0)$, $C(0,0,c)$, 原 928 | 点到平面$(A,B,C)$的距离为$d$, 求证 929 | \[\frac{1}{a^2}+\frac{1}{b^2}+\frac{1}{c^2}=\frac{1}{d^2}\] 930 | \item 在$Z$轴上求一点,使得到$A(-4,1,7)$, $B(3,5,-2)$两点的距离相等. 931 | \item 已知四面体$ABCD$,且$A(x_1,y_1,z_1)$, $B(x_2, 932 | y_2,z_2)$, $C(x_3,y_3, z_3)$, $D(x_4, y_4,z_4)$, 933 | 求它的重心的坐标. 934 | \item 已知$\vec{a}=(a_1,a_2,a_3)$, $\vec{b}=(b_1,b_2,b_3)$, 求证:以$\vec{a}$, $\vec{b}$为邻边的平行四边形面积 935 | \[S=\sqrt{\begin{vmatrix} 936 | a_2&a_3\\b_2&b_3 937 | \end{vmatrix}^2+\begin{vmatrix} 938 | a_3&a_1\\b_3&b_1 939 | \end{vmatrix}^2+\begin{vmatrix} 940 | a_1&a_2\\b_1&b_2 941 | \end{vmatrix}^2}\] 942 | 943 | \item 已知点$P(1,3,5)$和$\vec{a}(-2,1,1)$. 求通过$P$点 944 | 具有方向$\vec{a}$的直线与平面$2x+3y-z=1$的交点. 945 | 946 | \item 求通过如下三点的平面方程. 947 | \begin{enumerate} 948 | \item $(2,1,1),\qquad (3,-1,1),\qquad (4,1,-1)$ 949 | \item $(-2,3,-1),\qquad (2,2,3),\qquad (-4,-1,1)$ 950 | \item $(-5,-1,-2),\qquad (1,2,-1),\qquad (3,-1,2)$ 951 | \end{enumerate} 952 | 953 | \item 求下列通过已知点$P$且以$\vec{a}$为方向向量的直线方程. 954 | \begin{multicols}{2} 955 | \begin{enumerate} 956 | \item $P(2,1,3),\quad \vec{a}=(1,1,-2)$ 957 | \item $P(-5,3,4),\quad \vec{a}=(-2,2,1)$ 958 | \item $P(4,-3,2),\quad \vec{a}=(5,0,3)$ 959 | \item $P(0,0,0),\quad \vec{a}=(2,-3,5)$ 960 | \item $P(a,b,c),\quad \vec{a}=(\ell,m,n)$ 961 | \end{enumerate} 962 | \end{multicols} 963 | 964 | \end{enumerate} 965 | 966 | -------------------------------------------------------------------------------- /7.tex: -------------------------------------------------------------------------------- 1 | 2 | \chapter{极坐标与参数方程} 3 | 4 | \section{极坐标系与曲线的极坐标方程} 5 | \subsection{极坐标的概念} 6 | 7 | 如果知道了一点相对于一定点的距离和方向,那么这个 8 | 点的位置就被唯一确定了.这就是说,我们可用角度和距离 9 | 来确定平面上点的位置.这节,我们研究如何利用角度和距 10 | 离来建立坐标系. 11 | 12 | 在平面内取一个定点$O$, 叫做\textbf{极点},引射线$OA$, 叫做 13 | \textbf{极轴},再选定一个长度单位和角度的正向(通常取逆时针方 14 | 向).对于平面上任一点$P$, 但$P$不是极点,用$r$表示$\overline{OP}$ 15 | 的长度,$\theta$表示从$OA$转到$OP$的角度.这时$r$叫做$P$点的 16 | \textbf{极径},$\theta$叫做$P$点的\textbf{极角}.有序实数对$(r,\theta)$就叫做$P$点 17 | 的极坐标,并记作$P(r,\theta)$. 这样建立的坐标系叫做极坐 18 | 标系(图7.1). 19 | 20 | 在极坐标系中,$r=0$, 21 | 不论$\theta$是什么角,$(0,\theta)$ 22 | 都表示极点,除去极点,显 23 | 然,不同的点对应不同的极 24 | 坐标;反过来任取一对实数 25 | $(r,\theta)$, 其中$0=latex, scale=.8] 40 | \draw[->, very thick](0,0)node[left]{$O$}--(4,0)node[right]{$A$}; 41 | \draw[very thick](0,0)--node[above]{$r$}(40:4)node[right]{$P$}; 42 | \draw[->](.75,0) arc (0:40:.75)node[right]{$\theta$}; 43 | \tkzDrawPoint(40:4) 44 | \end{tikzpicture} 45 | \caption{} 46 | \end{minipage} 47 | \begin{minipage}[t]{0.48\textwidth} 48 | \centering 49 | \begin{tikzpicture}[>=latex, scale=.8] 50 | \draw[->, very thick](0,0)node[below]{$O$}--(4,0)node[right]{$A$}; 51 | \draw[very thick](0,0)--(40:3)node[right]{$M$}; 52 | \draw[very thick](0,0)--(-180+40:2); 53 | \tkzDefPoint(40:1.5){P} 54 | \tkzDefPoint(-180+40:1.5){P'} 55 | \tkzDrawPoints(P,P') 56 | \tkzLabelPoint[right=2pt](P){$P(r,\theta)$} 57 | \tkzLabelPoint[right=2pt](P'){$P'(-r,\theta)$} 58 | \end{tikzpicture} 59 | \caption{} 60 | \end{minipage} 61 | \end{figure} 62 | 63 | 64 | 以极轴$OA$为始边,作有向角$\angle AOM=\theta$, 如果$r>0$, 65 | 在射线$OM$上作$\overline{OP}=r$, 66 | 如果$r<0$, 在射线$OM$的反 67 | 向延长线上作$\overline{OP}=|r|$. 这 68 | 样,对任一对有序实数$(r,\theta)$,我们总可以在平面上 69 | 确定一点$P$; 反过来,对平 70 | 面上任一点$P$, 都可对应无限多有序实数对组成的极坐标, 71 | 如果已知$P(r,\theta)$, 那么$(r,\theta+2k\pi)$, 当$k$为任意整数 72 | 时,都可表示$P$点的极坐标. 73 | 74 | \begin{example} 75 | 在极坐标系中,作出下列各点 76 | \[B\left(4,\frac{\pi}{6}\right),\qquad C(2,0),\qquad D\left(4,\frac{5}{6}\pi\right),\qquad E\left(4,\frac{3}{2}\pi\right)\] 77 | \[F\left(-4,\frac{\pi}{6}\right),\qquad G\left(3,-\frac{\pi}{3}\right),\qquad H\left(1,\frac{\pi}{2}\right)\] 78 | \end{example} 79 | 80 | \begin{solution} 81 | 如图7.3所示. 82 | \begin{figure}[htp] 83 | \centering 84 | \begin{tikzpicture}[>=latex, scale=.7] 85 | \draw[->, thick](0,0)--(5,0)node[right]{$A$}; 86 | \foreach \x in {1,2,3,4} 87 | { 88 | \draw(0,0) circle(\x); 89 | } 90 | \foreach \x in {30,90,...,330} 91 | { 92 | \draw(0,0)--(\x:5); 93 | } 94 | \tkzDefPoint(30:4){B} 95 | \tkzDefPoint(0:2){C} 96 | \tkzDefPoint(150:4){D} 97 | \tkzDefPoint(270:4){E} 98 | \tkzDefPoint(30:-4){F} 99 | \tkzDefPoint(-60:3){G} 100 | \tkzDefPoint(90:1){H} 101 | \tkzDefPoints{0/0/O} 102 | \tkzDrawPoints(B,C,D,E,F,G,H) 103 | \tkzAutoLabelPoints[center=O](G) 104 | \node at (O)[below left]{$O$}; 105 | \tkzLabelPoints[above right](C,H,E) 106 | \tkzLabelPoints[above](D,B) 107 | \tkzLabelPoints[left](F) 108 | \end{tikzpicture} 109 | \caption{} 110 | \end{figure} 111 | \end{solution} 112 | 113 | \begin{ex} 114 | \begin{enumerate} 115 | \item 在极坐标系中,描出下各点. 116 | \[ L\left(3,\frac{\pi}{3}\right),\qquad M(3,0),\qquad N\left(1,\frac{\pi}{2}\right),\qquad P\left(-3,\frac{\pi}{4}\right)\] 117 | \[ Q\left(3, -\frac{\pi}{4}\right), \qquad R\left(-2,-\frac{2}{3}\pi\right),\qquad S\left(1,\frac{3}{4}\pi \right)\] 118 | \item 在极坐标系中,描出下列各点和它们关于原点和极轴的 119 | 对称点. 120 | \[P_1\left(2,\frac{\pi}{3}\right),\qquad P_2\left(-3,\frac{\pi}{2}\right),\qquad P_3\left(\frac{5}{4},\frac{\pi}{4}\right)\] 121 | \item 已知一等边三角形边长为$a$, 它的中心与极点重合,一 122 | 个顶点在极轴上,求三个顶点的极坐标. 123 | \item 已知一正方形边长是$2a$, 它的中心在极点,它的一边与 124 | 极轴垂直.求它的四个顶点的极坐标. 125 | \end{enumerate} 126 | \end{ex} 127 | 128 | \subsection{极坐标和直角坐标的关系} 129 | 130 | \begin{figure}[htp] 131 | \centering 132 | \begin{tikzpicture}[>=latex] 133 | \draw[-> ](-.5,0)--(4,0)node[right]{$X$}; 134 | \draw[-> ](0,-.5)--(0,2.5)node[right]{$Y$}; 135 | \draw(0,0)node[below left]{$O$}--node[left]{$r$}(30:3)node[above]{$P$}--node[right]{$y$}+(0,-1.5); 136 | \node at (1.5,0)[below]{$x$}; 137 | \draw(.6,0) arc(0:30:.6)node[right]{$\theta$}; 138 | \end{tikzpicture} 139 | \caption{} 140 | \end{figure} 141 | 142 | 在平面上建立一直角坐标系$OXY$和一极坐标系,使极 143 | 点和坐标原点$O$重合,极轴$OA$与$X$轴的正半轴重合.设平 144 | 面上任一点$P$在$OXY$中的 145 | 坐标为$(x,y)$, 在极坐 146 | 标系中的坐标为$(r,\theta)$. 147 | 若$P$点的极坐标为已知,且$r>0$, 则由三角学可知, 148 | $P$点的直角坐标可由变换公式 149 | \begin{equation} 150 | x=r\cos\theta,\qquad y=r\sin\theta 151 | \end{equation} 152 | 求得.若$r=0$, 公式(7.1)显然成立,若$r<0$, 则因 153 | $(r,\theta)$和$(-r,\theta+\pi)$表示同一点,故可用$(-r,\theta+\pi)$ 154 | 代替$(r,\theta)$来求$(x,y)$, 于是 155 | \[\begin{split} 156 | x&=-r\cos(\theta +\pi )=-r(-\cos\theta )=r\cos\theta \\ 157 | y&=-r\sin(\theta +\pi )=(-r)(-\sin\theta )=r\sin\theta 158 | \end{split}\] 159 | 因此,当$r<0$时,点的直角坐标仍可由公式(7.1)求得. 160 | 161 | 反过来,如果$P$点的直角坐标为已知,我们可由公式 162 | \begin{equation} 163 | r^2=x^2+y^2,\qquad \tan\theta=\frac{y}{x}\quad (x\ne 0) 164 | \end{equation} 165 | 求得该点的极坐标,由上一小节可知,点$P$的极坐标可对应 166 | 无穷多对数值,其中任一对数值都可作为点$P$的极坐标.在 167 | 一般情况下,我们只求$r\ge 0$, $0\le 0<2\pi$ 的一对数值就可 168 | 以了. 169 | 170 | \begin{example} 171 | 把点$P$的极坐标$\left(3,\frac{\pi}{3}\right)$化为直角坐标. 172 | \end{example} 173 | 174 | \begin{solution} 175 | 由于 176 | \[\begin{split} 177 | x&=3\cdot \cos \frac{\pi}{3}=3\cdot \frac{1}{2}=\frac{3}{2}\\ 178 | y&=3\cdot \sin \frac{\pi}{3}=3\cdot \frac{\sqrt{3}}{2}=\frac{3\sqrt{3}}{2}\\ 179 | \end{split}\] 180 | 因此:点$P$的直角坐标是$\left(\frac{3}{2},\frac{3\sqrt{3}}{2}\right)$ 181 | \end{solution} 182 | 183 | \begin{example} 184 | 把点$M(-1,-1)$化为极坐标. 185 | \end{example} 186 | 187 | \begin{solution} 188 | \[\begin{split} 189 | r&=\sqrt{(-1)^2+(-1)^2}=\sqrt{2}\\ 190 | \tan\theta&=\frac{-1}{-1}=1 191 | \end{split}\] 192 | 由于点$M$在第三象限.因此,取$\theta=\frac{5\pi}{4}$ 193 | 194 | $\therefore\quad $点$M$的极坐标为$\left(\sqrt{2},\frac{5}{4}\pi\right)$ 195 | \end{solution} 196 | 197 | \begin{ex} 198 | \begin{enumerate} 199 | \item 试求下列各点的直角坐标. 200 | \[M\left(-4,-\frac{\pi}{4}\right),\qquad N\left(-4,-\frac{5}{4}\pi\right),\qquad P(-3,8\pi)\] 201 | \[Q(7,0^{\circ}),\qquad R\left(5,-\frac{\pi}{2}\right),\qquad S\left(-2,-\frac{2}{3}\pi\right)\] 202 | \item 试求下列各点的极坐标. 203 | \[B(1,-1),\qquad C\left(3,\sqrt{3}\right),\qquad D(-1,1)\] 204 | \end{enumerate} 205 | \end{ex} 206 | 207 | \subsection{点的轨迹的极坐标方程} 208 | 我们已知,可用一对有序实数$(r,\theta)$来确定平面上 209 | 一点的位置,因此,平面上点的轨迹有时可用含有$r,\theta$两个 210 | 变量的方程来表示,这个方程叫做点的轨迹的\textbf{极坐标方程}或 211 | 简称\textbf{极方程}. 212 | 213 | 214 | \begin{example} 215 | 求通过极点$O$且与极轴成定角$\alpha$的直线的极坐标 216 | 方程(图7.5). 217 | \end{example} 218 | 219 | \begin{solution} 220 | 设点$P(r,\theta)$为已知直线上的任一点,则点$P$ 221 | 满足极方程 222 | \begin{equation} 223 | \theta=\alpha 224 | \end{equation} 225 | 反之,对任一实数$r$, 以$(r,\alpha)$为极坐标的点也一定满 226 | 足方程(7.3). 因此方程(7.3)就是所求的直线的极方程. 227 | \end{solution} 228 | 229 | \begin{figure}[htp]\centering 230 | \begin{minipage}[t]{0.48\textwidth} 231 | \centering 232 | \begin{tikzpicture}[>=latex, scale=.8] 233 | \draw[->, thick](0,0)node[below]{$O$}--(3,0)node[right]{$A$}; 234 | \draw[very thick](0,0)--(45:2.5)node[right]{$P$}; 235 | \draw[very thick](0,0)--(-180+45:2); 236 | \draw[->](.75,0) arc (0:45:.75)node[right]{$\alpha$}; 237 | 238 | \end{tikzpicture} 239 | \caption{} 240 | \end{minipage} 241 | \begin{minipage}[t]{0.48\textwidth} 242 | \centering 243 | \begin{tikzpicture}[>=latex, scale=.8] 244 | \draw[->, thick](0,0)node[below]{$O$}--(3,0)node[right]{$A$}; 245 | \draw[very thick](0,0) circle (1.5); 246 | \draw[->](0,0)--node[right]{$a$}(120:1.5); 247 | \end{tikzpicture} 248 | \caption{} 249 | \end{minipage} 250 | \end{figure} 251 | 252 | \begin{example} 253 | 求圆心在极点$O$, 半径为$a$的圆的极坐标方程 254 | (图7.6). 255 | \end{example} 256 | 257 | \begin{solution} 258 | 因为对任一点$P(r,\theta)$,当且仅当 259 | \begin{equation} 260 | r=a 261 | \end{equation} 262 | 时,$P$点才在已知圆上,所以(7.4)式就是所求圆的极方程. 263 | \end{solution} 264 | 265 | \begin{example} 266 | 试求以$C(a,0)$为圆心,以$a$为半径的圆的 267 | 极坐标方程(图7.7). 268 | \end{example} 269 | 270 | \begin{figure}[htp] 271 | \centering 272 | \begin{tikzpicture}[>=latex, scale=.8] 273 | \draw[->, thick](0,0)--(5,0)node[right]{$A$}; 274 | \draw[very thick](1.5,0) circle (1.5); 275 | \draw (0,0)node[left]{$O$}--(60:1.5)node[above]{$P$}--(3,0)node[below right]{$B$}; 276 | \tkzDefPoint(1.5,0){C} 277 | \tkzDrawPoint(C) 278 | \tkzLabelPoints[below](C) 279 | \end{tikzpicture} 280 | \caption{} 281 | \end{figure} 282 | 283 | \begin{solution} 284 | 由已知条件,圆心在 285 | 极轴上,圆经过极点$O$. 设圆 286 | 和极轴的另一个交点是$B$. 得 287 | 知$P(r,\theta)$在已知圆上的充要条件是$\angle OPB=\pi/2$, 即 288 | \[|\Vec{OP}|=|\Vec{OB}|\cos\theta\] 289 | \begin{equation} 290 | r=2a\cos\theta 291 | \end{equation} 292 | 因此(7.5)式就是所求圆的极方程. 293 | \end{solution} 294 | 295 | 如果某动点的轨迹在直角坐标系中的方程为已知,那 296 | 么,利用变换公式 297 | \[x=r\cos\theta,\qquad y=r\sin\theta\] 298 | 可求得该动点轨迹的极坐标方程;反之,若一动点的轨迹的 299 | 极方程为已知,我们也可用上节变换公式(7.2), 把它化为 300 | 在直角坐标系中的方程. 301 | 302 | \begin{example} 303 | 设一圆的方程为 304 | \[x^2+y^2-8y=0\] 305 | 如果以原直角坐标系的原点为极点,$X$轴的正半轴为极轴, 306 | 求这圆的极方程. 307 | \end{example} 308 | 309 | \begin{solution} 310 | 将$x=r\cos\theta$, $y=r\sin\theta $, 代入已知圆的方程,得 311 | \[r^2-8r\sin\theta =0\] 312 | 即$r=8\sin\theta$. 313 | 这就是已知圆的极坐标方程. 314 | \end{solution} 315 | 316 | \begin{example} 317 | 已知直线的极方程为$r\sin\theta=2$, 把它化为直角 318 | 坐标方程. 319 | \end{example} 320 | 321 | \begin{solution} 322 | 将$r=\sqrt{x^2+y^2}$, $\sin\theta =\frac{y}{r}$代入已知直线的极方 323 | 程,得 324 | \[\sqrt{x^2+y^2}\cdot \frac{y}{\sqrt{x^2+y^2}}=2\] 325 | 即 326 | $y=2$. 327 | 这就是已知直线的直角坐标方程. 328 | \end{solution} 329 | 330 | 我们可根据极方程用描点法近似地作出这个极方程的图 331 | 象,下面举例说明. 332 | 333 | \begin{example} 334 | 描出方程$r=a\theta\; (a>0)$的图象. 335 | \end{example} 336 | 337 | \begin{solution} 338 | 与直角坐标系中的作图步骤一样,先给出$\theta$一系列 339 | 的允许值,算出$r$的对应值,由此得到一对应值表.然后再 340 | 根据对应值表描点作图(图7.8). 341 | \begin{center} 342 | \begin{tabular}{c|cccccccccc} 343 | \hline 344 | $\theta$ & 0&$\frac{\pi}{4}$&$\frac{\pi}{2}$&$\frac{3\pi}{4}$&$\pi$&$\frac{5\pi}{4}$&$\frac{3\pi}{2}$&$\frac{7\pi}{4}$&$2\pi$&$\cdots$\\ 345 | \hline 346 | $r$ & 0& $0.78a$ & $1.57a$ & $2.36a$ & $3.14a$ & $3.93a$ & $4.71a$ & $5.50a$ & $6.28a$ & $\cdots$\\ 347 | \hline 348 | \end{tabular} 349 | \end{center} 350 | \begin{figure}[htp] 351 | \centering 352 | \begin{tikzpicture}[>=latex, scale=.8] 353 | \draw[->](0,0)--(5,0)node[right]{$A$}; 354 | \draw[domain=0:2.25*pi, samples=1000, thick]plot({\x r}: {.5*\x}); 355 | \draw[domain=-2.25*pi:0, samples=1000, thick, dashed]plot({\x r}: {.5*\x}); 356 | \end{tikzpicture} 357 | \caption{} 358 | \end{figure} 359 | 360 | 方程$r=a\theta,\; (a>0)$的图象,叫做\textbf{阿基米德螺线}. 361 | \end{solution} 362 | 363 | \begin{ex} 364 | \begin{enumerate} 365 | \item 求通过极点$O$且与极轴$OA$成$\pi/6$ 366 | 角的直线的极方程. 367 | \item 求以$C(3,\pi/3)$ 368 | 为圆心,半径等于2的圆的极方程. 369 | \item 求通过点$P(r,\theta )$且与原点距离等于$d$的直线的极方 370 | 程. 371 | \item 求$M(r_1,\theta_1)$, $N(r_2,\theta_2)$两点间的距离. 372 | \item 把下列直角坐标方程化为极方程. 373 | \begin{multicols}{2} 374 | \begin{enumerate} 375 | \item $x=6$ 376 | \item $y=2x$ 377 | \item $x^2+y^2-9=0$ 378 | \item $x^2+y^2-4x+8=0$ 379 | \item $xy=4$ 380 | \item $x^2-y^2=1$ 381 | \item $y^2=4x$ 382 | \item $(x^2+y^2)^2=a^2(x^2-y^2)$ 383 | \end{enumerate} 384 | \end{multicols} 385 | 386 | \item 把下列极坐标方程,化为直角坐标方程. 387 | \begin{multicols}{2} 388 | \begin{enumerate} 389 | \item $r=3$ 390 | \item $\theta=\frac{\pi}{4}$ 391 | \item $r=3\cos\theta$ 392 | \item $r=5\tan\theta$ 393 | \item $r^2\cos 2\theta=16$ 394 | \item $r=\frac{6}{1+2\cos\theta}$ 395 | \end{enumerate} 396 | \end{multicols} 397 | \item 画出心脏线$r=a(1+\cos\theta )$的图象. 398 | \item 画出双纽线$p^2=a^2\cos2\theta$的图象. 399 | \end{enumerate} 400 | \end{ex} 401 | 402 | \subsection{圆锥曲线的极坐标方程} 403 | 在第六章中,我们给圆锥曲线下了一个统一的定 404 | 义,现在我们根据这个定义来求圆锥曲线统一的极方程. 405 | 406 | 已知圆锥曲线的焦点$F$和准线$\ell$, 过$F$作$\ell$的垂线,设 407 | 垂足为$D$. 以$F$为极点,$\Vec{DF}$的方向作为极轴的方向建立极 408 | 坐标系(图7.9).设$P(r,\theta)$是曲线上任一点,作$\overline{PF}$, 409 | 再作$PQ\bot\ell$, $PM$垂直极轴,垂足分别为$Q,M$. 设$F$到 410 | 准线$\ell$的距离$\overline{FD}=p$, 则由圆锥曲线的定义,得 411 | \[\frac{PF}{PQ}=e\] 412 | 即:$r=e\cdot PQ$ 413 | 414 | $\because\quad PQ=DF+r\cos\theta=p+r\cos\theta$ 415 | 416 | $\therefore\quad r=e(p+r\cos\theta)$ 417 | 418 | 解出$r$,得: 419 | \begin{equation} 420 | \boxed{ r=\frac{ep}{1-e\cos\theta}} 421 | \end{equation} 422 | 423 | 这就是圆锥曲线的极方程.当$01$ 426 | 时,方程(7.6)表示双曲线,定点$F$是它的右焦点,定直线 427 | $\ell$是它的右准线(图7.10). 428 | 429 | \begin{figure}[htp]\centering 430 | \begin{minipage}[t]{0.48\textwidth} 431 | \centering 432 | \begin{tikzpicture}[>=latex, scale=1] 433 | \draw[->](-1,0)--(4,0)node[right]{$A$}; 434 | \draw[->](-.5,-2)--(-.5,2.5)node[right]{$\ell$}; 435 | \draw[domain=-1.75:1.75, samples=100, thick] plot({\x*\x}, \x); 436 | \draw(1.5,0)node[below]{$M$}--(1.5,1.22)node[above]{$P$}--(-.5,1.22)node[left]{$Q$}; 437 | \draw(.5,0)node[below]{$F$}--node[left]{$r$}(1.5,1.22); 438 | \draw(.75,0) arc (0:50.77:.25)node[right]{$\theta$}; 439 | \node at (-.5,0)[below left]{$D$}; 440 | \end{tikzpicture} 441 | \caption{} 442 | \end{minipage} 443 | \begin{minipage}[t]{0.48\textwidth} 444 | \centering 445 | \begin{tikzpicture}[>=latex, scale=1] 446 | \draw[->](-2,0)--(3,0); 447 | \draw(-1.3,-2.5)node[below]{$\ell$}--(-1.3,2.5); 448 | \draw[domain=0:2*pi, samples=300]plot({\x r}: {.6*1.3/(1-.5*cos(\x r))})node[below right]{$e<1$}; 449 | \draw[domain=0.4*pi:1.6*pi, samples=300, thick]plot({\x r}: {1*1.3/(1-1*cos(\x r))})node[right]{$e=1$}; 450 | \draw[domain=0.45*pi:1.55*pi, samples=300, very thick]plot({\x r}: {1.5*1.3/(1-1.5*cos(\x r))})node[right]{$e>1$}; 451 | \end{tikzpicture} 452 | \caption{} 453 | \end{minipage} 454 | \end{figure} 455 | 456 | \begin{ex} 457 | \begin{enumerate} 458 | \item 求证圆锥曲线$r=\frac{ep}{1-e\cos\theta}$, 459 | 当$01$时,直角坐标方程是$\sqrt{x^2+y^2}=e(x+p)$. 463 | \item 一颗慧星的轨道是抛物线,太阳位于这条抛物线的焦点 464 | 上,已知这颗慧星在距太阳为$1.6\x10^8$公里时,它的极 465 | 径和轨道轴成$60^{\circ}$角.求这颗慧星的轨道的极方程,并 466 | 且求出它的近日点与太阳的距离. 467 | \item 说明下列方程的图形是什么,并且画出草图. 468 | \begin{multicols}{2} 469 | \begin{enumerate} 470 | \item $r=\frac{5}{1-\cos\theta}$ 471 | \item $r=\frac{5}{3-4\cos\theta}$ 472 | \item $r=\frac{1}{2-\cos\theta}$ 473 | \item $r=\frac{4}{1+\cos\theta}$ 474 | \end{enumerate} 475 | \end{multicols} 476 | \end{enumerate} 477 | \end{ex} 478 | 479 | \section*{习题7.1} 480 | \addcontentsline{toc}{subsection}{习题7.1} 481 | 482 | \begin{enumerate} 483 | \item 作出下列极方程的图象,并说明它们各是什么曲线. 484 | \begin{multicols}{2} 485 | \begin{enumerate} 486 | \item $r=1$ 487 | \item $\theta=\frac{\pi}{3}$ 488 | \item $r\cos\theta =2$ 489 | \item $r\sin\theta =1$ 490 | \item $r=6\cos\theta$ 491 | \item $r=10\sin\theta$ 492 | \end{enumerate} 493 | \end{multicols} 494 | 495 | \item 求满足下列条件的各图形的极方程. 496 | \begin{enumerate} 497 | \item 经过点$P\left(2,\frac{\pi}{4}\right)$, 498 | 垂直于极轴的直线; 499 | \item 经过点$Q\left(3,\frac{\pi}{3}\right)$, 500 | 平行于极轴的直线; 501 | \item 圆心在极点,半径等于$a$的圆; 502 | \item 圆心在点$B\left(a,\frac{\pi}{2}\right)$ 503 | 半径等于3的圆; 504 | \item 圆心在$C(a,\pi)$, 半径等于$a$的圆; 505 | \item 经过点$P(a,0)$和极轴相交成$\alpha$角的直线. 506 | \end{enumerate} 507 | 508 | \item 从极点作圆$r=2a\cos\theta$的各弦,求各弦中点的轨迹的极 509 | 坐标方程. 510 | \item 从极点$O$作直线和直线$r\cos\theta=4$相交于$M$点,在$\overline{OM}$ 511 | 上取一点$P$, 使$\overline{OM}\cdot\overline{OP}=12$, 求$P$点轨迹的极坐标 512 | 方程. 513 | \item 求适合于下列条件的轨迹的极坐标方程,并且画出轨迹 514 | 的草图. 515 | \begin{enumerate} 516 | \item 点的极径和极角成正比例; 517 | \item 点的极径和极角成反比例. 518 | \end{enumerate} 519 | \item 把下列各直角坐标方程化为极方程. 520 | \begin{multicols}{2} 521 | \begin{enumerate} 522 | \item $y^2=12x$ 523 | \item $x^2+y^2=4y$ 524 | \item $x^2-2xy+y^2=x-4$ 525 | \item $y^2=4(1-x)$ 526 | \item $y^2=2px$ 527 | \item $\frac{x^2}{a^2}+\frac{y^2}{b^2}=1$ 528 | \item $\frac{x^2}{a^2}-\frac{y^2}{b^2}=1$ 529 | \end{enumerate} 530 | \end{multicols} 531 | \item 判别下列各极方程表示什么曲线. 532 | \begin{multicols}{2} 533 | \begin{enumerate} 534 | \item $r^2=\frac{400}{25\sin^2\theta+16\cos^2\theta}$ 535 | \item $r^2=\frac{4}{\cos^2\theta-\sin^2\theta}$ 536 | \item $r^2=\frac{6}{2\cos^2\theta+3\sin^2\theta}$ 537 | \item $r^2=\frac{1}{\cos^2\theta-2\sin^2\theta}$ 538 | \item $r^2=\frac{4\cos\theta}{\sin^2\theta}$ 539 | \item $r^2=\frac{4\sin\theta}{\cos^2\theta}$ 540 | \end{enumerate} 541 | \end{multicols} 542 | 543 | \item 把下列极坐标方程化成直角坐标方程. 544 | \begin{multicols}{2} 545 | \begin{enumerate} 546 | \item $r=64\sin2\theta$ 547 | \item $r+6\cot\theta\cdot \cos\theta=0$ 548 | \end{enumerate} 549 | \end{multicols} 550 | 551 | \end{enumerate} 552 | 553 | 554 | \section{参数方程} 555 | 556 | \subsection{参数方程的概念} 557 | 在直角坐标系$OXY$中,已知直线$\ell$过点$P_0(x_0,y_0)$ 558 | 且平行于已知向量$\vec{a}=(a_1,a_2)$ (图7.11). 如果$P(x, 559 | y)$是$\ell$上一动点,那么$\ell$的向量方程为 560 | \[\Vec{OP}=\Vec{OP_0}+t\vec{a}\] 561 | 换成坐标形式,即为 562 | \begin{equation} 563 | \begin{cases} 564 | x=x_0+a_1t\\ 565 | y=y_0+a_2t 566 | \end{cases} 567 | \end{equation} 568 | 569 | 570 | \begin{figure}[htp] 571 | \centering 572 | \begin{tikzpicture}[>=latex] 573 | \draw[->](-2,0)--(2.5,0)node[right]{$X$}; 574 | \draw[->](0,-1.5)--(0,2.5)node[right]{$Y$}; 575 | \node at (0,0)[below left]{$O$}; 576 | \draw[domain=-1.5:2, samples=10, very thick]plot(\x, {-\x+1})node[right]{$\ell$}; 577 | \draw[thick,->](0,0)--node[left]{$\vec{a}$}(.9,-.9); 578 | \draw[thick,->](0,0)--(-.4,1.4)node[above]{$P$}; 579 | \draw[thick,->](0,0)--(.25,.75)node[above]{$P_0$}; 580 | \end{tikzpicture} 581 | \caption{} 582 | \end{figure} 583 | 584 | 585 | 这就是说,直线$\ell$上的 586 | 点可以和实数$t$建立一一对 587 | 应关系. 588 | 589 | 一般来说,在取定的坐标系中,如果曲线上任一点的坐 590 | 标$x,y$都是某个变数$t$的函数时, 591 | \begin{equation} 592 | x=f(t),\qquad y=\varphi(t) 593 | \end{equation} 594 | 并且对于$t$的每一允许值,由方程(7.8)所确定的点$P(x,y)$ 595 | 都在这条曲线上.那么方程(7.8)就叫做这条\textbf{曲线的参数方 596 | 程}.例如,方程(7.7)就是通过$P_0(x_0,y_0)$且平行于已知 597 | 向量$\vec{a}$的直线的参数方程. 598 | 599 | 上面我们用参数来表示直角坐标系中点的坐标$x,y$, 600 | 同样我们也可用参数来表示在极坐标系中,点的极坐标$r, 601 | \theta$. 即 602 | \[ r=f(t),\qquad \theta=\varphi(t)\] 603 | 604 | 相对于参数方程来说,直接给出点的坐标间的关系的曲 605 | 线方程,叫做曲线的\textbf{普通方程}.如我们学过的直角坐标方程 606 | 和极方程. 607 | 608 | \subsection{曲线的参数方程} 609 | 除上节建立的直线参数方程外,现在我们来建立几种常 610 | 见的曲线的参数方程. 611 | 612 | \subsubsection{圆的参数方程} 613 | 614 | 615 | 以原点为圆心,$R$为半 616 | 径的圆,可以看作是一个质 617 | 点作等速圆周运动的轨迹 618 | (图7.12).设质点的运 619 | 动的角速度为$\omega$, 从圆周与 620 | $X$轴的正半轴的交点$A$的位 621 | 置开始按逆时针方向运动, 622 | 经过时间$t$后,质点到达圆周上一点$P(x,y)$的位置.由于 623 | $\angle AOP=\omega t$,所以 624 | \begin{equation} 625 | \begin{cases} 626 | x=R\cos\omega t\\ 627 | y=R\sin\omega t 628 | \end{cases} 629 | \end{equation} 630 | 在方程(7.9)中,对应$t$的每一个值,圆周上就有一点 631 | $P(x,y)$与它对应.当$t$的值从0逐渐增加到 632 | $2\pi/\omega$时,$P$ 633 | 点就从$A$点开始按逆时针方向描出一个圆,所以(7.9)式就 634 | 是表示以原点为中心,$R$为半径的圆的参数方程.如果直接 635 | 取$\angle AOP=\theta$作为参数,那么圆的参数方程是 636 | \[ \begin{cases} 637 | x=R\cos\theta\\ 638 | y=R\sin\theta 639 | \end{cases}\] 640 | 641 | \begin{figure}[htp]\centering 642 | \begin{minipage}[t]{0.48\textwidth} 643 | \centering 644 | \begin{tikzpicture}[>=latex, scale=1] 645 | \draw[->](-2,0)--(2.5,0)node[right]{$X$}; 646 | \draw[->](0,-2)--(0,2.5)node[right]{$Y$}; 647 | \node at (0,0)[below left]{$O$}; 648 | \draw[very thick](0,0) circle (1.5); 649 | \draw[thick](0,0)--node[left]{$R$}(45:1.5)node[above]{$P$}--(1.06,0); 650 | \node at (1.5,0)[below right]{$A$}; 651 | \draw[->](.5,0) arc (0:45:.5); 652 | \end{tikzpicture} 653 | \caption{} 654 | \end{minipage} 655 | \begin{minipage}[t]{0.48\textwidth} 656 | \centering 657 | \begin{tikzpicture}[>=latex, scale=1] 658 | \draw[->](-2,0)--(2.5,0)node[right]{$X$}; 659 | \draw[->](0,-2)--(0,2.5)node[right]{$Y$}; 660 | \node at (0,0)[below left]{$O$}; 661 | \draw(0,0) circle (1.5); 662 | \draw(0,0) circle (1); 663 | \draw[very thick] (0,0) ellipse [x radius=1.5, y radius=1]; 664 | \draw(0,.707)--(1.06,.707)node[right]{$P$}; 665 | \draw(0,0)--(45:1.5)node[above right]{$M$}--(1.06,0)node[below]{$Q$}; 666 | 667 | \tkzDefPoints{1.06/0/Q, 1.06/.707/P, 1.06/1.06/M, .707/.707/N} 668 | \tkzDrawPoints(Q,P,M,N) 669 | \node at (45:1)[above]{$N$}; 670 | 671 | \draw(.2,0) arc (0:45:.2); 672 | \draw(.707-.2,.707) arc (180:180+45:.2); 673 | \end{tikzpicture} 674 | \caption{} 675 | \end{minipage} 676 | \end{figure} 677 | 678 | 679 | \subsubsection{椭圆的参数方程} 680 | 设$P(x,y)$是椭圆 681 | $\frac{x^2}{a^2}+\frac{y^2}{b^2}=1$上任一点,以$O$为圆心, 682 | 分别以$a,b$为半径作两个辅助圆(图7.13). 683 | 过$P$点作直线$PQ$垂直于 684 | $X$轴,垂足为$Q$点,并交大 685 | 辅助圆于$M$点,作$\overline{OM}$. 686 | 687 | 设$\angle MOX\varphi$,则$x=\overline{OQ}=a\cos\varphi$. 688 | 把上式代入椭圆方程,得$y=b\sin\varphi$,因此: 689 | \begin{equation} 690 | \begin{cases} 691 | x=a\cos\varphi\\ 692 | y=b\sin\varphi 693 | \end{cases} 694 | \end{equation} 695 | 就是椭圆的一个参数方程,其中$\varphi$叫做\textbf{离心角}. 696 | 697 | \subsubsection{双曲线的参数方程} 698 | 699 | 由三角公式 700 | \[\sec^2\varphi-\tan^2\varphi=1\] 701 | 我们可得双曲线$\frac{x^2}{a^2}-\frac{y^2}{b^2}=1$的一个参数方程为 702 | \begin{equation} 703 | \begin{cases} 704 | x=a\sec\varphi\\ 705 | y=b\tan\varphi 706 | \end{cases} 707 | \end{equation} 708 | 709 | \subsubsection{抛物线$y^2=2px$的参数方程} 710 | 711 | 如果令$y=2pt$,则$x=2pt^2$,所以 712 | \begin{equation} 713 | \begin{cases} 714 | x=2pt^2\\ y=2pt 715 | \end{cases} 716 | \end{equation} 717 | 可作为抛物线$y^2=2px$的一个参数方程. 718 | 719 | \subsubsection{旋轮线的参数方程} 720 | 一个半径是$a$的车轮,沿一条直线轨道滚动,轮周上一 721 | 点$P$的轨迹叫做\textbf{旋轮线}(图7.14). 722 | \begin{figure}[htp] 723 | \centering 724 | \includegraphics[scale=.7]{fig/7-14.png} 725 | \caption{} 726 | \end{figure} 727 | 728 | 下面我们来建立旋轮线的参数方程. 729 | 730 | 取$P$点落在轨道上的个一位置作为原点.轨道所在直线 731 | 作为$X$轴,当车轮从开始起转过了$\varphi$角,设这时$P$点的坐标 732 | 是$(x,y)$, 车轮的圆心在$B$点,与轨道相切于$A$点,于 733 | 是$\wideparen{AP}$的长等于$\overline{OA}$的长,我们引入参数$\varphi$(弧度).(叫 734 | 做滚动角)来表示$x$和$y$. 735 | 736 | 作$PD\bot OX$于$D$点,$PC\bot BA$于$C$点,则 737 | \[\begin{split} 738 | x&=\overline{OD}=\overline{OA}-\overline{DA}=\wideparen{AP}-\overline{PC}=a\varphi-a\sin\varphi=a(\varphi-\sin\varphi)\\ 739 | y&=\overline{DP}=\overline{AC}=\overline{AB}-\overline{BC}=a -a\cos\varphi=a(1-\cos\varphi) 740 | \end{split}\] 741 | 因此,$P$点的轨迹的参数方程是 742 | \begin{equation} 743 | \begin{cases} 744 | x=a(\varphi-\sin\varphi)\\ 745 | y=a(1-\cos\varphi) 746 | \end{cases} 747 | \end{equation} 748 | (7.13)式就是旋轮线的一个参数方程. 749 | 750 | \subsubsection{圆的渐开线参数方程} 751 | 752 | 把一条没有伸缩性的绳子绕在一个固定的圆盘的侧面 753 | 上,拉开绳子的一端并拉直,使绳子和圆周始终相切,然后 754 | 逐渐展开.绳子端点的轨迹叫做圆的\textbf{渐开线}(图7.15). 755 | 这个圆叫做渐开线的\textbf{基圆}. 756 | \begin{figure}[htp] 757 | \centering 758 | \includegraphics[scale=.7]{fig/7-15.png} 759 | \caption{} 760 | \end{figure} 761 | 762 | 下面我们来建立渐开线的参数方程. 763 | 764 | 设定圆的圆心为$O$, 半径为$a$, 开始时绳子的外端在$A$ 765 | 点,$O$为极点,以射线$OA$ 766 | 为极轴建立极坐标系,设$B$ 767 | 是渐开线上任一点,$(r,\theta)$是 768 | 它的极坐标,其中$\theta$的单位 769 | 是弧度.$\overline{OA}=a$, 770 | $\angle BOC=\alpha$, 则$r=\frac{a}{\cos\alpha}$, 771 | $\overline{BC}=a\tan\alpha$. 根据题设应有 772 | \[\overline{BC}=\wideparen{AC}=a(\alpha+\theta)\] 773 | 解出$\theta$, 得 774 | \[\theta=\frac{\overline{BC}}{a}-\alpha=\tan\alpha-\alpha\] 775 | 因此,渐开线的极坐标的参数方程为 776 | \begin{equation} 777 | \begin{cases} 778 | r=\frac{a}{\cos\alpha}\\ 779 | \theta=\tan\alpha-\alpha 780 | \end{cases} 781 | \end{equation} 782 | 783 | 以$OA$为$X$轴的正半轴,建立直角坐标系. 784 | 取$\angle AOC=\varphi$ 作为参数,由于 785 | $\varphi =\alpha+\theta$, 786 | 应用公式(7.14)式的第二式可得 787 | $$\varphi =\tan\alpha$$ 788 | 设$B$点在$OXY$中的坐标为$(x,y)$, 则 789 | \begin{equation} 790 | \begin{split} 791 | x&=r\cos\theta=a(\cos\varphi+\varphi\sin\varphi)\\ 792 | y&=r\sin\theta=a(\sin\varphi-\varphi\cos\varphi) 793 | \end{split} 794 | \end{equation} 795 | 这是渐开线在直角坐标系中的参数方程. 796 | 797 | 由以上几种常见曲线的参数方程的推导可知,通常建立 798 | 曲线的参数方程有两种方法:一种是像(一)那样,把曲线 799 | 看作动点的轨迹,选取时间参数$t$, 使得曲线上的点的动坐 800 | 标$x,y$分别用$t$的函数来表示,另一种是像(二)、(三)那 801 | 样,从已知曲线的直角坐标方程引入适当的参数,从而求得 802 | 曲线的参数方程.最后,我们指出,一条曲线的参数方程不 803 | 是唯一的. 804 | 805 | 以后我们将会看到,利用参数方程研究曲线的形状和性 806 | 质比普通方程更加方便. 807 | 808 | \begin{example} 809 | 画出参数方程$\begin{cases} 810 | x=t^2\\ y=t^3 811 | \end{cases}$ 812 | 所表示的曲线. 813 | \end{example} 814 | 815 | 816 | \begin{solution} 817 | 列表 818 | \begin{center} 819 | \begin{tabular}{ccccccccc} 820 | \hline 821 | $t$&$\cdots$ &$-3$ &$-2$&0&1&2&3&$\cdots$\\ 822 | \hline 823 | $x$&$\cdots$ & 9&4&0&1&4&9 &$\cdots$ \\ 824 | $y$&$\cdots$ & $-27$&$-8$ &0&1&8&27 &$\cdots$ \\ 825 | \hline 826 | \end{tabular} 827 | \end{center} 828 | 829 | 用表中的数对$(x,y)$描点作图,就可 830 | 得到方程的曲线(图7.16).这条曲线叫做\textbf{半立方抛物线}. 831 | 832 | \begin{figure}[htp] 833 | \centering 834 | \begin{tikzpicture}[>=latex, scale=.3] 835 | \draw[->](-2,0)--(8,0)node[right]{$X$}; 836 | \draw[->](0,-8)--(0,8)node[right]{$Y$}; 837 | \draw[domain=0:4, samples=100, very thick]plot(\x, {\x*sqrt(\x)}); 838 | \draw[domain=0:4, samples=100, very thick]plot(\x, {-\x*sqrt(\x)}); 839 | \foreach \x in {2,4,6} 840 | { 841 | \draw(0,\x)node[left]{\x}--(.2,\x); 842 | \draw(\x,0)node[below]{\x}--(\x,.2); 843 | } 844 | \foreach \x in {-2,-4,-6} 845 | { 846 | \draw(0,\x)node[left]{$\x$}--(.2,\x); 847 | } 848 | 849 | 850 | \end{tikzpicture} 851 | \caption{} 852 | \end{figure} 853 | 854 | \end{solution} 855 | 856 | \begin{ex} 857 | \begin{enumerate} 858 | \item 写出下列直线的参数方程. 859 | \begin{enumerate} 860 | \item 过$P(2,3)$且平行于已知向量$\vec{a}=(1,4)$的 861 | 直线 862 | \item 过$P_1(3,4)$, $P_2(4,-3)$两点的直线 863 | \item $y=kx$ 864 | \item $y=kx+b$ 865 | \end{enumerate} 866 | \item 一质点沿方向$\vec{S}=\left(\cos\frac{\pi}{6},\sin\frac{\pi}{6}\right)$, 867 | 从点$P_0(1,2)$, 868 | 以3m/s的速率运动,写出运动轨迹的参数方程. 869 | \item 已知一条直线上两点$M_1(x_1,y_1)$, $M_2(x_2,y_2)$, 以 870 | 分点$M(x,y)$分$\overline{M_1M_2}$所成的比$\lambda$为参数.写出这条直线的参数方程. 871 | \item 已知抛物线$x=2pt^2$, $y=2pt$, 写出通过对应于参数 872 | $t_1,t_2$两点的弦的方程. 873 | \item 作下列参数方程的图形. 874 | \begin{enumerate} 875 | \item $x=t$, $y=3t$ 876 | \item $x=3\sin\theta$, $y=4\cos\theta$ 877 | \item $x=4t^2$, $y=2t$ 878 | \end{enumerate} 879 | \end{enumerate} 880 | \end{ex} 881 | 882 | \subsection{参数方程和普通方程的互化} 883 | 设曲线的参数是 884 | \[\begin{cases} 885 | x=f(t)\\ 886 | y=\varphi(t) 887 | \end{cases}\] 888 | 如果我们能从这个方程消去参数$t$, 那么我们就可求出当线 889 | 的普通方程. 890 | 891 | \begin{example} 892 | 把参数方程$\begin{cases} 893 | x=5\cos t+2\\ 894 | y=2\sin t-3 895 | \end{cases}$化为普通方程. 896 | \end{example} 897 | 898 | \begin{solution} 899 | 由已知参数方程可得 900 | \[\frac{x-2}{5}=\cos t,\qquad \frac{y+3}{2}=\sin t\] 901 | 两式两边平方后相加,得 902 | \[\frac{(x-2)^2}{25}+\frac{(y+3)^2}{4}=1\] 903 | 这就是已知曲线的普通方程. 904 | \end{solution} 905 | 906 | \begin{example} 907 | 把参数方程 908 | \begin{numcases}{} 909 | x=at^2\\ 910 | y=a^2t^3 911 | \end{numcases} 912 | 化为普通方程. 913 | \end{example} 914 | 915 | \begin{solution} 916 | (7.16)式两边立方,(7.17)式两边平方,得 917 | \begin{align} 918 | x^3=a^3t^6\\ 919 | y^2=a^4t^6 920 | \end{align} 921 | 由(7.18), (7.19)两式可得 922 | \[y^2=ax^3\] 923 | \end{solution} 924 | 925 | \begin{example} 926 | 化直线的点斜式方程$y-y_0=k(x-x_0)$为参数 927 | 方程. 928 | \end{example} 929 | 930 | \begin{solution} 931 | 直线的点斜式方程可变为 932 | \[kx-y+y_0-kx_0=0\] 933 | 因此直线具有方向向量为$\vec{S}=(1,k)$, 所以,直线方程的 934 | 参数方程可写为 935 | \[\begin{cases} 936 | x=x_0+t\\ y=y_0+kt 937 | \end{cases}\] 938 | \end{solution} 939 | 940 | 941 | 942 | 943 | \begin{ex} 944 | \begin{enumerate} 945 | \item 把下列曲线的参数方程化为普通方程. 946 | \begin{multicols}{2} 947 | \begin{enumerate} 948 | \item $\begin{cases} 949 | x=3+2t\\y=2-3t 950 | \end{cases}$ 951 | \item $\begin{cases} 952 | x=2+3\cos\theta\\ 953 | y=4-3\sin\theta 954 | \end{cases}$ 955 | \item $\begin{cases} 956 | x=t\\y=4t^2 957 | \end{cases}$ 958 | \item $\begin{cases} 959 | x=\cos^2 t\\ y=\sin t 960 | \end{cases}$ 961 | \end{enumerate} 962 | \end{multicols} 963 | 964 | \item 把下列普通方程化为参数方程. 965 | \begin{multicols}{2} 966 | \begin{enumerate} 967 | \item $\frac{x-2}{4}=\frac{x+5}{3}$ 968 | \item $y=4x$ 969 | \item $\frac{(x-h)^2}{a^2}+\frac{(y-k)^2}{b^2}=1$ 970 | \item $\frac{(x-h)^2}{a^2}-\frac{(y-k)^2}{b^2}=1$ 971 | \item $y=6x^2$ 972 | \item $y^2=8x$ 973 | \end{enumerate} 974 | \end{multicols} 975 | \end{enumerate} 976 | \end{ex} 977 | 978 | \section*{习题7.2} 979 | \addcontentsline{toc}{subsection}{习题7.2} 980 | 981 | \begin{enumerate} 982 | \item 已知直线$\ell$通过点$P_0(x_0,y_0)$并且与已知单位向量 983 | $\vec{e}=(\cos\alpha,\sin\alpha)$平行,求直线$\ell$的参数方程. 984 | \item 求经过点$P(1,3)$, 倾角是$\pi/4$ 985 | 的参数方程. 986 | \item 已知$M(x,y)$从原点以常速度向量$\vec{v}(v_x,v_y)$运动. 987 | 求$M$点的轨迹的参数方程.并且把它化为普通方程.如 988 | 果$M$点从$A(a,b)$点开始运动,$M$点的轨迹的参数方 989 | 程怎样? 990 | \item 把下列参数方程化成普通方程. 991 | \begin{multicols}{2} 992 | \begin{enumerate} 993 | \item $\begin{cases} 994 | x=t^2-2t\\y=t^2+2 995 | \end{cases}$ 996 | \item $\begin{cases} 997 | x=\frac{a(1-t^2)}{1+t^2}\\ 998 | y=\frac{2bt}{1+t^2} 999 | \end{cases}$ 1000 | \item $\begin{cases} 1001 | x=a\sec\varphi\\ 1002 | y=b\tan\varphi 1003 | \end{cases}$ 1004 | \item $\begin{cases} 1005 | x=5t^2-1\\y=10t^2+4 1006 | \end{cases}$ 1007 | \item $\begin{cases} 1008 | x=\frac{a}{2}\left(t+\frac{1}{t}\right)\\ 1009 | y=\frac{b}{2}\left(t-\frac{1}{t}\right) 1010 | \end{cases}$ 1011 | \end{enumerate} 1012 | \end{multicols} 1013 | \item 已知抛物线$x=2pt^2$, $y=2pt$, 求证:通过对应$t_1$, 1014 | $t_2$两点的直线方程是 1015 | \[x-(t_1+t_2)y+2pt_1t_2=0\] 1016 | \item 已知抛物线$x=2pt^2$, $y=2pt$, 求证:抛物线在点$t_1$ 1017 | 处的切线方程为 1018 | \[x-2t_1y+2pt_1^2=0\] 1019 | \item 求证:抛物线$x=2pt^2$, $y=2pt$, 在点$t_1$、$t_2$处的切 1020 | 线交点的坐标是 1021 | \[\big(2pt_1t_2,\; p(t_1+t_2)\big)\] 1022 | \item 利用第7题的结论,证明:抛物线通过焦点的弦的两个端 1023 | 点处的切线相交在准线上. 1024 | 1025 | \end{enumerate} 1026 | 1027 | \section*{复习题七} 1028 | \addcontentsline{toc}{section}{复习题七} 1029 | 1030 | \begin{enumerate} 1031 | \item 画出下列各极坐标方程的图形. 1032 | \begin{multicols}{2} 1033 | \begin{enumerate} 1034 | \item $r\theta =a$ 1035 | \item $r=2\theta$ 1036 | \item $r=5(1-\cos\theta )$ 1037 | \item $r=a\sin3\theta$ 1038 | \item $r=a\cos\theta +b$ 1039 | \item $r^2=16\sin 2\theta$ 1040 | \end{enumerate} 1041 | \end{multicols} 1042 | 1043 | \item 把下列各曲线的极坐标方程化为直角坐标方程. 1044 | \begin{multicols}{2} 1045 | \begin{enumerate} 1046 | \item $r\sin\theta =10$ 1047 | \item $r=4\sin\theta$ 1048 | \item $r(5+3\cos\theta )=16$ 1049 | \item $r(4+5\cos\theta )=9$ 1050 | \item $r^2\cos2\theta =-1$ 1051 | \item $r(\sin\theta +2\cos\theta )=6$ 1052 | \item $r=2\cos\theta +3\sin\theta $ 1053 | \item $\theta =45^{\circ}$ 1054 | \item $r=\frac{3}{2+3\sin\theta}$ 1055 | \item $r^2=9\cos2\theta $ 1056 | \end{enumerate} 1057 | \end{multicols} 1058 | 1059 | \item 把下列各直角坐标方程化为极坐标方程. 1060 | \begin{multicols}{2} 1061 | \begin{enumerate} 1062 | \item $\frac{x^2}{a^2}-\frac{y^2}{b^2}=1$ 1063 | \item $\frac{x^2}{a+x}=\frac{y^2}{a-x}$ 1064 | \item $x^2+y^2=3xy$ 1065 | \item $y^2=\frac{x^3}{2a-x}$ 1066 | \item $x^2+y^2+2Dx+2Ey+F=0$ 1067 | \item $(x^2+y^2)^3=4x^2y^2$ 1068 | \item $x^4+x^2y^2-(x+y)^2=0$ 1069 | \item $(x^2+y^2)^3=16x^2y^2(x^2-y^2)^2$ 1070 | \end{enumerate} 1071 | \end{multicols} 1072 | 1073 | 1074 | \item 说明下列两条直线的位置关系. 1075 | \begin{enumerate} 1076 | \item $\theta =\alpha$ 和$r \cos(\theta-\alpha)=a$ ($a>0$且为定值); 1077 | \item $\theta =\alpha$ 和$r\sin(\theta -\alpha)=a$. 1078 | \end{enumerate} 1079 | 1080 | \item 求证:经$P(r_1,\theta_1)$点和极轴交成$\alpha$角的直线方程是 1081 | \[r\sin(\theta -\alpha )=r_1\sin(\theta_1-\alpha)\] 1082 | \item $O$点是原点,$P$点是椭圆$x=3\cos\varphi$, $y=2\sin\varphi$上相 1083 | 当于$\varphi=\pi/6$的一点,求直线$OP$的倾角. 1084 | \item 已知椭圆 1085 | $x=a\cos\varphi$, $y=b\sin\varphi$, 求证:通过对应 1086 | $\varphi =\alpha$和$\varphi =\beta$ 椭圆上两点的直线方程是 1087 | \[\frac{x}{a}\cos\frac{1}{2}(\alpha+\beta )+\frac{y}{b}\sin\frac{1}{2}(\alpha+\beta )=\cos\frac{1}{2}(\alpha-\beta )\] 1088 | \item 已知椭圆$x=a\cos\varphi$, $y=b\sin\varphi$. 求证:椭圆上对应 1089 | $\varphi_1$点的切线方程是 1090 | \[\frac{x}{a}\cos\varphi_1+\frac{y}{b}\sin\varphi_1=1\] 1091 | \item 一个圆的圆心在$C(a,b)$, 半径为$R$. 求这个圆以圆 1092 | 心角$\theta$(从$X$轴的正方向算起)为参数的参数方程. 1093 | \item 已知$P$、$Q$是椭圆 1094 | $x=a\cos\varphi$, $y=b\sin\varphi$ 上分别对应 1095 | $\varphi_1$和$\varphi_2$的两点.求证:直线$OP$和$OQ$为椭圆共轭直 1096 | 径的条件是$|\varphi_1-\varphi_2|=90^{\circ}$. 1097 | \item 已知椭圆$x=a\cos\varphi$, $y=b\sin\varphi$, $P$、$Q$是椭圆上对 1098 | 应$\varphi$ 和$\varphi +90^{\circ}$的两点,求证: 1099 | \[|\Vec{OP}|^2+|\Vec{OQ}|^2=a^2+b^2\] 1100 | 1101 | \item 画出下列参数方程表示的图形. 1102 | \begin{multicols}{2} 1103 | \begin{enumerate} 1104 | \item $\begin{cases} 1105 | x=3t-5\\y=t^3-t 1106 | \end{cases}$ 1107 | \item $\begin{cases} 1108 | x=t-\sin t\\ y=1-\cos t 1109 | \end{cases}$ 1110 | \end{enumerate} 1111 | \end{multicols} 1112 | \end{enumerate} 1113 | 1114 | 1115 | -------------------------------------------------------------------------------- /4.tex: -------------------------------------------------------------------------------- 1 | \chapter{向量几何初步} 2 | 在第三章,我们学习了向量运算与运算律,这一章我们 3 | 要用向量代数的方法来研究几何学,把对几何学的研究推进 4 | 到有效能算的定量的水平. 5 | 6 | \section{平行与相似} 7 | \subsection{直线的向量方程} 8 | \begin{figure}[htp] 9 | \centering 10 | \begin{tikzpicture}[>=latex, yscale=1.5] 11 | \tkzDefPoints{2/.5/A, .5/2/P, 0/0/O} 12 | \tkzDefMidPoint(A,P) 13 | \tkzGetPoint{B} 14 | \draw[<->, thick](P)--(O)--(A); 15 | \draw[->, thick](O)--(B); 16 | \tkzDrawLines[add =.25 and .25](A,P) 17 | \tkzLabelPoints[right](A,B,P) 18 | \tkzLabelPoint[below](O){$O$} 19 | \end{tikzpicture} 20 | \caption{} 21 | \end{figure} 22 | 23 | 给定空间任意两点$A$、$B$(图4.1), 由平行向量基本定理可知,对空 24 | 间中一点$P$与$A$、$B$两点共线的充要条件是存在一实数$t$, 使 25 | \[\Vec{AP}=t\Vec{AB}\] 26 | 对空间任一点$O$, 这个条件还可 27 | 写为 28 | \[\Vec{OP}-\Vec{OA}=t\left(\Vec{OB}-\Vec{OA}\right)\] 29 | \begin{equation} 30 | \Vec{OP}=(1-t)\Vec{OA}+t\Vec{OB} 31 | \end{equation} 32 | 这就是说,如果$P$点在直线$AB$上,则一定存在实数$t$使(4.1) 33 | 式成立,反之,任给一实数$t$, 由等式(4.1)所确定的$P$点也 34 | 一定在直线$AB$上,方程(4.1)通常叫做\textbf{直线$AB$的向量方 35 | 程}.其中\textbf{参数}$t$的几何意义是,$|t|=|\Vec{AP}|:|\Vec{AB}|$, 当$P$点在 36 | 射线$AB$上,$t\ge 0$. 当$P$在射线$AB$的反向延长线上,$t<0$. 37 | 38 | 由直线$AB$的向量方程(4.1)可知,如果 39 | \[\Vec{OP}=x\Vec{OA}+y\Vec{OB}\] 40 | 那么点$P$在直线$AB$上的充要条件是$x+y=1$ 41 | 42 | \begin{example} 43 | 已知$\vec{a},\vec{b}$是两个线性无关的向量, 44 | $\Vec{OA}=\alpha\vec{a}$, $\Vec{OB}=\beta\vec{b}\quad (\alpha\ne 0,\;\beta\ne 0)$,$\Vec{OC}=x\vec{a}+y\vec{b}$(图4.2), 45 | 则$A$、$B$、$C$三点共线的充要条件是 46 | \[\frac{x}{\alpha}+\frac{y}{\beta}=1\] 47 | \end{example} 48 | 49 | \begin{figure}[htp] 50 | \centering 51 | \begin{tikzpicture}[>=latex] 52 | \draw(0,0)--(-40:4); 53 | \draw[->](0,-3)node[below]{$O$}--(-40:.5)node[right]{$B$}node[below left]{$\beta\vec{b}$}; 54 | \draw[->](0,-3)--(-40:1.5)node[right]{$C$}; 55 | \draw[->](0,-3)--(-40:2.5)node[right]{$A$}node[below]{$\alpha\vec{a}$}; 56 | 57 | \tkzDefPoint(0,-3){O} 58 | \tkzDefPoint(-40:.5){B} 59 | \tkzDefPoint(-40:2.5){A} 60 | \tkzDefMidPoint(O,B) \tkzGetPoint{B1} 61 | \tkzDefMidPoint(O,A) \tkzGetPoint{A1} 62 | \draw[->](O)--(A1)node[right]{$\vec{a}$}; 63 | \draw[->](O)--(B1)node[left]{$\vec{b}$}; 64 | 65 | \end{tikzpicture} 66 | \caption{} 67 | \end{figure} 68 | 69 | \begin{proof} 70 | 由直线的向量方程可知,点$C$在直线$AB$上的充要条 71 | 件是存在实数$t$使 72 | \[\Vec{OC}=(1-t)\Vec{OA}+t\Vec{OB}\] 73 | 即\[\Vec{OC}=(1-t)\alpha\vec{a}+t\beta\vec{b}\] 74 | 但已知$\Vec{OC}=x\vec{a}+y\vec{b}$且$\vec{a}\nparallel\vec{b}$,所以 75 | \[x=(1-t)\alpha,\qquad y=t\beta\] 76 | 消去$t$则可得$A$、$B$、$C$三点共线的充要条件为 77 | \[\frac{x}{\alpha}+\frac{y}{\beta}=1\] 78 | \end{proof} 79 | 80 | \begin{example} 81 | 如图4.3, 设$O$、$A$、$B$三点不共线, 82 | $\Vec{OA}=\vec{a}$, $\Vec{OB}=\vec{b}$, $\Vec{OA_1}=\alpha_1\vec{a}$, $\Vec{OA_2}=\alpha_2\vec{a}$, $\Vec{OB_1}=\beta_1\vec{b}$, $\Vec{OB_2}=\beta_2\vec{b}$ 83 | 且$\alpha_1$、$\alpha_2$、$\beta_1$、$\beta_2$都不为零,又设$A_1B_2$与$A_2B_1$交于$C$点,试以$\vec{a}$、$\vec{b}$、$\alpha_1$、$\alpha_2$、$\beta_1$、$\beta_2$, 表示$\vec{c}=\Vec{OC}$ 84 | \end{example} 85 | 86 | \begin{figure}[htp] 87 | \centering 88 | \begin{tikzpicture}[>=latex] 89 | \tkzDefPoints{0/0/O, 1/0/A, 2/0/A_1, 4.5/0/A_2} 90 | \tkzDefPoint(45:1){B} 91 | \tkzDefPoint(45:2.5){B_1}\tkzDefPoint(45:4.5){B_2} 92 | \tkzDrawSegments[->, thick](O,B_1 O,B_2 O,A_1 O,A_2 A_1,B_2 B_1,A_2) 93 | \tkzInterLL(A_1,B_2)(B_1,A_2) \tkzGetPoint{C} 94 | \tkzDrawSegments[->, thick](O,C) 95 | \tkzLabelPoints[below](A,A_1,A_2) 96 | \tkzLabelPoints[above](B,B_1,B_2) 97 | \tkzLabelPoints[right](C) 98 | \tkzLabelPoints[left](O) 99 | \tkzDrawPoints(A,B) 100 | \end{tikzpicture} 101 | \caption{} 102 | \end{figure} 103 | 104 | 105 | \begin{solution} 106 | 设$\vec{c}=x\vec{a}+y\vec{b}$, 因为$A_1$、$C$、$B_2$三点共线,$A_2$、$C$、$B_1$三点共线,由例4.1有方程组 107 | \[\begin{cases} 108 | \frac{x}{\alpha_1}+\frac{y}{\beta_2}=1\\ 109 | \frac{x}{\alpha_2}+\frac{y}{\beta_1}=1\\ 110 | \end{cases}\] 111 | 由于$A_1B_2$与$A_2B_1$相交于$C$点,可知$\alpha_1\beta_1-\alpha_2\beta_2\ne 0$, 解这个方程组可得 112 | \[x=\alpha_1\alpha_2\frac{\beta_2-\beta_1}{\alpha_2\beta_2-\alpha_1\beta_1},\qquad y=\beta_1\beta_2\frac{\alpha_2-\alpha_1}{\alpha_2\beta_2-\alpha_1\beta_1}\] 113 | \end{solution} 114 | 115 | \begin{ex} 116 | \begin{enumerate} 117 | \item 已知$\vec{a}$、$\vec{b}$线性无关,$\Vec{OA}=\vec{a}$, $\Vec{OB}=\vec{b}$, $\Vec{AP_1}=\frac{1}{3}\Vec{AB}$, 118 | $\Vec{AP_2}=\frac{1}{2}\Vec{AB}$, $\Vec{AP_3}=\frac{3}{2}\Vec{AB}$, 119 | 试用向量$\vec{a}$、$\vec{b}$表示 120 | $\Vec{OP_1},\Vec{OP_2},\Vec{OP_3}$. 121 | 122 | \item 已知$\vec{a}$、$\vec{b}$线性无关,$\Vec{OA}=\vec{a}$, $\Vec{OB}=\vec{b}$, $P$点满足 123 | $\Vec{AP}=\mu \Vec{PB}\; (\mu\in\mathbb{R})$, $P$点叫做$AB$的\textbf{定比分点}.求证: 124 | \[\Vec{OP}=\frac{1}{1+\mu}\vec{a}+\frac{\mu}{1+\mu}\vec{b}\] 125 | \item 已知$\vec{a}$、$\vec{b}$线性无关,$\Vec{OA}=\vec{a}$, $\Vec{OB}=\vec{b}$,$\Vec{AP_1}=\frac{1}{2}\Vec{P_1B}$, 126 | $\Vec{AP_2}=-\frac{1}{2}\Vec{P_2B}$, $\Vec{AP_3}=-\frac{3}{2}\Vec{P_3B}$, 127 | 试用向量$\vec{a}$、$\vec{b}$表 128 | 示$\Vec{OP_1},\Vec{OP_2},\Vec{OP_3}$. 129 | \item 已知$\vec{a}$、$\vec{b}$不平行且$\Vec{OP_1}= 130 | x_1\vec{a}+y_1\vec{b}$, $\Vec{OP_2}=x_2\vec{a}+y_2\vec{b}$, 131 | $P$点在直线$P_1P_2$上且以$k$为比值定比分割$\Vec{P_1P_2}$, 132 | 若$\Vec{OP}=x\vec{a}+y\vec{b}$, 133 | 试用$k$、$x_1$、 134 | $x_2$、$y_1$、$y_2$表示$x$、$y$. 135 | \item 如果$\Vec{OA}=\vec{a}$, $\Vec{OB}=\vec{b}$, $\Vec{OC}=\vec{c}$, 那么,$A$、$B$、 136 | $C$三点共线的充要条件是存在三个不全为零的实数$\alpha$、 137 | $\beta$、$\gamma$使 138 | \[ \alpha\vec{a}+\beta\vec{b}+\gamma\vec{c}=0\quad \text{且}\quad \alpha+\beta+\gamma=0\] 139 | (提示:应用直线的向量方程(4.1)) 140 | \end{enumerate} 141 | \end{ex} 142 | 143 | \subsection{几何证明举例} 144 | 在上一章我们已详细的分析了平 145 | 行、相似与向量的加法、倍积运算之间的密切关系,下面我们 146 | 举例说明向量加法与倍积运算在几何证题中的应用. 147 | 148 | 149 | \begin{example} 150 | 证明三角形中位线定理. 151 | 152 | 已知:在$\triangle ABC$中,$D$、$E$分别是$\overline{AB}$、$\overline{AC}$的中点. 153 | 154 | 求证:$DE\parallel BC$, 且$\overline{DE}=\frac{1}{2}\overline{BC}$ (图4.4) 155 | \end{example} 156 | 157 | \begin{figure}[htp] 158 | \centering 159 | \begin{tikzpicture}[>=latex, scale=.7] 160 | \tkzDefPoints{0/0/B, 4/0/C, 3/4/A} 161 | \tkzDefMidPoint(A,B) \tkzGetPoint{D} 162 | \tkzDefMidPoint(A,C) \tkzGetPoint{E} 163 | \tkzDrawSegments[->, thick](A,B A,C D,E B,C) 164 | \tkzLabelPoints[below](B,C) 165 | \tkzLabelPoints[left](D) 166 | \tkzLabelPoints[right](E) 167 | \tkzLabelPoints[above](A) 168 | \end{tikzpicture} 169 | \caption{} 170 | \end{figure} 171 | 172 | \begin{proof} 173 | 因$D$、$E$分别是$\overline{AB}$、$\overline{AC}$的中点,所以 174 | \[\Vec{AD}=\frac{1}{2} \Vec{AB},\qquad \Vec{AE}=\frac{1}{2}\Vec{AC}\] 175 | \[\Vec{DE}=\Vec{AE}-\Vec{AD}=\frac{1}{2}\left(\Vec{AC}-\Vec{AB}\right)=\frac{1}{2}\Vec{BC}\] 176 | 即:$DE\parallel BC$, $\overline{DE}=\frac{1}{2}\overline{BC}$ 177 | \end{proof} 178 | 179 | 由例4.3的证明,大致可以看出用向量运算证明几何题的 180 | 主要步骤: 181 | \begin{enumerate} 182 | \item 选择基底向量 183 | $\Vec{AB}$、$\Vec{AC}$, 184 | 把已知条件($D$、$E$ 185 | 是$\Vec{AB}$、$\Vec{AC}$的中点)写为向量形式($\Vec{AD}=\frac{1}{2}\Vec{AB}$, $\Vec{AE}=\frac{1}{2}\Vec{AC}$) 186 | \item 进行向量运算,算出结果($\Vec{DE}=\frac{1}{2}\Vec{BC}$) 187 | \item 把结果转化为几何结论. 188 | \end{enumerate} 189 | 190 | 对例4.3的进一步分析,我们还会看到,证明中应用了倍 191 | 积分配律和向量平行的条件,这正好与几何中应用平行四边 192 | 形定理(或相似形定理)相对应. 193 | 194 | \begin{example} 195 | 已知五边形$ABCDE$, $M$、$N$、$P$、$Q$分别是$\overline{AB}$、 196 | $\overline{CD}$、$\overline{BC}$、$\overline{DE}$的中点,$K$、$L$是$\overline{MN}$与$\overline{PQ}$的中点,求 197 | 证:$\overline{KL}=\frac{1}{4}\overline{AE}$ 且$KL\parallel AE$(图4.5). 198 | \end{example} 199 | 200 | \begin{proof} 201 | 在平面上任选一点$O$作为基点,则 202 | \[\begin{split} 203 | \overline{KL}&=\overline{OL}-\overline{OK}=\frac{1}{2}\left(\overline{OP}+\overline{OQ}\right)-\frac{1}{2}\left(\overline{OM}+\overline{ON}\right)\\ 204 | &=\frac{1}{2}\left[\frac{1}{2}\left(\overline{OB}+\overline{OC}\right)+\frac{1}{2}\left(\overline{OD}+\overline{OE}\right)\right]-\frac{1}{2}\left[\frac{1}{2}\left(\overline{OA}+\overline{OB}\right)+\frac{1}{2}\left(\overline{OC}+\overline{OD}\right)\right]\\ 205 | &=\frac{1}{4}\left(\overline{OB}+\overline{OC}+\overline{OD}+\overline{OE}\right)-\frac{1}{4}\left(\overline{OA}+\overline{OB}+\overline{OC}+\overline{OD}\right)\\ 206 | &=\frac{1}{4}\left(\overline{OE}-\overline{OA}\right)=\frac{1}{4}\overline{AE} 207 | \end{split}\] 208 | 即:$KL\parallel AE$且$\overline{KL}=\frac{1}{4}\overline{AE}$ 209 | \end{proof} 210 | 211 | 在例4.4中,基点选为任一点 212 | $O$, 这样对题中各点的位置向量 213 | 表达就比较对称,计算起来就较为方便. 214 | 215 | \begin{figure}[htp]\centering 216 | \begin{minipage}[t]{0.48\textwidth} 217 | \centering 218 | \begin{tikzpicture}[>=latex, scale=1] 219 | \tkzDefPoints{0/0/C, 2/0/D, 1.9/1.7/E, 0/2.5/A, -1/1.2/B, 1.3/3/O} 220 | \tkzDefMidPoint(A,B) \tkzGetPoint{M} 221 | \tkzDefMidPoint(C,B) \tkzGetPoint{P} 222 | \tkzDefMidPoint(C,D) \tkzGetPoint{N} 223 | \tkzDefMidPoint(D,E) \tkzGetPoint{Q} 224 | \tkzDefMidPoint(M,N) \tkzGetPoint{K} 225 | \tkzDefMidPoint(P,Q) \tkzGetPoint{L} 226 | 227 | \tkzDrawPolygon(A,B,C,D,E) 228 | \tkzDrawSegments(M,N P,Q) 229 | \tkzDrawSegments[->](K,L) 230 | \tkzDrawSegments[dashed, ->](O,K O,L) 231 | 232 | \tkzLabelPoints[left](A,B,M,K,P) 233 | \tkzLabelPoints[right](E,Q) 234 | \tkzLabelPoints[below](C,N,D,L) 235 | \tkzLabelPoints[above](O) 236 | 237 | \end{tikzpicture} 238 | \caption{} 239 | \end{minipage} 240 | \begin{minipage}[t]{0.48\textwidth} 241 | \centering 242 | \begin{tikzpicture}[>=latex, scale=1] 243 | \tkzDefPoints{0/0/B, 3/0/C, 2.2/2.2/A} 244 | \tkzDefPointWith[linear, K=.6](A,B) \tkzGetPoint{E} 245 | \tkzDefPointWith[linear, K=.6](C,A) \tkzGetPoint{F} 246 | \tkzDefPointWith[linear, K=.6](C,B) \tkzGetPoint{D} 247 | \tkzDrawSegments[->, thick](A,D A,E A,F) 248 | \tkzDrawSegments(E,B E,D F,D F,C B,C) 249 | \tkzLabelPoints[below](B,D,C) 250 | \tkzLabelPoints[left](E) 251 | \tkzLabelPoints[right](F) 252 | \tkzLabelPoints[above](A) 253 | \end{tikzpicture} 254 | \caption{} 255 | \end{minipage} 256 | \end{figure} 257 | 258 | \begin{example} 259 | 已知$\triangle ABC$, $D$是$\overline{BC}$上任一点,$\overline{DE}\parallel \overline{CA}$, 260 | $\overline{DF}\parallel\overline{BA}$, 261 | 262 | 求证:$\frac{\overline{ED}}{\overline{AC}}+\frac{\overline{FD}}{\overline{AB}}=1$ (图4.6). 263 | \end{example} 264 | 265 | \begin{proof} 266 | 设$\frac{\overline{ED}}{\overline{AC}}=x$, $\frac{\overline{FD}}{\overline{AB}}=y$,则: 267 | \[\Vec{ED}=x\Vec{AC},\qquad \Vec{FD}=y\Vec{AB}\] 268 | 由求和法则可得 269 | \[\Vec{AD}=\Vec{AE}+\Vec{AF}=\Vec{ED}+\Vec{FD}=x\Vec{AC}+y\Vec{AB}\] 270 | 又因$D$在直线$BC$上,所以$x+y=1$,即: 271 | $$\frac{\overline{ED}}{\overline{AC}}+\frac{\overline{FD}}{\overline{AB}}=1$$ 272 | \end{proof} 273 | 274 | 例4.5中的结论,实际上就是一点在直线上的一个必要条 275 | 件,换用向量表达式就一目了然了,应注意,题中若设$D$点 276 | 是直线$BC$上任一点,结论同样成立. 277 | 278 | 279 | \begin{example} 280 | 已知$\parallelogram ABCD$, $M$是$\overline{AB}$的中点,$\overline{DM}$交对角线 281 | $\overline{AC}$于$H$点, 282 | 283 | 求证: 284 | $\overline{AH}=\frac{1}{3} \overline{AC},\qquad \overline{MH}=\frac{1}{3} \overline{MD}$ (图4.7). 285 | \end{example} 286 | 287 | \begin{proof} 288 | 设$\Vec{AH}=x\Vec{AC}$,$\Vec{MH}=y\Vec{MD}$,则 289 | \[\begin{split} 290 | \Vec{AH}&=X\Vec{AC}=x\left(\Vec{AB}+\Vec{AD}\right)=x\Vec{AB}+x\Vec{AD}\\ 291 | \Vec{AH}&=(1-y)\Vec{AM}+y\Vec{AD}=\frac{1}{2}(1-y)\Vec{AB}+y\Vec{AD} 292 | \end{split}\] 293 | 由于$\Vec{AB}$、$\Vec{AD}$线性无关,所以有 294 | \[\begin{cases} 295 | 2x+y=1\\ 296 | y=x 297 | \end{cases}\] 298 | 解方程组可得:$x=\frac{1}{3},\quad y=\frac{1}{3}$ 299 | 300 | 所以: 301 | \[\Vec{AH}=\frac{1}{3}\Vec{AC},\qquad \Vec{MH}=\frac{1}{3}\Vec{MD}\] 302 | \[\overline{AH}=\frac{1}{3}\overline{AC},\qquad \overline{MH}=\frac{1}{3}\overline{MD}\] 303 | \end{proof} 304 | 305 | 例4.6中,证明的关键是设未知数,根据已知条件,用基 306 | 向量$\Vec{AB}$, $\Vec{AD}$写出$\Vec{AH}$的两个表达式,然后由基向量的线 307 | 性无关性转化为方程组来求解.这和在代数中设未知数列方 308 | 程的解问题的方法相似. 309 | 310 | \begin{figure}[htp]\centering 311 | \begin{minipage}[t]{0.48\textwidth} 312 | \centering 313 | \begin{tikzpicture}[>=latex, scale=1] 314 | \tkzDefPoints{0/0/A, 3/0/B, 1/2/D, 4/2/C} 315 | \tkzDefMidPoint(A,B) \tkzGetPoint{M} 316 | \tkzDrawSegments[->](A,D A,M A,B A,C M,D) 317 | \tkzDrawSegments(B,C D,C) 318 | \tkzLabelPoints[below](A,M,B) 319 | \tkzLabelPoints[above](C,D) 320 | \tkzInterLL(A,C)(D,M) \tkzGetPoint{H} 321 | \tkzLabelPoints[above](H)) 322 | \tkzDrawSegments[->](M,H A,H) 323 | \end{tikzpicture} 324 | \caption{} 325 | \end{minipage} 326 | \begin{minipage}[t]{0.48\textwidth} 327 | \centering 328 | \begin{tikzpicture}[>=latex, scale=1] 329 | \tkzDefPoints{0/0/B, 3/0/C, 2/2.6/A} 330 | \tkzDefMidPoint(A,B) \tkzGetPoint{F} 331 | \tkzDefMidPoint(A,C) \tkzGetPoint{E} 332 | \tkzDefMidPoint(C,B) \tkzGetPoint{D} 333 | 334 | \tkzDrawPolygon(A,B,C) 335 | \tkzDrawSegments[->, thick](A,D B,E C,F) 336 | \tkzInterLL(A,D)(B,E) \tkzGetPoint{G} 337 | 338 | \tkzLabelPoints[below](D,C,B) 339 | \tkzLabelPoints[above](A,G) 340 | \tkzLabelPoints[left](F) 341 | \tkzLabelPoints[right](E) 342 | \end{tikzpicture} 343 | \caption{} 344 | \end{minipage} 345 | \end{figure} 346 | 347 | 348 | \begin{example} 349 | 已知$\triangle ABC$, 证明:三条中线$\overline{AD}$、$\overline{BE}$、$\overline{CF}$相交于一点$G$且$\overline{AG}=\frac{2}{3} \overline{AD}$, $\overline{BG}=\frac{2}{3}\overline{BE}$, $\overline{CG}=\frac{2}{3}\overline{CF}$ (图4.8). 350 | \end{example} 351 | 352 | \begin{proof} 353 | 设$\overline{AD},\overline{BE}$相交于 354 | $G$点,$\Vec{AG}=x\Vec{AD}$, $\Vec{BG}=y\Vec{BE}$,$\Vec{AB}=a$, $\Vec{AC}=\vec{b}$,则: 355 | \[\Vec{AG}=x\Vec{AD}=x\x \frac{1}{2}(\vec{a}+\vec{b})=\frac{1}{2}x\vec{a}+\frac{1}{2}x\vec{b}\] 356 | 又 357 | \[\Vec{AG}=(1-y)\Vec{AB}+y\Vec{AE}=(1-y)\vec{a}+\frac{1}{2}y\vec{b}\] 358 | 由$\vec{a},\vec{b}$线性无关,得: 359 | \[\begin{cases} 360 | \frac{1}{2}x=1-y\\ 361 | \frac{1}{2}x=\frac{1}{2}y 362 | \end{cases}\] 363 | 解之得:$x=\frac{2}{3},\quad y=\frac{2}{3}$,所以 364 | \[\Vec{AG}=\frac{2}{3}\Vec{AD},\qquad \Vec{BG}=\frac{2}{3}\Vec{BE}\] 365 | \[\overline{AG}=\frac{2}{3}\overline{AD},\qquad \overline{BG}=\frac{2}{3}\overline{BE}\] 366 | 367 | 如果$\overline{BE},\overline{CF}$相交于$G'$点,那么同样可证, 368 | \[\overline{BG'}=\frac{2}{3} \overline{BE},\qquad \overline{CG'}=\frac{2}{3}\overline{CF}\] 369 | 于是$G$点与$G'$点重合,题中结论得证. 370 | \end{proof} 371 | 372 | \begin{example} 373 | 已知三棱锥$S-ABC$, $K_1$、$L_1$、$M_1$分别是侧棱 374 | $\overline{SA}$、$\overline{SB}$、$\overline{SC}$的中点,$K_2$、$L_2$、$M_2$分别是$\overline{BC}$、$\overline{CA}$、 375 | $\overline{AB}$的中点,求证:$\overline{K_1K_2}$, $\overline{L_1L_2}$, $\overline{M_1M_2}$相交于一点,并在 376 | 这点互相平分(图4.9). 377 | \end{example} 378 | 379 | \begin{proof} 380 | 设$\Vec{SA}=\vec{a}$, $\Vec{SB}=\vec{b}$, $\Vec{SC}=\vec{c}$, $O_1$为$\overline{K_1K_2}$的中点,则 381 | \[\Vec{SO_1}=\frac{1}{2}(\Vec{SK_1}+\Vec{SK_2})=\frac{1}{2}\left[\frac{1}{2}\vec{a}+\frac{1}{2}(\vec{b}+\vec{c})\right] 382 | =\frac{1}{4}\left(\vec{a}+\vec{b}+\vec{c}\right)\] 383 | 取$L_1L_2$的中点$O_2$, $M_1M_2$的中点$O_3$, 同理可证 384 | \[\Vec{SO_2}=\Vec{SO_3}=\frac{1}{4}\left(\vec{a}+\vec{b}+\vec{c}\right)\] 385 | 因此三点$O_1$、$O_2$、$O_3$必重合于一点$O$并在$O$点互相平分. 386 | \end{proof} 387 | 388 | \begin{figure}[htp]\centering 389 | \begin{minipage}[t]{0.48\textwidth} 390 | \centering 391 | \begin{tikzpicture}[>=latex, scale=.8] 392 | \tkzDefPoints{-.5/0/A, 5.5/0/C, 3.5/3/S, 3.5/-1/B} 393 | \tkzDrawPolygon(A,B,C,S) 394 | 395 | \tkzDrawSegments(S,B) 396 | \tkzDefMidPoint(S,A) \tkzGetPoint{K_1} 397 | \tkzDefMidPoint(S,B) \tkzGetPoint{L_1} 398 | \tkzDefMidPoint(S,C) \tkzGetPoint{M_1} 399 | 400 | \tkzDefMidPoint(B,C) \tkzGetPoint{K_2} 401 | \tkzDefMidPoint(C,A) \tkzGetPoint{L_2} 402 | \tkzDefMidPoint(B,A) \tkzGetPoint{M_2} 403 | 404 | \tkzDefMidPoint(M_1,M_2) \tkzGetPoint{O_1} 405 | \tkzDrawSegments[dashed](M_1,M_2 K_1,K_2 A,C L_1,L_2) 406 | 407 | \tkzLabelPoints[below](A,B,C,M_2,K_2,L_2) 408 | \tkzLabelPoints[above](M_1,K_1,L_1,S,O_1) 409 | \end{tikzpicture} 410 | \caption{} 411 | \end{minipage} 412 | \begin{minipage}[t]{0.48\textwidth} 413 | \centering 414 | \begin{tikzpicture}[>=latex, scale=1] 415 | \tkzDefPoints{0/0/O'} 416 | \tkzDefPoint(180-15:3.5){A'}\tkzDefPoint(180-15:2.5){A} 417 | \tkzDefPoint(180:5){C'}\tkzDefPoint(180:1.8){C} 418 | \tkzDefPoint(180+15:4.2){B'}\tkzDefPoint(180+15:2.2){B} 419 | 420 | \tkzDrawPolygon[pattern=north east lines](A,B,C) 421 | \tkzDrawPolygon[pattern=north west lines](A',B',C') 422 | \tkzLabelPoints[left](A',B',C') 423 | \tkzLabelPoints[right](A,B,C) 424 | 425 | \tkzInterLL(A',B')(A,B) \tkzGetPoint{P} 426 | \tkzInterLL(A',C')(A,C) \tkzGetPoint{R} 427 | \tkzInterLL(C',B')(C,B) \tkzGetPoint{Q} 428 | \tkzLabelPoints[right](O') 429 | \tkzLabelPoints[above](P,R) 430 | \tkzLabelPoints[below](Q) 431 | \tkzDrawSegments(Q,B' Q,B Q,P P,A' P,A R,A' R,A) 432 | \tkzDrawSegments[dashed](O',B' O',A' O',C') 433 | 434 | \end{tikzpicture} 435 | \caption{} 436 | \end{minipage} 437 | \end{figure} 438 | 439 | \begin{example} 440 | 试证 Desargues 定理:如图4.10, 设$\triangle ABC$与$\triangle A'B'C'$的顶点连 441 | 线$AA'$、$BB'$、$CC'$相交于一点$O$, 则对应边$AB$与$A'B'$、 442 | $BC$与$B'C'$、$CA$与$C'A'$的延长线分别相交于$P$、$Q$、$R$, 443 | 444 | 试证$P$、$Q$、$R$共线. 445 | \end{example} 446 | 447 | \begin{proof} 448 | 设$\Vec{OA}=\vec{a}$, $\Vec{OB}=\vec{b}$, $\Vec{OC}=\vec{c}$, 则存在$\alpha,\beta,\gamma\in \mathbb{R}$,使$\Vec{OA'}=\alpha\vec{a}$, $\Vec{OB'}=\beta\vec{b}$, $\Vec{OC'}=\gamma\vec{c}$ 449 | 450 | 因$P$点既在$AB$上又在$A'B'$上,所以存在$x,y\in\mathbb{R}$,使 451 | \[\begin{split} 452 | \Vec{OP}&=(1-x)\vec{a}+x\vec{b}\\ 453 | \Vec{OP}&=(1-y)\alpha\vec{a}+\beta y\vec{b}\\ 454 | \end{split}\] 455 | 由此可得 456 | \[x=\frac{\beta(\alpha-1)}{\alpha-\beta},\qquad y=\frac{\alpha-1}{\alpha-\beta}\] 457 | \begin{equation} 458 | \Vec{OP}=\frac{\alpha(1-\beta)}{\alpha-\beta}\vec{a}+\frac{\beta(\alpha-1)}{\alpha-\beta}\vec{b} 459 | \end{equation} 460 | 同理可求得 461 | \begin{align} 462 | \Vec{OQ}&=\frac{\beta(1-\gamma)}{\beta-\gamma}\vec{b}+\frac{\gamma(\beta-1)}{\beta-\gamma}\vec{c}\\ 463 | \Vec{OR}&=\frac{\gamma(1-\alpha)}{\gamma-\alpha}\vec{c}+\frac{\alpha(\gamma-1)}{\gamma-\alpha}\vec{a} 464 | \end{align} 465 | \[\begin{split} 466 | \Vec{PR}&=\Vec{OR}-\Vec{OP}=\frac{\alpha(\alpha-1)(\gamma-\beta)}{(\gamma-\alpha)(\alpha-\beta)}\vec{a}+\frac{\beta(1-\alpha)}{\alpha-\beta}\vec{b}+\frac{\gamma(1-\alpha)}{\gamma-\alpha}\vec{c}\\ 467 | \Vec{PQ}&=\Vec{OQ}-\Vec{OP}=\frac{\alpha(\beta-1)}{\alpha-\beta}\vec{a}+\frac{\beta(\beta-1)(\gamma-\alpha)}{(\alpha-\beta)(\beta-\gamma)}\vec{b}+\frac{\gamma(\beta-1)}{\beta-\gamma}\vec{c} 468 | \end{split}\] 469 | 470 | 容易验证$\vec{a},\vec{b},\vec{c}$的系数成比例,所以 471 | $\Vec{PR}$与$\Vec{PQ}$共 472 | 线,即$P$、$Q$、$R$共线. 473 | 474 | 此题还可另证,设$x=(y-1)(\alpha-\beta)$, 475 | $y=(\alpha-1)(\beta-\gamma)$, $z=(\beta-1)(\gamma-\alpha)$, 则可得: 476 | \[x\Vec{OP}+y\Vec{OQ}+z\Vec{OR}=\vec{0}\] 477 | 并且$x+y+z=0$. 478 | 479 | $x$、$y$、$z$至少有一不为0, 480 | 这便证明了$P$、$Q$、$R$三点共线. 481 | \end{proof} 482 | 483 | \begin{ex} 484 | 试用向量运算证明以下各题. 485 | \begin{enumerate} 486 | \item 试证平行四边形的对角线互相平分. 487 | \item 设$\triangle ABC$和$\triangle A'B'C'$的对应顶点连线$AA'$、$BB'$、 488 | $CC'$相交于一点$O$. 试证若 $AB\parallel A'B'$, $BC\parallel B'C'$, 则 489 | $AC\parallel A'C'$. 490 | 491 | \item 设$\ell$、$\ell'$相交于$O$点,$A,B,C\in\ell$, $A',B',C'\in\ell'$. 试 492 | 证如果$AB'\parallel A'B$, $BC'\parallel B'C$, 则有$AC'\parallel A'C$. 493 | \item 证明梯形中位线定理. 494 | \item 已知:$\triangle ABC$中,$D$是$\overline{BC}$的中点,过$D$任作一直线分别交$\overline{AC}$于$E$, 交$AB$的延长线于$F$, 求证: 495 | $\overline{AE}:\overline{EC}=\overline{AF}:\overline{FB}$. 496 | \item 已知:梯形$ABCD$, $AB\parallel DC$, $\overline{AB}=2\overline{CD}$, $\overline{AC}$、$\overline{BD}$ 497 | 相交于$E$, 求证:$\overline{CE}=\frac{1}{3}\overline{AC}$. 498 | \item 已知:梯形$ABCD$中,$E$、$F$是上、下底$\overline{AD}$、$\overline{BC}$的 499 | 中点,$\overline{AC}$、$\overline{BD}$相交于$G$, 求证:$E$、$G$、$F$三点共线. 500 | \end{enumerate} 501 | \end{ex} 502 | 503 | \begin{figure}[htp]\centering 504 | \begin{minipage}[t]{0.48\textwidth} 505 | \centering 506 | \begin{tikzpicture}[>=latex, scale=1] 507 | \tkzDefPoints{0/0/O, 4/0/A', 4.2/1.5/B', 3/2.5/C'} 508 | \tkzDefPointWith[linear, K=.6](O,A') \tkzGetPoint{A} 509 | \tkzDefPointWith[linear, K=.6](O,B') \tkzGetPoint{B} 510 | \tkzDefPointWith[linear, K=.6](O,C') \tkzGetPoint{C} 511 | \tkzDrawSegments(O,C' O,A' O,B') 512 | \tkzDrawPolygon(A,B,C) 513 | \tkzDrawPolygon(A',B',C') 514 | \tkzLabelPoints[below](A,A')\tkzLabelPoints[above](C,C') 515 | \tkzLabelPoints[below right](B,B')\tkzLabelPoints[left](O) 516 | \end{tikzpicture} 517 | \caption*{第2题} 518 | \end{minipage} 519 | \begin{minipage}[t]{0.48\textwidth} 520 | \centering 521 | \begin{tikzpicture}[>=latex, scale=.7] 522 | \tkzDefPoints{0/0/O, 2/0/A, 3.5/0/B, 5/0/C} 523 | \tkzDefPoint(40:2){C'}\tkzDefPoint(40:3.1){B'} 524 | \tkzDefPoint(40:5){A'} 525 | \tkzLabelPoints[above](A',B',C') 526 | \tkzLabelPoints[below](A,B,C) 527 | \draw(0,0)node[left]{$O$}--(6,0)node[right]{$\ell$}; 528 | \draw(0,0)--(40:6)node[right]{$\ell'$}; 529 | \tkzDrawSegments(A,B' A,C' B,C' B,A' C,B' C,A') 530 | \end{tikzpicture} 531 | \caption*{第3题} 532 | \end{minipage} 533 | \end{figure} 534 | 535 | \section*{习题4.1} 536 | \addcontentsline{toc}{subsection}{习题4.1} 537 | 538 | \begin{enumerate} 539 | \item 已知:$O$是一个定点,$\vec{a}$、$\vec{b}$是两个线性无关的向量, 540 | $\Vec{OP_1}=2\vec{a}+\vec{b}$, $\Vec{OP_2}=\vec{a}-\vec{b}$, $P$是直线$P_1P_2$上任意一 541 | 点且$\Vec{OP}=x\vec{a}+y\vec{b}$, 求$x$、$y$满足的代数关系式. 542 | \item 已知:在$\triangle ABC$中,$D$、$E$分别是$\overline{BC}$、$\overline{AC}$边上的点且 543 | $\overline{BD}:\overline{DC}=1:2$, $\overline{CE}:\overline{EA}=1:1$, $\overline{AD}$与$\overline{BE}$相交于$O$ 544 | 点,设$\Vec{AB}=\vec{a}$, $\Vec{AC}=6$, $\Vec{CO}=x\vec{a}+y\vec{b}$, 求$x$、$y$. 545 | \item 设$\vec{a}$、$\vec{b}$是两个线性无关的向量,$\Vec{OD}=h\vec{a}+k\vec{b}\ne \vec{0}$. 546 | $P$是直线$OD$上任意一点,令$\Vec{OP}=x\vec{a}+y\vec{b}$, 求证: 547 | $x$、$y$满足方程$kx-hy=0$. 548 | \item 已知$D$、$E$、$F$分别是$\triangle ABC$的边$\overline{AB}$、$\overline{BC}$、$\overline{AC}$上的一 549 | 点且$\frac{\overline{AD}}{\overline{AB}}=\frac{\overline{BE}}{\overline{BC}}=\frac{\overline{CF}}{\overline{CA}}$ 550 | 551 | 求证: 552 | 所有这样的 553 | $\triangle DEF$重心是一个定点. 554 | 555 | \item 如图,已知平行六面体$OADB-CEFG$, 求证:若对角 556 | 线$\overline{AG}$与平面$(O,D,E)$相交于$H$, $OH$与侧面$ADFE$ 557 | 相交于$K$, 求证$\overline{AH}=\frac{1}{3}\overline{AG}$,$\overline{OH}=\frac{2}{3}\overline{OK}$ 558 | 559 | \begin{figure}[htp]\centering 560 | \begin{minipage}[t]{0.48\textwidth} 561 | \centering 562 | \begin{tikzpicture}[>=latex, scale=1] 563 | \tkzDefPoints{0/0/O, 3/0/A, 3.5/1/D, .5/1/B, .4/3/C} 564 | \tkzDefPointsBy[translation = from O to C](A,B,D){E,G,F} 565 | \tkzDrawPolygon(C,G,F,E) 566 | \tkzDrawSegments(A,E D,F E,D F,A E,O A,D) 567 | \tkzDrawSegments[dashed](A,G B,G B,D O,D) 568 | \tkzDrawSegments[->, thick](O,A O,C) 569 | \tkzDrawSegments[->, dashed, thick](O,B) 570 | \tkzInterLL(E,D)(F,A) \tkzGetPoint{K} 571 | \tkzDrawSegments[dashed](O,K) 572 | 573 | \node at (1.5,0)[below]{$\vec{a}$}; 574 | \node at (.25,0.5)[right]{$\vec{b}$}; 575 | \node at (.2,1.5)[left]{$\vec{c}$}; 576 | 577 | \tkzLabelPoints[below](O,A) 578 | \tkzLabelPoints[above right](B,D) 579 | \tkzLabelPoints[right](K) 580 | \tkzLabelPoints[above left](C,E) 581 | \tkzInterLL(A,G)(O,K) \tkzGetPoint{H} 582 | \tkzLabelPoints[above ](G,F,H) 583 | \end{tikzpicture} 584 | \caption*{第5题} 585 | \end{minipage} 586 | \begin{minipage}[t]{0.48\textwidth} 587 | \centering 588 | \begin{tikzpicture}[>=latex, scale=1] 589 | \tkzDefPoints{0/0/B, 3/0/C, 2/2.6/A} 590 | \tkzDefMidPoint(A,B) \tkzGetPoint{R} 591 | \tkzDefMidPoint(A,C) \tkzGetPoint{Q} 592 | \tkzDefMidPoint(C,B) \tkzGetPoint{P} 593 | 594 | \tkzDrawPolygon(A,B,C) 595 | \tkzDrawSegments(A,P B,Q C,R) 596 | \tkzLabelPoints[below](P,C,B) 597 | \tkzLabelPoints[above](A) 598 | \tkzLabelPoints[left](R) 599 | \tkzLabelPoints[right](Q) 600 | \end{tikzpicture} 601 | \caption*{第6题} 602 | \end{minipage} 603 | \end{figure} 604 | 605 | \item 在$\triangle ABC$的三条边上各取一点$P$、$Q$、$R$, 若$\Vec{AR}=x\Vec{RB}$, $\Vec{BP}=y\Vec{PC}$, $\Vec{CQ}=z\Vec{QA}$. 求证:$AP$、$BQ$、$CR$共 606 | 点的充要条件是 607 | $xyz=1$. 608 | 609 | \item 在$\triangle ABC$三边所在的直线 610 | 上各取一点$P$、$Q$、$R$, 使 611 | \[\Vec{AR}=x\Vec{RB},\qquad \Vec{BP}=y\Vec{PC},\qquad \Vec{CQ}=z\Vec{QA}\] 612 | 求证:$P$、$Q$、$R$三点共线的充要条件是$xyz=-1$. 613 | \begin{figure}[htp]\centering 614 | \begin{tikzpicture}[>=latex, scale=1] 615 | \tkzDefPoints{0/0/B, 3/0/C, 2/2.6/A} 616 | \tkzDefPointWith[linear, K=.7](A,B) \tkzGetPoint{R} 617 | \tkzDefPointWith[linear, K=.4](A,C) \tkzGetPoint{Q} 618 | \tkzInterLL(B,C)(R,Q)\tkzGetPoint{P} 619 | \tkzDrawPolygon(A,B,C) 620 | \tkzDrawSegments(Q,P B,P C,R) 621 | \tkzLabelPoints[below](P,C,B) 622 | \tkzLabelPoints[above](A) 623 | \tkzLabelPoints[above left](R) 624 | \tkzLabelPoints[right](Q) 625 | \end{tikzpicture} 626 | \caption*{第7题} 627 | \end{figure} 628 | \end{enumerate} 629 | 630 | \section{垂直与度量问题} 631 | 这节我们举例说明,用内积运算处理度量问题的一般方 632 | 法和技巧. 633 | 634 | \begin{example} 635 | 证明勾股定理(图4.11). 636 | \end{example} 637 | 638 | \begin{proof} 639 | 在直角$\triangle ABC$中,$\angle B=90^{\circ}$, 则 640 | \[\begin{split} 641 | \Vec{AC}&=\Vec{AB}+\Vec{BC}\\ 642 | \Vec{AC}\cdot \Vec{AC}&= \left(\Vec{AB}+\Vec{BC}\right)\cdot \left(\Vec{AB}+\Vec{BC}\right)\\ 643 | &=\Vec{AB}\cdot \Vec{AB}+\Vec{BC}\cdot \Vec{BC} 644 | \end{split}\] 645 | 即:$\overline{AC}^2=\overline{AB}^2+\overline{BC}^2$ 646 | \end{proof} 647 | 648 | \begin{figure}[htp]\centering 649 | \begin{minipage}[t]{0.48\textwidth} 650 | \centering 651 | \begin{tikzpicture}[>=latex, scale=.8] 652 | \tkzDefPoints{0/0/A, 3/0/B, 3/4/C} 653 | \tkzDrawSegments[->](A,B B,C A,C) 654 | \tkzLabelPoints[below](A,B) 655 | \tkzLabelPoints[above](C) 656 | \end{tikzpicture} 657 | \caption{} 658 | \end{minipage} 659 | \begin{minipage}[t]{0.48\textwidth} 660 | \centering 661 | \begin{tikzpicture}[>=latex, scale=1] 662 | \tkzDefPoints{0/0/A, 3/0/B, 2/3/C} 663 | \tkzDrawSegments[->](A,B B,C A,C) 664 | \tkzLabelPoints[below](A,B) 665 | \tkzLabelPoints[above](C) 666 | \node at (1.5,0)[below]{$c$}; 667 | \node at (1,1.5)[left]{$b$}; 668 | \node at (2.5,1.5)[right]{$a$}; 669 | \end{tikzpicture} 670 | \caption{} 671 | \end{minipage} 672 | \end{figure} 673 | 674 | 675 | \begin{example} 676 | 证明余弦定理(图4.12). 677 | \end{example} 678 | 679 | \begin{proof} 680 | 在$\triangle ABC$中, 681 | \[\begin{split} 682 | \Vec{AC}&=\Vec{AB}+\Vec{BC}\\ 683 | \Vec{AC}\cdot \Vec{AC}&=\left(\Vec{AB}+\Vec{BC}\right)\cdot \left(\Vec{AB}+\Vec{BC}\right)\\ 684 | &=\Vec{AB}\cdot \Vec{AB}+\Vec{BC}\cdot \Vec{BC}+2\Vec{AB}\cdot \Vec{BC} 685 | \end{split}\] 686 | 即 687 | \[\begin{split} 688 | |\Vec{AC}|^2&=|\Vec{AB}|^2+|\Vec{BC}|^2-2|\Vec{AB}||\Vec{BC}|\cos B\\ 689 | b^2&=c^2+a^2-2ca\cos B 690 | \end{split}\] 691 | 同理可证: 692 | \[\begin{split} 693 | c^2&=a^2+b^2-2ab\cos C\\ 694 | a^2&=b^2+c^2-2bc\cos A 695 | \end{split}\] 696 | \end{proof} 697 | 698 | 由例4.10、例4.11的证明过程可以看到,为了得到三角形的边 699 | 角关系,只要写出三角形中边向量所要满足的关系,然后作 700 | 内积运算,向量关系就可转化为数量关系. 701 | 702 | 703 | 704 | \begin{example} 705 | 证明射影定理与正弦定理. 706 | \end{example} 707 | 708 | \begin{proof} 709 | 过$A$点引单位向量$\vec{e}_1\parallel \Vec{AC}$, $\vec{e}_2 710 | \bot \vec{e}_1$ (图4.13) 711 | \begin{equation} 712 | \begin{split} 713 | \Vec{AC}&=\Vec{AB}+\Vec{BC}\\ 714 | \Vec{AC}\cdot \vec{e}_1&=\Vec{AB}\cdot \vec{e}_1+\Vec{BC}\cdot \vec{e}_1\\ 715 | |\Vec{AC}|&=|\Vec{AB}|\cos A+|\Vec{BC}|\cos C 716 | \end{split} 717 | \end{equation} 718 | 即:$b=c\cos A+a\cos C$ 719 | 同理可证: 720 | \[c=a\cos B+b\cos A,\qquad a=b\cos C+c\cos B\] 721 | (4.5)式两边分别对$\vec{e}_2$取内积运算,则 722 | \[\begin{split} 723 | \Vec{AC}\cdot \vec{e}_2&=\Vec{AB}\cdot \vec{e}_2+\Vec{BC}\cdot \vec{e}_2\\ 724 | 0&=|\Vec{AB}|\cos(90^{\circ}-A)+| \Vec{BC} |\cos(90^{\circ}+C) 725 | \end{split}\] 726 | 即:$0=c\sin A-a\sin C\quad \Rightarrow\quad \frac{a}{\sin A}=\frac{c}{\sin C}$ 727 | 728 | 同理可证:$\frac{a}{\sin A}=\frac{b}{\sin B}$, 729 | 所以 730 | \[\frac{a}{\sin A}=\frac{b}{\sin B}=\frac{c}{\sin C}\] 731 | \end{proof} 732 | 733 | 在例4.12的证明中,我们先写出三角形三个边向量所满足 734 | 的向量关系式,然后,分别对$\vec{e}_1$、$\vec{e}_2$两个互相垂直的单位向量取内积运算,这样就很容易地证明了射影定理和正弦定 735 | 理.适当选取单位向量,对题设条件所满足的向量关系式进 736 | 行内积运算是处理一些直线形边角关系的 737 | 基本方法之一. 738 | 739 | \begin{figure}[htp]\centering 740 | \begin{minipage}[t]{0.48\textwidth} 741 | \centering 742 | \begin{tikzpicture}[>=latex, scale=1] 743 | \tkzDefPoints{0/0/A, 3/0/C, 2/2/B} 744 | \tkzDrawSegments[->](A,B B,C A,C) 745 | \tkzLabelPoints[below](A,C) 746 | \tkzLabelPoints[above](B) 747 | \node at (1.5,0)[below]{$b$}; 748 | \node at (1,1)[left]{$c$}; 749 | \node at (2.5,1)[right]{$a$}; 750 | \draw[thick,->](0,0)--node[below]{$\vec{e}_1$}(1,0); 751 | \draw[thick,->](0,0)--node[left]{$\vec{e}_2$}(0,1); 752 | 753 | \end{tikzpicture} 754 | \caption{} 755 | \end{minipage} 756 | \begin{minipage}[t]{0.48\textwidth} 757 | \centering 758 | \begin{tikzpicture}[>=latex, scale=1] 759 | \tkzDefPoints{0/0/B, 4/0/C, 3.5/2/A, 2.7/0/P} 760 | \node at (1.75,1)[left]{$c$}; 761 | \node at (3.75,1)[right]{$b$}; 762 | \node at (2,0)[below]{$a$}; 763 | \tkzDrawSegments[->](A,B A,C A,P) 764 | \tkzDrawSegments(B,C) 765 | \tkzLabelPoints[below](B,P,C) 766 | \tkzLabelPoints[above](A) 767 | \draw[->](P)--node[above]{$\vec{a}$}+(160:1); 768 | 769 | \end{tikzpicture} 770 | \caption{} 771 | \end{minipage} 772 | \end{figure} 773 | 774 | \begin{example} 775 | 利用内积运算证明角平分线定理. 776 | \end{example} 777 | 778 | 779 | \begin{proof} 780 | 设$\overline{AP}$为$\triangle ABC$中内角$A$的平 781 | 分线,则$\angle BAP=\angle PAC=\alpha$, 取单位向量 782 | $\vec{e}\bot \Vec{AP}$(图4.14) 783 | \[\begin{split} 784 | \Vec{AB}\cdot \vec{e}&=|\Vec{AB} |\cos(90^{\circ}-\alpha)=|\Vec{AB} |\sin\alpha\\ 785 | \Vec{AC}\cdot \vec{e}&=|\Vec{AC}|\cos(90^{\circ}+\alpha)=-|\Vec{AC}|\sin\alpha 786 | \end{split}\] 787 | \[\frac{\Vec{AB}}{\Vec{AC}}=\frac{\Vec{AB}\cdot \vec{e}}{\Vec{AC}\cdot \vec{e}}=\frac{|(\Vec{AP}+\Vec{PB})\cdot \vec{e}|}{|(\Vec{AP}+\Vec{PC})\cdot \vec{e}|}=\frac{|\Vec{PB}\cdot \vec{e}|}{\Vec{PC}\cdot \vec{e}}=\frac{\Vec{PB}}{\Vec{PC}}\] 788 | \end{proof} 789 | 790 | 791 | \begin{example} 792 | 在直角$\triangle ABC$中,$AD$是斜边$\overline{BC}$上的高,作 793 | $DE\bot AB$, $DF\bot AC$, $E$、$F$是垂足. 794 | 795 | 求证:$\frac{\overline{BE}}{\overline{CF}}=\frac{\overline{AB}^3}{\overline{AC}^3}$ 796 | \end{example} 797 | 798 | \begin{proof} 799 | 设$\frac{\overline{EB}}{\overline{AB}}=\lambda$, $\frac{\overline{FC}}{\overline{AC}}=\mu$,则: 800 | \[\Vec{EB}=\lambda\Vec{AB},\quad \Vec{FC}=\mu\Vec{AC},\quad \Vec{AE}=(1-\lambda)\Vec{AB},\quad \Vec{AF}=(1-\mu)\Vec{AC}\] 801 | \[\Vec{AD}=\Vec{AE}+\Vec{AF}=(1-\lambda)\Vec{AB}+(1-\mu)\Vec{AC}\] 802 | 因为$D\in BC$,则: 803 | \[(1-\lambda)+(1-\mu)=1\quad \Rightarrow\quad \lambda+\mu=1\] 804 | 又$\because\quad \Vec{AD}\bot \Vec{BC}$,因此: 805 | \[\left[(1-\lambda)\Vec{AB}+(1-\mu)\Vec{AC}\right]\cdot \left(\Vec{AC}-\Vec{AB}\right)=0\] 806 | 由此可得: 807 | \[\frac{\lambda}{\mu}=\frac{\overline{AB}^2}{\overline{AC}^2}\] 808 | \[\frac{\overline{EB}}{\overline{FC}}=\frac{\lambda\overline{AB}}{\mu\overline{AC}}=\frac{\overline{AB}^3}{\overline{AC}^3}\] 809 | \end{proof} 810 | 811 | \begin{figure}[htp]\centering 812 | \begin{minipage}[t]{0.48\textwidth} 813 | \centering 814 | \begin{tikzpicture}[>=latex, scale=1] 815 | \tkzDefPoints{0/0/B, 4/0/C, 2.5/2/A, 2.5/0/D} 816 | \tkzDefPointBy[projection =onto A--B](D) \tkzGetPoint{E} 817 | \tkzDefPointBy[projection =onto A--C](D) \tkzGetPoint{F} 818 | \tkzDrawPolygon(A,C,B) 819 | \tkzDrawSegments(D,E D,F) 820 | \tkzDrawSegments[->](A,E A,F A,D) 821 | \tkzLabelPoints[below](B,C,D) 822 | \tkzLabelPoints[left](E) 823 | \tkzLabelPoints[right](F) 824 | \tkzLabelPoints[above](A) 825 | \end{tikzpicture} 826 | \caption{} 827 | \end{minipage} 828 | \begin{minipage}[t]{0.48\textwidth} 829 | \centering 830 | \begin{tikzpicture}[>=latex, scale=1] 831 | \tkzDefPoints{0/0/A, 5/0/B, 2.5/0/O} 832 | \tkzDefPoint(36.87:4){P} 833 | \tkzDrawPolygon(A,B,P) \tkzDrawSegments(P,O) 834 | \tkzLabelPoints[below](B,A,O) 835 | \tkzLabelPoints[above](P) 836 | \end{tikzpicture} 837 | \caption{} 838 | \end{minipage} 839 | \end{figure} 840 | 841 | \begin{example} 842 | 设一动点$P$, 到两点$A$、$B$的距 843 | 离的平方和等于常数$k$, 求$P$点的轨迹. 844 | \end{example} 845 | 846 | \begin{solution} 847 | 取$\overline{AB}$的中点$O$, 则 848 | \[\Vec{PA}=\Vec{PO}+\Vec{OA},\qquad \Vec{PB}=\Vec{PO}+\Vec{OB}\] 849 | \[\begin{split} 850 | \overline{PA}^2&=\overline{PO}^2+\overline{OA}^2+2\Vec{PO}\cdot\Vec{OA}\\ 851 | \overline{PB}^2&=\overline{PO}^2+\overline{OB}^2+2\Vec{PO}\cdot \Vec{OB} 852 | \end{split}\] 853 | 因为$\overline{PA}^2+\overline{PB}^2=k,\quad \Vec{PO}\cdot\Vec{OA}=-\Vec{PO}\cdot \Vec{OB}$ 854 | 855 | 所以: 856 | \[2\left(\overline{PO}^2+\overline{OA}^2\right)=k,\qquad \overline{PO}=\sqrt{\frac{k}{2}-\overline{OA}^2}\] 857 | 于是, 858 | $P$点到$O$点的距离是一个常数.即$P$点的轨迹是以$O$为 859 | 圆心,$\sqrt{\frac{k}{2}-\overline{OA}^2}$为半径的圆. 860 | \end{solution} 861 | 862 | \begin{example} 863 | 已知正方形$ABCD$(图4.17), $P$为$\overline{BD}$上任一点, 864 | $\overline{PE}\bot \overline{BC}$于$E$点,$\overline{PF}\bot \overline{CD}$于$F$点,求证: 865 | $\overline{AP}=\overline{EF}$ 866 | 且$AP\bot EF$. 867 | \end{example} 868 | 869 | \begin{proof} 870 | 设$\Vec{BP}=\lambda \Vec{BD}$, 正方形边长为$a$, 则 871 | \[\begin{split} 872 | \Vec{AP}&=\Vec{AB}+\Vec{BP}+(1-\lambda)\Vec{AB}+\lambda\Vec{AD}\\ 873 | \Vec{EF}&=\Vec{EC}+\Vec{CF}=(1-\lambda)\Vec{BC}+\lambda\Vec{CD} 874 | \end{split}\] 875 | \[\begin{split} 876 | \overline{AP}^2&=(1-\lambda )^2 a^2+\lambda a^2+2\lambda(1-\lambda)\Vec{AB}\Vec{AD}\\ 877 | &=(1-\lambda )^2 a^2+\lambda a^2\\ 878 | \overline{EF}^2&=(1-\lambda )^2 a^2+\lambda a^2 879 | \end{split}\] 880 | 所以:$\overline{AP}=\overline{EF}$.因为 881 | \[\Vec{AP}\cdot \Vec{EF}=\lambda (1-\lambda)\Vec{AB}\cdot \Vec{CD}+\lambda(1-\lambda)\Vec{AD}\cdot \Vec{BC}=0\] 882 | 所以 883 | $\overline{AP} \bot \overline{EF}$. 884 | \end{proof} 885 | 886 | \begin{figure}[htp]\centering 887 | \begin{minipage}[t]{0.48\textwidth} 888 | \centering 889 | \begin{tikzpicture}[>=latex, scale=1] 890 | \tkzDefPoints{0/0/B, 3/0/C, 3/3/D, 0/3/A, 2.5/2.5/P, 3/2.5/F,2.5/0/E} 891 | \tkzDrawPolygon(A,B,C,D) 892 | \tkzDrawPolygon(P,E,F) 893 | \tkzDrawSegments(B,D A,P) 894 | \tkzLabelPoints[below](B,E,C) 895 | \tkzLabelPoints[above](A,D,P) 896 | \tkzLabelPoints[right](F) 897 | \end{tikzpicture} 898 | \caption{} 899 | \end{minipage} 900 | \begin{minipage}[t]{0.48\textwidth} 901 | \centering 902 | \begin{tikzpicture}[>=latex, scale=1] 903 | \tkzDefPoints{0/0/B, 4/0/C, 2.5/2.5/A, 2/0/D} 904 | \tkzDrawSegments(A,D) 905 | \tkzDrawSegments[->](B,C A,B C,A) 906 | \node at (1.25,1.25)[left]{$\vec{c}$}; 907 | \node at (3.25,1.25)[right]{$\vec{b}$}; 908 | \node at (1.5,0)[below]{$\vec{a}$}; 909 | \node at (2.25,1.25)[right]{$m_a$}; 910 | \tkzLabelPoints[below](B,D,C) 911 | \tkzLabelPoints[above](A) 912 | \end{tikzpicture} 913 | \caption{} 914 | \end{minipage} 915 | \end{figure} 916 | 917 | \begin{example} 918 | 已知$\triangle ABC$ (图4.18), 919 | $\overline{BC}=a$, $\overline{CA}=b$, $\overline{AB}=c$, 920 | 求: 921 | \begin{enumerate} 922 | \item 三边上的中线$m_a$, $m_b$, $m_c$; 923 | \item 三个角平分线$t_a$、$t_b$、$t_c$; 924 | \item 三角形的面积$S$. 925 | \end{enumerate} 926 | \end{example} 927 | 928 | \begin{solution} 929 | \begin{enumerate} 930 | \item 如图:设$\Vec{BC}=\vec{a}$, $\Vec{CA}=\vec{b}$, $\Vec{AB}=\vec{c}$, 则$BC$边上的中线 931 | $\Vec{AD}=\frac{1}{2}(-\vec{b}+\vec{c})$, 932 | \[\begin{split} 933 | \Vec{AD}\cdot \Vec{AD}&=\frac{1}{4}(b^2+c^2-2\vec{b}\cdot \vec{c})\\ 934 | |\Vec{AD}|^2&=\frac{1}{4}[b^2+c^2-(a^2-b^2-c^2)]=\frac{1}{4}(2b^2+2c^2-a^2) 935 | \end{split}\] 936 | 因此 937 | \[m_a=\frac{1}{2}\sqrt{2b^2+2c^2-a^2}\] 938 | 同理可得: 939 | \[ 940 | m_b=\frac{1}{2}\sqrt{2a^2+2c^2-b^2},\qquad 941 | m_c=\frac{1}{2}\sqrt{2a^2+2b^2-c^2} 942 | \] 943 | 944 | \item 因$\vec{t}_a=\frac{c}{b+c}\vec{b}+\frac{b}{b+c}\vec{c}$,则: 945 | \[\begin{split} 946 | \left|\vec{t_{a}}\right|^{2} &=\frac{c^{2} b^{2}}{(b+c)^{2}}+\frac{b^{2} c^{2}}{(b+c)^{2}}+2 \frac{b c}{(b+c)^{2}} \vec{b} \cdot \vec{c} \\ 947 | &=\frac{b c}{(b+c)^{2}}\left(2 b c+b^{2}+c^{2}-a^{2}\right) \\ 948 | &=\frac{b c}{(b+c)^{2}}\left[(b+c)^{2}-a^{2}\right] \\ 949 | &=\frac{b c}{(b+c)^{2}}[(a+b+c)(b+c-a)] 950 | \end{split}\] 951 | 令$a+b+c=2p$, 则 952 | \[\begin{split} 953 | t^2_a&=\frac{4bc}{(b+c)^2}p(p-a)\\ 954 | t_a&=\frac{2\sqrt{bc}}{b+c}\sqrt{p(p-a)} 955 | \end{split}\] 956 | 同理可得: 957 | \[ 958 | t_b=\frac{2\sqrt{ac}}{a+c}\sqrt{p(p-b)},\qquad 959 | t_c=\frac{2\sqrt{ab}}{a+b}\sqrt{p(p-c)} 960 | \] 961 | 962 | \item 963 | \[\begin{split} 964 | S^{2} &=\frac{1}{4} b^{2} c^{2} \sin ^{2} A=\frac{1}{4} b^{2} c^{2}\left(1-\cos ^{2} A\right) \\ 965 | &=\frac{1}{4} b^{2} c^{2}\left[1-\left(\frac{\vec{b} \cdot \vec{c}}{b c}\right)^{2}\right] =\frac{1}{4} b^{2} c^{2}\left[1-\frac{\left(b^{2}+c^{2}-a^{2}\right)^{2}}{4 b^{2} c^{2}}\right] \\ 966 | &=\frac{1}{16}\left[4 b^{2} c^{2}-\left(b^{2}+c^{2}-a^{2}\right)^{2}\right] \\ 967 | &=\frac{1}{16}(a+b+c)(a+b-c)(a+c-b)(b+c-a) \\ 968 | &=p(p-a)(p-b)(p-c) \\ 969 | \end{split}\] 970 | \[ S=\sqrt{p(p-a)(p-b)(p-c)}\] 971 | \end{enumerate} 972 | \end{solution} 973 | 974 | \begin{example} 975 | 证明定理:如果一条直线$a$垂直于平面$\pi$上的两条 976 | 相交直线$b$、$c$, 那么$a\bot\pi $ (图4.19). 977 | \end{example} 978 | 979 | \begin{proof} 980 | 在平面$\pi$上,任取一条直 981 | 线$d$, 在$a,b,c,d$上分别取向量$\vec{a}$、$\vec{b}$、$\vec{c}$、$\vec{d}$. 由于$b$、$c$相交, 982 | 依共面向量定理,存在唯一的数偶 983 | $(x,y)$, 使 984 | \[\begin{split} 985 | \vec{d}&=x\vec{b}+y\vec{c}\\ 986 | \vec{a}\cdot \vec{d}&=\vec{a}\cdot \left(x\vec{b}+y\vec{c}\right)=x\vec{a}\cdot \vec{b}+y\vec{a}\cdot \vec{c}=0 987 | \end{split}\] 988 | 所以:$\vec{a}\bot \vec{d}$,即:$a\bot d$ 989 | 990 | 这就是说$a$垂直于平面$\pi$上的任一条直线,所以$a\bot\pi$. 991 | \end{proof} 992 | 993 | \begin{figure}[htp]\centering 994 | \begin{minipage}[t]{0.48\textwidth} 995 | \centering 996 | \begin{tikzpicture}[>=latex, yscale=.8] 997 | \tkzDefPoints{0/0/A, 4/0/B, 5/2.5/C, 1/2.5/D} 998 | \tkzDrawPolygon(A,B,C,D) 999 | \draw[->](3,.5)--(1.4,1.7)node[above right]{$\vec{b}$}; 1000 | \draw[->](1,.5)--(3.5,2)node[right]{$\vec{c}$}; 1001 | \draw[->](3.5,.5)--node[right]{$\vec{d}$}(4,1.5); 1002 | \node at (0,0)[above right]{$\pi$}; 1003 | \draw[->](2,2)--node[right]{$\vec{a}$}(2,3.5); 1004 | \end{tikzpicture} 1005 | \caption{} 1006 | \end{minipage} 1007 | \begin{minipage}[t]{0.48\textwidth} 1008 | \centering 1009 | \begin{tikzpicture}[>=latex, scale=1] 1010 | \tkzDefPoints{0/0/A, 4/0/C, 2.3/-1/B, 2.8/2/O} 1011 | \tkzDrawPolygon(O,A,B,C) 1012 | \tkzDrawSegments(O,B) 1013 | \tkzDefMidPoint(O,A) \tkzGetPoint{E} 1014 | \tkzDefMidPoint(O,B) \tkzGetPoint{F} 1015 | \tkzDefMidPoint(B,C) \tkzGetPoint{G} 1016 | \tkzDefMidPoint(C,A) \tkzGetPoint{H} 1017 | \tkzDrawSegments[dashed](A,C E,F F,G G,H E,H) 1018 | \tkzLabelPoints[above](O) 1019 | \tkzLabelPoints[right](F,G) 1020 | \tkzLabelPoints[left](E) 1021 | \tkzLabelPoints[below](A,B,C,H) 1022 | \end{tikzpicture} 1023 | \caption{} 1024 | \end{minipage} 1025 | \end{figure} 1026 | 1027 | \begin{example} 1028 | 已知空间四边形$O-ABC$, $\overline{OA}=\overline{OB}$, $\overline{CA}=\overline{CB}$. 1029 | $E$、$F$、$G$、$H$分别为$\overline{OA}$、$\overline{OB}$、$\overline{CB}$、$\overline{CA}$的中点,求证 1030 | 四边形$EFGH$是矩形(图4.20). 1031 | \end{example} 1032 | 1033 | \begin{proof} 1034 | 由于$E$、$F$、$G$、$H$分别是$\overline{OA}$ 1035 | 、$\overline{OB}$、$\overline{CB}$、$\overline{CA}$的中点,所以 1036 | \[\Vec{EF}=\frac{1}{2}\Vec{AB},\qquad \Vec{HG}=\frac{1}{2}\Vec{AB}\] 1037 | \[\Vec{EF}=\Vec{HG}\quad \Rightarrow\quad \overline{EF}=\overline{HG},\quad EF\parallel HG\] 1038 | $\therefore\quad EFGH$是平行四边形. 1039 | 1040 | 因为 1041 | $\Vec{EF}=\frac{1}{2}\Vec{AB}=\frac{1}{2}(\Vec{OB}-\Vec{OA}),\quad \Vec{EH}=\frac{1}{2}\Vec{OC}$,所以 1042 | \[\begin{split} 1043 | \Vec{E F} \cdot \Vec{E H} &=\frac{1}{2}(\Vec{O B}-\Vec{O A}) \cdot \frac{1}{2} \Vec{O C} \\ 1044 | &=\frac{1}{4}(\Vec{O B} \cdot \Vec{O C}-\Vec{O A} \cdot \Vec{O C}) \\ 1045 | &=\frac{1}{4}\left(|\Vec{O B}|^{2}+|\Vec{O C}|^{2}-|\Vec{B C}|^{2}-|\Vec{O A}|^{2}-|\Vec{O C}|^{2}+|\Vec{C A}|^{2}\right) \\ 1046 | \end{split}\] 1047 | 但已知$\Vec{O A}=\Vec{O B},\qquad \Vec{C A}=\Vec{C B}$,所以 1048 | \[\Vec{E F} \cdot \Vec{E H} =0 \quad \Rightarrow\quad E F \bot E H\] 1049 | 故得四边形$EFGH$是矩形. 1050 | \end{proof} 1051 | 1052 | 由以上各例可看到在空间证明两线垂直,内积运算仍是 1053 | 非常有效的工具. 1054 | 1055 | \begin{example} 1056 | 一定长线段$\overline{AB}$的两个端 1057 | 点,沿互相垂直的两条异面直线$\ell$、 1058 | $m$运动,求它的中点的轨迹. 1059 | \end{example} 1060 | 1061 | \begin{solution} 1062 | 如图4.21设$\overline{MN}$为$\ell$、$m$的 1063 | 公垂线,$\overline{AB}=a$, $\overline{MN}=b$, $O$、$P$分 1064 | 别为$\overline{MN}$、$\overline{AB}$的中点, 1065 | 则 1066 | \[\Vec{OP}=\frac{1}{2}\left(\Vec{OA}+\Vec{OB}\right)=\frac{1}{2}\left(\Vec{OM}+\Vec{MA}+\Vec{ON}+\Vec{NB}\right)\] 1067 | 因为$\Vec{OM}=-\Vec{ON}$,所以 1068 | \[\begin{split} 1069 | \Vec{OP}&=\frac{1}{2}(\Vec{MA}+\Vec{NB})\\ 1070 | \Vec{OP}\cdot\Vec{MN}&=\frac{1}{2}\left(\Vec{MA}+\Vec{NB}\right)\cdot \Vec{MN}=0 1071 | \end{split} \] 1072 | 因此:$P$点一定在$\overline{MN}$的垂直平分面上. 1073 | 1074 | 因为$\Vec{OP}\cdot \Vec{OP}=\frac{1}{4}(|\Vec{MA}|^2+|\Vec{NB}|^2)$, 1075 | 连$AN$, 易证,$\triangle AMN$与$\triangle ABN$都是直角三角形. 1076 | 所以 1077 | \[\begin{split} 1078 | \Vec{OP}\cdot \Vec{OP}&=\frac{1}{4}\left(\overline{AN}^2-\overline{MN}^2+\overline{AB}^2-\overline{AN}^2\right)\\ 1079 | &=\frac{1}{4}\left(\overline{AB}^2-\overline{MN}^2\right)=\frac{1}{4}(a^2-b^2) 1080 | \end{split}\] 1081 | 即:$|\Vec{OP}|^2=\frac{1}{4}(a^2-b^2)$ 1082 | 1083 | 由上式可知$P$点在以$O$为圆心,以$\frac{1}{2}\sqrt{a^2-b^2}$为半径的 1084 | 圆上,因此$P$点的轨迹是$\overline{MN}$的垂直平分面上的一个圆: 1085 | $\odot\left(O,\frac{1}{2}\sqrt{a^2-b^2}\right)$ 1086 | \end{solution} 1087 | 1088 | \begin{figure}[htp]\centering 1089 | \begin{minipage}[t]{0.48\textwidth} 1090 | \centering 1091 | \begin{tikzpicture}[>=latex, scale=1] 1092 | \tkzDefPoints{0/3/A, 0/2/M, 3/2/N, 1.5/0.5/B} 1093 | \tkzDefMidPoint(M,N) \tkzGetPoint{O} 1094 | \tkzDefMidPoint(A,B) \tkzGetPoint{P} 1095 | \tkzDrawSegments(A,B A,N M,N O,P) 1096 | \tkzDrawLines[add = 1 and 1](A,M) 1097 | \tkzDrawLines[add = .5 and .5](B,N) 1098 | \node at (0,4)[right]{$\ell$}; 1099 | \node at (3.5,3)[right]{$m$}; 1100 | \tkzLabelPoints[left](A,M,P) 1101 | \tkzLabelPoints[right](B,N) 1102 | \tkzLabelPoints[above](O) 1103 | \end{tikzpicture} 1104 | \caption{} 1105 | \end{minipage} 1106 | \begin{minipage}[t]{0.48\textwidth} 1107 | \centering 1108 | \begin{tikzpicture}[>=latex, scale=1] 1109 | \tkzDefPoints{0/0/B, 4/0/D, 2.8/-1/C, 2.3/2/A} 1110 | \tkzDrawPolygon(D,A,B,C) 1111 | \tkzDrawSegments[dashed](D,B) 1112 | \tkzDrawSegments(A,C) 1113 | \tkzLabelPoints[below](D,B,C) 1114 | \tkzLabelPoints[above](A) 1115 | \end{tikzpicture} 1116 | \caption{} 1117 | \end{minipage} 1118 | \end{figure} 1119 | 1120 | \begin{example} 1121 | 如图4.22, 已知四面体 1122 | $A-BCD$, $AB\bot CD$, $AC\bot BD$. 1123 | 1124 | 求证:$AD\bot BC$. 1125 | \end{example} 1126 | 1127 | \begin{proof} 1128 | $\because\quad AB\bot CD,\quad AC\bot BD$ 1129 | 1130 | 所以 1131 | \[ \Vec{AB}\cdot \Vec{CD}=0,\qquad \Vec{AC}\cdot \Vec{BD}=0\] 1132 | \[\left(\Vec{AD}+\Vec{DB}\right)\cdot \Vec{CD}=0,\qquad \left(\Vec{AD}+\Vec{DC}\right)\cdot \Vec{BD}=0\] 1133 | \[\Vec{AD}\cdot \Vec{CD}=\Vec{BD}\cdot \Vec{CD},\qquad \Vec{AD}\cdot \Vec{BD}=\Vec{BD}\cdot \Vec{CD}\] 1134 | \[\begin{split} 1135 | \Vec{AD}\cdot \Vec{CD}&=\Vec{AD}\cdot \Vec{BD} 1136 | \\ 1137 | \Vec{AD}\cdot \left(\Vec{BD}+\Vec{DC}\right)&=0\\ 1138 | \Vec{AD}\cdot \Vec{BC}&=0 1139 | \end{split}\] 1140 | 即:$\Vec{AD}\bot \Vec{BC},\qquad AD\bot BC$ 1141 | \end{proof} 1142 | 1143 | 1144 | \begin{example} 1145 | 如图4.23在直二面角的棱上有两点$A$、$B$, $AC$和 1146 | $BD$各在这个二面角的一个面内,并且都垂直于棱$AB$,设 1147 | $\overline{AB}=8$, $\overline{AC}=6$, $\overline{BD}=24$, 求$\overline{CD}$的长. 1148 | \end{example} 1149 | 1150 | \begin{solution} 1151 | 如图4.23, 1152 | \[|\Vec{CD}|^2=\Vec{CB}\cdot \Vec{CD}=\left(\Vec{CA}+\Vec{AB}+\Vec{BD}\right)\cdot \left(Vec{CA}+\Vec{AB}+\Vec{BD}\right)\] 1153 | 因为 1154 | $Vec{AC}$、$Vec{AB}$、$Vec{BD}$互相正交, 1155 | 所以 1156 | \[|\Vec{CD}|^2=|\Vec{CA}|^2+|\Vec{AB}|^2+|\Vec{BD}|^2=6^2+8^2+24^2=676\] 1157 | $\therefore\quad \overline{CD}=\sqrt{676}=26$ 1158 | \end{solution} 1159 | 1160 | \begin{figure}[htp]\centering 1161 | \begin{minipage}[t]{0.48\textwidth} 1162 | \centering 1163 | \begin{tikzpicture}[>=latex, scale=1] 1164 | \tkzDefPoints{0/0/a1, .5/1.9/a2, 1.5/-1.8/a3, 3/0/a4} 1165 | \tkzDefPointsBy[translation= from a1 to a4](a2,a3){a2',a3'} 1166 | \tkzDrawPolygon(a1,a2,a2',a4) 1167 | \tkzDrawPolygon(a1,a3,a3',a4) 1168 | \draw(1-.5,0)node[below]{$A$}--(1.25-.5,0.95)node[above]{$C$}--(2.5+.75,-.9)node[below]{$D$}--(1+1.5,0)node[above]{$B$}; 1169 | \end{tikzpicture} 1170 | \caption{} 1171 | \end{minipage} 1172 | \begin{minipage}[t]{0.48\textwidth} 1173 | \centering 1174 | \begin{tikzpicture}[>=latex, scale=.8] 1175 | \tkzDefPoints{0/0/B, 4/0/C, 5/1/D, 1/1/A, 0/4/F} 1176 | \tkzDefPointsBy[translation= from B to F](C,D,A){G,H,E} 1177 | \tkzDrawPolygon(E,F,G,H) 1178 | \tkzDrawSegments(B,C C,D B,F C,G H,D G,D) 1179 | \tkzDrawSegments[->, dashed](A,B A,D A,E A,G) 1180 | 1181 | \tkzDefPointWith[linear, K=.65](G,D) \tkzGetPoint{Q} 1182 | \tkzDefPointWith[linear, K=.6](A,C) \tkzGetPoint{P} 1183 | \tkzDrawSegments[dashed](A,C P,Q) 1184 | \node at (.75,.75)[left]{$\vec{e_1}$}; 1185 | \node at (1,2.5)[right]{$\vec{e_3}$}; 1186 | \node at (3,1)[above]{$\vec{e_2}$}; 1187 | \tkzLabelPoints[below](B,C) 1188 | \tkzLabelPoints[above](E,H,G) 1189 | \tkzLabelPoints[right](Q,D) 1190 | \tkzLabelPoints[left](F,A,P) 1191 | \end{tikzpicture} 1192 | \caption{} 1193 | \end{minipage} 1194 | \end{figure} 1195 | 1196 | \begin{example} 1197 | 已知正方体$ABCD-EFGH$ (图4.24) 其棱长为 1198 | 1. 求:$AC$与$DG$的公垂线的垂足$P$、$Q$的位置和$AC$与$DG$ 1199 | 间的距离. 1200 | \end{example} 1201 | 1202 | \begin{solution} 1203 | 设$\Vec{AB}=\vec{e}_1$, $\Vec{AD}=\vec{e}_2$, $\Vec{AE}=\vec{e}_3$, $\Vec{AP}=x\Vec{AC}$, $\Vec{DQ}=y\Vec{DG}$ 1204 | 1205 | 由于$\Vec{AC}=\vec{e}_1+\vec{e}_2,\quad \Vec{DG}=\vec{e}_1+\vec{e}_3$,所以 1206 | \[\Vec{AP}=x\vec{e}_1+x\vec{e}_2,\qquad \Vec{AQ}=y\vec{e}_1+y\vec{e}_3\] 1207 | \[\begin{split} 1208 | \Vec{AQ}&=\Vec{AD}+\Vec{DQ}=y\vec{e}_1+\vec{e}_2+y\vec{e}_3\\ 1209 | \Vec{PQ}&=\Vec{AQ}-\Vec{AP}=(y-x)\vec{e}_1+(1-x)\vec{e}_2+y\vec{e}_3\\ 1210 | \end{split}\] 1211 | 又因 $\Vec{P Q} \perp \Vec{A C}$, $\Vec{P Q} \perp \Vec{D G}$, 则 1212 | \[\begin{cases} 1213 | {\left[(y-x) \vec{e}_{1}+(1-x) \vec{e}_{2}+y \vec{e}_{3}\right] \cdot\left(\vec{e}_{1}+\vec{e}_{2}\right)=0} \\ 1214 | {\left[(y-x) \vec{e}_{1}+(1-x) \vec{e}_{2}+y \vec{e}_{3}\right] \cdot\left(\vec{e}_{1}+\vec{e}_{3}\right)=0} 1215 | \end{cases}\] 1216 | 由此可得方程组 1217 | \[\begin{cases} 1218 | 2 x-y=1 \\ 1219 | x-2 y=0 1220 | \end{cases}\] 1221 | 解之得:$x=\frac{2}{3}, \qquad y=\frac{1}{3}$ 1222 | 1223 | 所以:$\Vec{AP}=\frac{2}{3}\Vec{AC},\quad \Vec{DQ}=\frac{1}{3}\Vec{DG}$ 1224 | \[\Vec{PQ}=\frac{1}{3}\left(-\vec{e}_1+\vec{e}_2+\vec{e}_3\right)\] 1225 | 于是$P$、$Q$两点的位置可定,且$AC$与$DG$的距离就是$|\Vec{PQ}|$. 1226 | \[|\Vec{PQ}|=\sqrt{\Vec{PQ}\cdot \Vec{PQ}}=\sqrt{\frac{1}{9}(1+1+1)}=\frac{\sqrt{3}}{3}\] 1227 | \end{solution} 1228 | 1229 | 1230 | \section*{习题4.2} 1231 | \addcontentsline{toc}{subsection}{习题4.2} 1232 | 1233 | 试用向量运算证明以下各题: 1234 | 1235 | \begin{enumerate} 1236 | \item 试证:点$P$在$\overline{AB}$的垂直平分上的充要条件是$|\Vec{PA}|=|\Vec{PB}|$ 1237 | \item 试证:对角线互相垂直的平行四边形是菱形. 1238 | \item 试证:平行四边形的对角线等长的充要条件是这个平行 1239 | 四边形是矩形. 1240 | \item 求证:直角三角形斜边上的中线等于斜边的一半. 1241 | \item 等腰三角形顶角的平分线是底边上的高. 1242 | \item 求证:四边形$ABCD$中,对角线互相垂直的充要条件是 1243 | \[\overline{AB}^2 +\overline{CD}^2=\overline{AD}^2+\overline{BC}^2\] 1244 | \item 求证:平行四边形$ABCD$中的锐角$A$为$45^{\circ}$的充要条件 1245 | 是$$\overline{AC}^2\cdot \overline{BD}^2=\overline{AB}^4+\overline{AD}^4$$ 1246 | \item 已知$\ell$是一条直线,$A$、$B$为$\ell$外同侧的两个定点,$P\in\ell$, 1247 | 1248 | 求 1249 | 证:$|\Vec{AP}|+|\Vec{PB}|$取极小值的充要条件是 1250 | \[\frac{\vec{a}\cdot \vec{n}}{|\vec{a}|}=\frac{\vec{b}\cdot \vec{n}}{|\vec{b}|}\] 1251 | (其中$\vec{n}\bot \ell$且方向指向$A$、$B$所在的 1252 | 那一侧)并解释其几何意义. 1253 | \item 求证:长方体的对角线的平方等于长、宽、高的平方 1254 | 和. 1255 | \item 已知$\overline{AB}$在平面$\pi$内,$\overline{AC}\bot\pi$, $\overline{BD}\bot \overline{AB}$且与$\pi$所成 1256 | 角为$30^{\circ}$, 若$\overline{AB}=a$, $\overline{AC}=\overline{BD}=b$, 求$C$和$D$间的 1257 | 距离. 1258 | \item 线段 1259 | $\overline{DE}$ 1260 | 同时垂直于矩形$ABCD$的两边 1261 | $\overline{DA}$、$\overline{DC}$. 设 1262 | $\overline{AB}=12$cm, $\overline{BC}=9$cm, $\overline{DE} 1263 | =8$cm, 求$B$、$E$两点间 1264 | 的距离. 1265 | \item 一点$P$在两个相交平面上的投影各为$A$、$B$, 求证连线$AB$ 1266 | 垂直于两个平面的交线. 1267 | \end{enumerate} 1268 | 1269 | 1270 | \section{圆} 1271 | 1272 | 一个以$O$点为圆心,$R$为半径的圆是平面$\pi$上所有满足 1273 | $\Vec{OX}\cdot\Vec{OX}=R^2$的点$X$的集合.即 1274 | \[\odot (O,R)=\left\{X:X\in \pi,\; \Vec{OX}\cdot\Vec{OX}=R^2\right\}\] 1275 | 1276 | 在平面几何中,关于圆有下列几个基本定理: 1277 | \begin{enumerate} 1278 | \item 圆周角定理; 1279 | \item 弦切角定理; 1280 | \item 圆幂定理. 1281 | \end{enumerate} 1282 | 1283 | 当时,这三个定理是按1、2、3的顺序来证明的,现在我们用向量的运算律,直接证明圆幂定理, 1284 | 并由此再推出弦切角定理与圆周角定理. 1285 | 1286 | \begin{blk}{定理1} 1287 | 如图4.25设平面上有$\odot (O,R)$及圆外一点$P$, 过 1288 | $P$点引圆割线交圆于$B$、$C$点,则有$\Vec{PB}\cdot \Vec{PC}=|OP|^2-R^2$. 1289 | \end{blk} 1290 | 1291 | \begin{figure}[htp] 1292 | \centering 1293 | \begin{tikzpicture}[>=latex, scale=1.6] 1294 | \tkzDefPoints{0/0/O, -2.5/0/P} 1295 | \tkzDefPoint(80:.5){C1} 1296 | \tkzDefPoint(60:1){C2} 1297 | \draw(O) circle(1); 1298 | \tkzInterLC(C1,P)(O,C2) \tkzGetPoints{B}{C} 1299 | 1300 | \tkzDefPointWith[linear, K=0.8](B,C) \tkzGetPoint{X} 1301 | \tkzDrawSegments[->](P,C O,X) 1302 | \tkzLabelPoints[above](C,X) 1303 | \tkzLabelPoints[left](P) 1304 | \tkzLabelPoints[below](O) 1305 | \tkzLabelPoints[below left](B) 1306 | \draw(P)--(O); 1307 | \draw(P)--+(23.58:3.5); 1308 | \node at (-.4,.916)[above]{$T_1$}; 1309 | \tkzDefPointWith[linear, K=0.4](P,B) \tkzGetPoint{X1} 1310 | \draw[thick,->](P)--(X1)node[above left]{$\vec{u}$}; 1311 | \end{tikzpicture} 1312 | \caption{} 1313 | \end{figure} 1314 | 1315 | 1316 | \begin{proof} 1317 | 在$\Vec{PC}$上取单位向量 1318 | $\vec{u}$, 对$PC$上任意一点$X$, 都存在实数$x$, 使 1319 | $\Vec{PX}=x\vec{u}$.这里$x$就是$\Vec{PX}$ 1320 | 的长度.由向量加法,有 1321 | \[\Vec{OX}=\Vec{OP}+\Vec{PX}=\Vec{OP}+x\vec{u}\] 1322 | 因此: 1323 | \[\begin{split} 1324 | \Vec{OX}\cdot \Vec{OX}&=\left(\Vec{OP}+x\vec{u}\right)^2\\ 1325 | &=\Vec{OP}\cdot \Vec{OP}+2\left(\Vec{OP}\cdot \vec{u}\right)x+x^2(\vec{u}\cdot \vec{u})\\ 1326 | &=x^2+2\left(\Vec{OP}\cdot \vec{u}\right)+|\Vec{OP}|^2 1327 | \end{split}\] 1328 | 1329 | 当点$X$与点$B$或$C$重合时,$\Vec{OX}\cdot \Vec{OX}=R^2$,这时有 1330 | \[x^2+2\left(\Vec{OP}\cdot \vec{u}\right)x+\left(|\Vec{OP}|^2-R^2\right)=0\] 1331 | 这里$x$是$\Vec{PB}$或$\Vec{PC}$的长度,若令$|\Vec{PB}|=\beta$, 1332 | $|\Vec{PC}|=\gamma$, 1333 | 则由韦达定理得: 1334 | \[\beta\gamma=|\Vec{OP}|^2-R^2\] 1335 | 1336 | $\because\quad \Vec{PB}$与$\Vec{PC}$ 1337 | 同向 1338 | 1339 | $\therefore\quad \Vec{PB}\cdot \Vec{PC}=\beta\gamma$, 因此: 1340 | \[\Vec{PB}\cdot \Vec{PC}=|\Vec{OP}|^2-R^2=|PT_1|^2\] 1341 | \end{proof} 1342 | 1343 | \begin{analyze} 1344 | \begin{enumerate} 1345 | \item 上面的证明对$P$为圆内一点也是适用的,不过 1346 | 这时设$\Vec{PX}=x\vec{u}$, 实数$x$可正可负,$|x|=|\Vec{PX}|$. 1347 | 1348 | 由于$\Vec{PB}$与 1349 | $\Vec{PC}$反向,则$\Vec{PB}\cdot \Vec{PC}$和$|OP|^2-R^2$都是负值. 1350 | 1351 | \item 在证明过程中,我们还得到 1352 | \[\beta+\gamma=-2\Vec{OP}\cdot \vec{u}\] 1353 | 我们令$B$、$C$、$P'$、$P$成调和点列,即此四点共线 1354 | 且满足 1355 | \[\frac{\Vec{BP}}{\Vec{PC}}\cdot \frac{\Vec{CP'}}{\Vec{P'B}}=-1\] 1356 | 令$\Vec{PP'}=y\vec{u}$,则有 1357 | \[-1=\frac{\Vec{BP}}{\Vec{PC}}\cdot \frac{\Vec{CP'}}{\Vec{P'B}}=\frac{-\beta}{\gamma}\x \frac{y-\gamma}{\beta-y}\] 1358 | 得:$y=\frac{2\beta\gamma}{\beta+\gamma}$,因此: 1359 | \[\begin{split} 1360 | \Vec{PP'}\cdot \Vec{PO}&=\left(\frac{2\beta \gamma}{\beta+\gamma}\vec{u}\right)\cdot \left(-\Vec{OP}\right)\\ 1361 | &=\frac{\beta \gamma}{\beta+\gamma}\left(-2\Vec{OP}\cdot \vec{u}\right)\\ 1362 | &=\beta\gamma =|\Vec{OP}|^2-R^2 1363 | \end{split}\] 1364 | 这就是说$\Vec{PP'}$在$\Vec{PO}$上的投影的长度等于常数 1365 | $\frac{|\Vec{OP}|^2-R^2}{|\Vec{OP}|}$ 1366 | 1367 | 由于 1368 | \[\frac{|\Vec{OP}|^2-R^2}{|\Vec{OP}|}=\frac{|\Vec{PT_1}|^2}{|\Vec{OP}|}=|\Vec{PT_1}|\cos\angle T_1PO=|PQ|\] 1369 | 其中$Q$是线段$T_1T_2$的中点(图4.26) 1370 | 1371 | $\therefore\quad $过$P$点的任一割线与$T_1T_2$的交 1372 | 点$P'$与$B$、$C$、$P$成调和点列. 1373 | \end{enumerate} 1374 | \end{analyze} 1375 | 1376 | 下面,我们由圆幂定理推导 1377 | 弦切角定理与圆周角定理. 1378 | 1379 | \begin{figure}[htp]\centering 1380 | \begin{minipage}[t]{0.48\textwidth} 1381 | \centering 1382 | \begin{tikzpicture}[>=latex, scale=1.5] 1383 | \tkzDefPoints{0/0/O, -2/0/P} 1384 | \tkzDefPoint(120:1){T_1} \tkzDefPoint(-120:1){T_2} 1385 | \draw(0,0) circle (1); 1386 | \tkzDrawSegments(P,O P,T_1 P,T_2 T_1,T_2) 1387 | \tkzLabelPoints[above](P,T_1) 1388 | \tkzLabelPoints[below](O,T_2) 1389 | \node at (-.5,0)[above right]{$Q$}; 1390 | \end{tikzpicture} 1391 | \caption{} 1392 | \end{minipage} 1393 | \begin{minipage}[t]{0.48\textwidth} 1394 | \centering 1395 | \begin{tikzpicture}[>=latex, scale=1.5] 1396 | \tkzDefPoints{0/0/O, -2.5/-1/P,0/-1/T} 1397 | \tkzDefPoint(-15:1){C} \tkzDefPoint(60:1){C'} 1398 | \draw(0,0) circle (1); 1399 | \tkzInterLC(C,P)(O,C') \tkzGetPoints{B'}{B} 1400 | \tkzInterLL(C',B)(P,T) \tkzGetPoint{P'} 1401 | \tkzDrawSegments[thick](C,T C',T C,B C',B P,T P,C B,T B,P') 1402 | \tkzLabelPoints[below](P,T,P') 1403 | \tkzLabelPoints[right](C,C') 1404 | \tkzLabelPoints[above](B) 1405 | \end{tikzpicture} 1406 | \caption{} 1407 | \end{minipage} 1408 | \end{figure} 1409 | 1410 | 1411 | 如图4.27由圆幂定理,在 1412 | $\triangle PTB$与$\triangle PCT$中, 1413 | 由于$\frac{\Vec{PT}}{\Vec{PB}}=\frac{\Vec{PC}}{\Vec{PT}}$ 1414 | ,$\angle P$是公共角, 1415 | 1416 | $\therefore\quad \triangle PTB \backsim \triangle PCT$ 1417 | 1418 | 得$\angle PTB=\angle PCT$, 1419 | , 这就是弦切角定理. 1420 | 1421 | 同理$\angle P'TB=\angle BC'T$, 1422 | 则$\angle BCT=\angle BC'T$, 这就是圆周 1423 | 角定理. 1424 | 1425 | 1426 | \section*{习题4.3} 1427 | \addcontentsline{toc}{subsection}{习题4.3} 1428 | 1429 | \begin{enumerate} 1430 | \item 试证圆的相交弦定理. 1431 | \item 从圆$O$外一点$P$引圆的切线$PT_1$和$PT_2$, $T_1$、$T_2$为切点. 1432 | 再引圆$O$的割线$PQR$, 交圆$O$于$Q$、$R$, 交$T_1T_2$于$T$, 1433 | 设$|PQ|=a$, $|PR|=b$, $|PT|=t$, 求证: 1434 | \[\frac{1}{a}+\frac{1}{b}=\frac{2}{t}\] 1435 | \end{enumerate} 1436 | 1437 | \section*{复习题四} 1438 | \addcontentsline{toc}{section}{复习题四} 1439 | 1440 | \begin{enumerate} 1441 | 1442 | \item 在空间中,设有线性关系$\lambda_1\vec{a}_1+\lambda_2\vec{a}_2+\lambda_3\vec{a}_3+\lambda_4\vec{a}_4+\lambda_5\vec{a}_5=\vec{0}$,且$\lambda_1\lambda_2\lambda_3\lambda_4\lambda_5\ne 0$, 若 1443 | \begin{enumerate} 1444 | \item $\vec{a}_1,\vec{a}_2,\vec{a}_3,\vec{a}_4,\vec{a}_5$都是非零向量; 1445 | \item 有且只有$\vec{a}_4=\vec{a}_5=\vec{0}$; 1446 | \item 有且只有三个零向量. 1447 | \end{enumerate} 1448 | 问在各种情况下,它们的几何意义 1449 | 分别是什么? 1450 | \item 试作一给定有向线段$\Vec{AB}$的定比分点,其比值分别为: 1451 | $\frac{1}{2},2,-2,-\frac{1}{2}$. 1452 | \item 设$P$、$A$、$B$是共线的相异三点, 1453 | $\Vec{AP}=\rho\Vec{PB}$, 1454 | 试用$\rho$去表达下列五个实数$\alpha$、$\beta$、$\gamma$、$\delta$、$\varepsilon$. 1455 | \[\begin{split} 1456 | \Vec{BP}&=\alpha\Vec{PA},\qquad \Vec{PA}=\beta\Vec{AB},\qquad \Vec{BA}=\gamma\Vec{AP}\\ 1457 | \Vec{PB}&=\delta\Vec{BA},\qquad \Vec{AB}=\varepsilon\Vec{BP} 1458 | \end{split}\] 1459 | \item 如图,$\ell_1,\ell_2$交于$O$点,$\vec{u},\vec{v}$是$\ell_1,\ell_2$方向的单位向量, 1460 | 设 1461 | \[\begin{split} 1462 | \Vec{OA_1}=\alpha_1\vec{u},\qquad \Vec{OB_1}=\beta_1\vec{u},\qquad \Vec{OC_1}=\gamma_1\vec{u}\\ 1463 | \Vec{OA_2}=\alpha_2\vec{v},\qquad \Vec{OB_2}=\beta_2\vec{v},\qquad \Vec{OC_2}=\gamma_2\vec{v} 1464 | \end{split}\] 1465 | 试用$\vec{u},\vec{v}$的线性组合表示 1466 | $\Vec{OP}$、$\Vec{OQ}$、$\Vec{OR}$. 并证明$P$、$Q$、$R$三点共线. 1467 | 1468 | \begin{figure}[htp] 1469 | \centering 1470 | \begin{tikzpicture}[>=latex] 1471 | \tkzDefPoints{0/0/O, 2/0/A_2, 3.5/0/B_2, 5/0/C_2} 1472 | \tkzDefPoint(40:2.5){A_1}\tkzDefPoint(40:4){B_1} 1473 | \tkzDefPoint(40:5.5){C_1} 1474 | \tkzLabelPoints[above](A_1,B_1,C_1) 1475 | \tkzLabelPoints[below](A_2,B_2,C_2) 1476 | \draw(0,0)node[left]{$O$}--(6,0)node[right]{$\ell_2$}; 1477 | \draw(0,0)--(40:6)node[right]{$\ell_1$}; 1478 | \tkzDrawSegments(A_1,C_2 A_1,B_2 B_1,A_2 B_1,C_2 C_1,A_2 C_1,B_2) 1479 | \draw[thick,->](0,0)--node[below]{$\vec{v}$}(1,0); 1480 | \draw[thick,->](0,0)--node[above]{$\vec{u}$}(40:1); 1481 | 1482 | \tkzInterLL(A_1,B_2)(A_2,B_1) \tkzGetPoint{P}; 1483 | \tkzInterLL(A_1,C_2)(A_2,C_1) \tkzGetPoint{Q}; 1484 | \tkzInterLL(C_1,B_2)(C_2,B_1) \tkzGetPoint{R}; 1485 | \tkzDrawPoints(P,Q,R) 1486 | \tkzLabelPoints[right](R) 1487 | \tkzLabelPoints[left](P) 1488 | \tkzLabelPoints[above](Q) 1489 | 1490 | \end{tikzpicture} 1491 | \caption*{第4题} 1492 | \end{figure} 1493 | 1494 | 1495 | \item 在$\triangle ABC$的外面作正方形$ABEF$和$ACGH$, 又设$D$为 1496 | $\Vec{BC}$的中点,求证: 1497 | \begin{enumerate} 1498 | \item $\Vec{AF}\cdot \Vec{AH}=\Vec{AB}\cdot \Vec{AC}$ 1499 | \item $BH\bot CF$且$\overline{BH}=\overline{CF}$ 1500 | \item $AD\bot FH$且$\overline{AD}=\frac{1}{2} \overline{FH}$ 1501 | \end{enumerate} 1502 | 1503 | \item 已知四边形$ABCD$内接于圆且$AC\bot BD$于$E$, $F$是边 1504 | $\Vec{BC}$的中点,求证:$EF\bot AD$ 1505 | \item 已知$O$、$M$、$H$三点分别是$\triangle ABC$的外心,重心和 1506 | 垂心,求证:$O$、$M$、$H$三点共线且$\overline{OM}=\frac{1}{2}\overline{MH}$. 1507 | \item 求证:连结四面体的一个顶点和这个顶点所对的面的重 1508 | 心的四条线段交于同一点,且这交点分线段的比例都 1509 | 是3:1. 1510 | \item 求证平行六面体的四条对角线相交于一点. 1511 | \item 在四面体$ABCD$中,如果$AB\bot DC$且$AD\bot BC$, 试 1512 | 证明: 1513 | \[|\Vec{AB}|^2+|\Vec{DC}|^2=|\Vec{AD}|^2+|\Vec{BC}|^2=|\Vec{AC}|^2+|\Vec{BD}|^2\] 1514 | \item 已知四面体$ABCD$, $G_1,G_2$分别是$\triangle ABC$和$\triangle ABD$的 1515 | 重心,$M$是棱$CD$的中点,试确定过$G_1$、$G_2$、$M$三点的 1516 | 平面与棱$AB$的交点的位置. 1517 | \item 已知正方体$ABCD-A_1B_1C_1D_1$的棱长为1, $E$、$F$分别 1518 | 是棱$\overline{BC}$, $\overline{CC_1}$的中点,求下列各异面直线的距离. 1519 | \begin{multicols}{3} 1520 | \begin{enumerate} 1521 | \item $AA_1$与$BD_1$ 1522 | \item $AC$与$BD_1$ 1523 | \item $AC$与$EF$ 1524 | \end{enumerate} 1525 | \end{multicols} 1526 | \end{enumerate} -------------------------------------------------------------------------------- /6.tex: -------------------------------------------------------------------------------- 1 | \chapter{圆锥曲线} 2 | 远在古希腊时代,人们就开始研究一个平面和一个正瞬 3 | 锥的截线的性质,并且获得了丰硕的成果,这些截线分别有 4 | 椭圆、抛物线和双曲线,并统称为圆锥曲线,在第二章 5 | 附录里,我们已用球面切线长相等原理证明了它们分别具有 6 | 如下几何特征. 7 | 8 | 椭圆有两个焦点$F_1,F_2$, 对椭圆上任一点$P$有 9 | \[\overline{PF_1}+\overline{PF_2}=\text{常数}\] 10 | 11 | 双曲线有两个焦点$F_1,F_2$, 对双曲线上任一点$P$有 12 | \[\overline{PF_1}-\overline{PF_2}=\text{常数}\] 13 | 14 | 抛物线有一个焦点,对抛物线上任一点$P$到焦点$F$与到 15 | 一定直线$\ell$的距离$d$相等,即 16 | \[\overline{PF}:d=1\] 17 | 18 | 这一章,我们将要根据上述圆锥曲线的几何特性来定义 19 | 椭圆、双曲线和抛物线,建立它们在平面直角坐标系中的标 20 | 准方程,并利用标准方程进一步研究圆锥曲线其它的几何特 21 | 性. 22 | 23 | \section{圆锥曲线的标准方程及其性质} 24 | \subsection{椭圆的标准方程和形状} 25 | \begin{blk} 26 | {定义} 平面内与两定点的距离之和等于常数(这常数必 27 | 须大于两定点间的距离)的点的轨迹叫做椭圆.这两定点叫 28 | 做椭圆的\textbf{焦点}.两焦点间的距离叫做\textbf{焦距}. 29 | \end{blk} 30 | 31 | 下面,我们根据椭圆的定义来建立椭圆的方程. 32 | 33 | 设$F_1$、$F_2$是椭圆的两个焦点,取射线$F_1F_2$作为$X$轴 34 | 的正半轴,$\overline{F_1F_2}$的垂直平分线作为$Y$轴(图6.1).设 35 | 焦距$\overline{F_1F_2}=2c\; (c>0)$,则 36 | \[F_1(-c,0),\qquad F_2(c,0)\] 37 | 38 | \begin{figure}[htp] 39 | \centering 40 | \begin{tikzpicture}[>=latex] 41 | \draw[->](-2.5,0)--(2.5,0)node[right]{$X$}; 42 | \draw[->](0,-2)--(0,2)node[right]{$Y$}; 43 | \draw[thick](0,0) ellipse [x radius=2, y radius=1.25]; 44 | \tkzDefPoints{-1.56/0/F_1, 1.56/0/F_2, 1/1.08/P} 45 | \tkzDrawSegments[thick](F_1,P P,F_2) 46 | \tkzLabelPoints[below](F_1,F_2) 47 | \tkzLabelPoints[above](P) 48 | \node at (0,0)[below left]{$O$}; 49 | \end{tikzpicture} 50 | \caption{} 51 | \end{figure} 52 | 53 | 54 | 设$P(x,y)$是椭圆上的任一点, 55 | 它到$F_1$、$F_2$的距离之和等于常数 56 | $2a\; (a>0)$, 则 57 | \[\overline{PF_1}+\overline{PF_2}=2a\] 58 | 由求两点的距离公式得 59 | \[\sqrt{(x+c)^2+y^2}+\sqrt{(x-c)^2+y^2}=2a\] 60 | 去根号,整理得 61 | \begin{equation} 62 | (a^2-c^2)x^2+a^2y^2=a^2(a^2-c^2) 63 | \end{equation} 64 | 因$\overline{PF_1}+\overline{PF_2}>\overline{F_1F_2}$, 所以$a>c,\; a^2-c^2>0$, 65 | 设$a^2-c^2=b^2\; (b>0)$, 代入(6.1)式得 66 | \[b^2x^2+a^2y^2=a^2b^2\] 67 | 两边同除$a^2b^2$得 68 | \begin{equation} 69 | \boxed{\frac{x^2}{a^2}+\frac{y^2}{b^2}=1} 70 | \end{equation} 71 | 这就是说,椭圆上任一点的坐标都满足方程(6.2); 反过来, 72 | 设$P(x_1,y_1)$的坐标满足方程(6.2), 则 73 | \[\frac{x_1^2}{a^2}+\frac{y_1^2}{b^2}=1\] 74 | \[y^2_1=b^2\left(1-\frac{x^2_1}{a^2}\right)=(a^2-c^2)\left(1-\frac{x^2_1}{a^2}\right)\] 75 | 于是 76 | \[\begin{split} 77 | \overline{PF_1}&=\sqrt{(x_1+c)^2+y^2_1}\\ 78 | &=\sqrt{(x_1+c)^2+(a^2-c^2)\left(1-\frac{x^2_1}{a^2}\right)}\\ 79 | &=\sqrt{a^2+2cx_1+\frac{c^2}{a^2}x^2_1}\\ 80 | &=\left|a+\frac{c}{a}x_1\right| 81 | \end{split}\] 82 | 由$\frac{x_1^2}{a^2}+\frac{y_1^2}{b^2}=1$,可推知 83 | $\frac{x_1^2}{a^2}\le 1$,$|x_1|\le a$. 84 | 85 | 又因$c0$,因此: 87 | \begin{equation} 88 | \overline{PF_1}=a+\frac{c}{a}x_1 89 | \end{equation} 90 | 同理可证, 91 | \[\overline{PF_2}=a-\frac{c}{a}x_1\] 92 | 所以:$\overline{PF_1}+\overline{PF_2}=2a$ 93 | 94 | 这就是说,坐标满足方程(6.2)的点$P$也一定在椭圆上,由以 95 | 上证明,所以方程(6.2)是所求的椭圆方程,并把方程(6.2) 96 | 叫做\textbf{椭圆的标准方程}. 97 | 98 | 下面,用椭圆的标准方程来研究椭圆的几何形状. 99 | 100 | 首先,由于方程(6.2)中只含有$x,y$的平方,故把一 101 | 个坐标变号,对于方程没有影响,这就表明:如果$M(x,y)$ 102 | 在椭圆上,那么,$M_1(x,-y)$, $M_2(-x,-y)$, $M_3(-x, 103 | y)$各点也都在椭圆上,所以椭圆既是以$X$轴或$Y$轴为对称轴的 104 | 轴对称图形,又是以坐标原点为对称中心的中心对称图形,对 105 | 称中心又叫做椭圆的中心. 106 | 107 | 其次,由方程(6.2)得 108 | \[\frac{x^2}{a^2}\le 1,\qquad \frac{y^2}{b^2}\le 1\] 109 | \[-a\le x\le a,\qquad -b\le y\le b\] 110 | 这两个不等式表明椭圆全部包含在如图6.2所示的长方形 111 | 内. 112 | 113 | 最后,我们来讨论椭圆在第 114 | I象限内的性态.由(6.2)得 115 | \[y=\pm\frac{b}{a}\sqrt{a^2-x^2}\] 116 | 在第I象限,椭圆方程可写为 117 | \[y=\frac{b}{a}\sqrt{a^2-x^2},\qquad 0\le x\le a\] 118 | 当$x=0$时,$y=b$, 当$x$递增时, 119 | $y$递减,当$x=a$时,$y=0$, 120 | 因此椭圆在第I象限内的轨迹大致是$B_2A_2$这部分曲线,由 121 | 对称性可画出整个椭圆的图象(图6.2). 122 | 123 | \begin{figure}[htp] 124 | \centering 125 | \begin{tikzpicture}[>=latex] 126 | \draw[->](-2.5,0)--(2.75,0)node[right]{$X$}; 127 | \draw[->](0,-2)--(0,2.5)node[right]{$Y$}; 128 | \draw[thick](0,0) ellipse [x radius=2, y radius=1.25]; 129 | \node at (0,0)[below left]{$O$}; 130 | \draw(-2,-1.25) rectangle (2,1.25); 131 | \node at (-2,0)[below left]{$A_1$}; 132 | \node at (2,0)[below right]{$A_2$}; 133 | \node at (0,1.25)[above right]{$B_2$}; 134 | \node at (0,-1.25)[below right]{$B_1$}; 135 | \tkzDrawPoint(1.5,0)\tkzDrawPoint(-1.5,0) 136 | \end{tikzpicture} 137 | \caption{} 138 | \end{figure} 139 | 140 | 当$y=0$, $x=\pm a$, 点$A_1(-a,0)$, $A_2(+a,0)$ 141 | 是$X$轴上距$Y$轴最远的两个点,当$x=0$, $y=\pm b$, 点 142 | $B_1(0,-b)$, $B_2(0,+b)$是$Y$轴上距$X$轴距离最远的 143 | 两个点,这四点,$A_1$、$A_2$、$B_1$、$B_2$叫做\textbf{椭圆的顶点}. 144 | $\overline{A_1A_2},\overline{B_1B_2}$分别叫做椭圆的长和轴短轴.$\overline{A_1A_2}=2a$, 145 | $\overline{B_1B_2}=2b$, $a$和$b$分别叫做椭圆的\textbf{长半轴长和短半轴长}. 146 | 长轴和短轴的交点叫做\textbf{椭圆的中心}. 147 | 148 | 如果$a=b$, 那么方程(6.2)化为 149 | \[x^2+y^2=a^2\] 150 | 这时椭圆成为圆,$c=\sqrt{a^2-b^2}=0$, 即椭圆的两个焦点重 151 | 合于圆心,因此可以说\textbf{圆是椭圆的特殊情形}. 152 | 153 | 由以上讨论可以看出,椭圆的形状依赖于$a$和$b$, 数量 154 | $c=\sqrt{a^2-b^2}$可表示出椭圆离开圆的偏差.由$c^2=a^2-b^2$ 155 | 可得 156 | \[\frac{c}{a}=\sqrt{1-\left(\frac{b}{a}\right)^2},\qquad \frac{b}{a}=\sqrt{1-\left(\frac{c}{a}\right)^2}\] 157 | 比值 158 | \[e=\frac{c}{a}=\frac{\sqrt{a^2-b^2}}{a}\] 159 | 叫做\textbf{椭圆的离心率},用它可同样来表出椭圆的形状.由$c< 160 | a$, 可知$e<1$, 当离心率愈来愈大时,也就是愈来愈接近 161 | 1时,$1-e^2$就越小,椭圆的形状就愈扁平;反之,就愈接 162 | 近于圆,当$e=0$时,$a=b$椭圆就成为圆了. 163 | 164 | 如果椭圆的中心在原点,焦点在$Y$轴上,那么长轴也- 165 | 定在$Y$轴上,这时两个焦点$F_1,F_2$的坐标分别是$(0,-c)$, 166 | $(0,c)$ (图6.3), 求得圆的标准方程是 167 | \begin{equation} 168 | \boxed{\frac{x^2}{b^2}+\frac{y^2}{a^2}=1}\qquad a\ge b>0 169 | \end{equation} 170 | 把方程(6.2)的变量$x$和$y$互换就可得到方程(6.6). 171 | 172 | \begin{figure}[htp]\centering 173 | \centering 174 | \begin{tikzpicture}[>=latex, scale=1] 175 | \draw[->](-1.5,0)--(1.75,0)node[right]{$X$}; 176 | \draw[->](0,-2)--(0,2.5)node[right]{$Y$}; 177 | \draw[thick](0,0) ellipse [x radius=.7, y radius=1.5]; 178 | \tkzDefPoints{0/-1.2/F_1, 0/1.2/F_2} 179 | \tkzLabelPoints[right](F_1,F_2) 180 | \tkzDrawPoints(F_1,F_2) 181 | \node at (0,0)[below left]{$O$}; 182 | \node at (0,1.5)[above right]{$B(0,a)$}; 183 | \node at (0.7,0)[below right]{$A(b,0)$}; 184 | \end{tikzpicture} 185 | \caption{} 186 | \end{figure} 187 | 188 | 189 | \begin{example} 190 | 已知椭圆的长轴长是10, 焦距是8, 求椭圆的标准方程. 191 | \end{example} 192 | 193 | \begin{solution} 194 | 由已知条件得$2a=10,\quad 2c=8$,所以: 195 | \[a=5,\qquad c=4,\qquad b^2=a^2-c^2=5^2-4^2=9\] 196 | 因此所求椭圆的标准方程为 197 | \[\frac{x^2}{25}+\frac{y^2}{9}=1\] 198 | \end{solution} 199 | 200 | \begin{example} 201 | 求椭圆$4x^2+9y^2=36$的长轴、短轴长、离心率、 202 | 焦点和顶点的坐标,并用描点法画出它的图形. 203 | \end{example} 204 | 205 | \begin{solution} 206 | 已知方程可化为 207 | \[\frac{x^2}{9}+\frac{y^2}{4}=1\] 208 | 这是长轴在$X$轴上,中心在坐标原点的椭圆标准方程. 209 | 因此$a=3$, $b=2$, $c=\sqrt{3^2-2^2}=\sqrt{5}$, 顶点$A'(-3, 210 | 0)$, $A(3,0)$, $B'(0,-2)$, $B(0,2)$. 焦点 211 | $F_1(-\sqrt{5},0)$, $F_2(\sqrt{5},0)$. 离心率$e=\frac{c}{a}=\frac{\sqrt{5}}{3}$. 212 | 213 | 在第I象限已知椭圆方程可写为 214 | \[y=\frac{2}{3}\sqrt{9-x^2},\qquad 0\le x\le 3\] 215 | 算出一些满足所求椭圆方程的点的坐标$(x,y)$: 216 | \begin{center} 217 | \begin{tabular}{cccccccc} 218 | \hline 219 | $x$&0&0.5&1&1.5&2&2.5&3\\ 220 | \hline 221 | $y$&2&1.97&1.89&1.73&1.49&1.11&0\\ 222 | \hline 223 | \end{tabular} 224 | \end{center} 225 | 描点画出椭圆在第I象限的图 226 | 象,然后根据椭圆的对称性就可 227 | 画出整个椭圆的图象(图6.4). 228 | \end{solution} 229 | 230 | \begin{figure}[htp]\centering 231 | \begin{minipage}[t]{0.48\textwidth} 232 | \centering 233 | \begin{tikzpicture}[>=latex, scale=.7] 234 | \draw[->](-4,0)--(4,0)node[right]{$X$}; 235 | \draw[->](0,-3)--(0,3)node[right]{$Y$}; 236 | \draw[thick](0,0)node[below left]{$O$} ellipse [x radius=3, y radius=2]; 237 | \node at (-3,0)[below left]{$A'$}; 238 | \node at (3,0)[below right]{$A$}; 239 | \node at (0,2)[above right]{$B$}; 240 | \node at (0,-2)[below right]{$B'$}; 241 | \tkzDefPoints{0/2/A, .5/1.97/B, 1/1.89/C, 1.5/1.73/D, 2/1.49/E, 2.5/1.11/F, 3/0/G} 242 | \tkzDrawPoints(A,B,C,D,E,F,G) 243 | 244 | \tkzDefPoints{0/-2/A, .5/-1.97/B, 1/-1.89/C, 1.5/-1.73/D, 2/-1.49/E, 2.5/-1.11/F, 3/0/G} 245 | \tkzDrawPoints(A,B,C,D,E,F,G) 246 | 247 | \tkzDefPoints{0/2/A, -.5/1.97/B, -1/1.89/C, -1.5/1.73/D, -2/1.49/E, -2.5/1.11/F, -3/0/G} 248 | \tkzDrawPoints(A,B,C,D,E,F,G) 249 | 250 | \tkzDefPoints{0/-2/A, -.5/-1.97/B, -1/-1.89/C, -1.5/-1.73/D, -2/-1.49/E, -2.5/-1.11/F, -3/0/G} 251 | \tkzDrawPoints(A,B,C,D,E,F,G) 252 | 253 | \foreach \x in {-2.5,-2,...,2.5} 254 | { 255 | \draw(\x,0)--(\x,.1); 256 | } 257 | \foreach \x in {-1.5,-1,...,1.5} 258 | { 259 | \draw(0,\x)--(.1,\x); 260 | } 261 | \end{tikzpicture} 262 | \caption{} 263 | \end{minipage} 264 | \begin{minipage}[t]{0.48\textwidth} 265 | \centering 266 | \begin{tikzpicture}[>=latex, scale=.7] 267 | \draw[->](-4,0)--(4,0)node[right]{$X$}; 268 | \draw[->](0,-3)--(0,3)node[right]{$Y$}; 269 | \draw(.5,0)node[above]{$F_2$} circle(1.5); 270 | \draw(-.5,0)node[above]{$F_1$} ellipse[x radius= 3, y radius= 2.7]; 271 | \tkzDrawPoint(.5,0)\tkzDrawPoint(-.5,0) 272 | \node at (0,0) [below left]{$O$}; 273 | \node at (2.5,0) [below right]{$A_2$}; 274 | \node at (-3.5,0) [below left]{$A_1$}; 275 | \end{tikzpicture} 276 | \caption{} 277 | \end{minipage} 278 | \end{figure} 279 | 280 | 281 | \begin{example} 282 | 我国第一颗人造地球 283 | 卫星的运行轨道是以地球中心为 284 | 一焦点的椭圆,卫星的近地点与 285 | 地球表面距离为439公里;远地点 286 | 与地球表面距离为2384公里,已知地球半径约为6371公里, 287 | 试求卫星轨道的近似方程及其离心率. 288 | \end{example} 289 | 290 | 291 | \begin{solution} 292 | 设地球中心$F_2$在$X$轴上(图6.5),所求方程为 293 | \[\frac{x^2}{a^2}+\frac{y^2}{b^2}=1\] 294 | 依题意 295 | \[\begin{split} 296 | \overline{A_1F_2}&=a+c=6371+2284=8755\\ 297 | \overline{A_2F_2}&=a-c=6371+439=6810 298 | \end{split}\] 299 | 由以上两式联立求解得 300 | \[a=7782.5,\qquad c=972.5,\qquad b=\sqrt{a^2-c^2}=7721.5\] 301 | 所以,所求卫星轨道的近似方程为 302 | \[\frac{x^2}{(7782.5)^2}+\frac{y^2}{(7721.5)^2}=1\] 303 | 其离心率 304 | $e=\frac{c}{a}\approx 0.125$ 305 | \end{solution} 306 | 307 | \begin{ex} 308 | \begin{enumerate} 309 | \item 已知椭圆的长轴长是6, 短轴长是2, 焦点在$X$轴上, 310 | 求这椭圆的标准方程并画出这椭圆的草图. 311 | \item 在第1题中,若焦点在$Y$轴,椭圆的标准方程为何? 312 | \item 已知椭圆的一个焦点是$F_1(-3,0)$与$X$轴一个交点 313 | $A(4,0)$, 求此椭圆的方程. 314 | \item 求以下椭圆的长轴长,短轴长、焦点的坐标及其离心率. 315 | \begin{multicols}{2} 316 | \begin{enumerate} 317 | \item $\frac{x^2}{100}+\frac{y^2}{36}=1$ 318 | \item $25x^2+9y^2=100$ 319 | \item $\frac{x^2}{16}+\frac{y^2}{64}=1$ 320 | \item $49x^2+9y^2=2500$ 321 | \end{enumerate} 322 | \end{multicols} 323 | 324 | \item 已知椭圆中心在原点,一焦点是$F(3,0)$, 椭圆与$X$ 325 | 轴相交于$A$、$A'$两点,$\overline{AF}=2$, $\overline{A'F}=8$, 求此椭圆的方程. 326 | \item 已知地球的轨道是一个椭圆,太阳在它的一个焦点上, 327 | 长轴长约30亿公里,离心率$e=1/60$, 328 | 求地球的轨道方程,地球的轨道中心与太阳的距离,以及近日点,远日 329 | 点到太阳的距离. 330 | \item 试求平分圆$x^2+y^2=25$上各点的纵坐标,而横坐标不 331 | 变的点的轨迹方程. 332 | \item 试求把圆$x^2+y^2=100$上各纵坐标分为2:3, 而横 333 | 坐标不变的点的轨迹方程. 334 | \item 一动点与直线$x=8$的距离是它与点$(2,0)$的距离的 335 | 2倍,求这动点的轨迹方程. 336 | \item 一定长为$a$的线段,两端在互相垂直的二直线上移动, 337 | 试求此线段上任意一点的轨迹方程. 338 | \item 设一三角形的一边的两个端点为$(0,6)$, $(0,-6)$,其它两 339 | 边斜率的乘积是$-\frac{4}{9}$, 340 | 试求另一顶点的轨迹. 341 | \item 已知$A>0$, $B>0$, 且$A=latex, yscale=.7] 360 | \draw[->](-2.5,0)--(2.5,0)node[right]{$X$}; 361 | \draw[->](0,-2.5)--(0,2.5)node[right]{$Y$}; 362 | \draw[domain=-2:2, samples=100, very thick]plot({sqrt(1+\x*\x)}, \x); 363 | \draw[domain=-2:2, samples=100, very thick]plot({-sqrt(1+\x*\x)}, \x); 364 | \tkzDefPoints{-1.414/0/F_1, 1.414/0/F_2, 2/1.732/P} 365 | \tkzDrawSegments[thick](F_1,P F_2,P) 366 | \tkzLabelPoints[below](F_1, F_2) 367 | \tkzLabelPoints[above](P) 368 | \node at (0,0)[below left]{$O$}; 369 | \end{tikzpicture} 370 | \caption{} 371 | \end{figure} 372 | 373 | 设$F_1$、$F_2$是双曲线的两个焦点,取射线$F_1F_2$的方向作 374 | 为$X$轴的正方向,$\overline{F_1F_2}$的垂直平分线作为$Y$轴(图6.6). 375 | 若$\overline{F_1F_2}=2c$, 则两焦点的坐标分别为$F_1(-c,0)$, $F_2 376 | (c,0)$. 377 | 378 | 再设$P(x,y)$是双曲线上任一点,则由双曲线的定义 379 | 有 380 | \[|\overline{PF_1}-\overline{PF_2}|=2a\] 381 | 因$\overline{PF_1}=\sqrt{(x+c)^2+y^2}$, $\overline{PF_2}= 382 | \sqrt{(x-c)^2+y^2}$, 383 | 代入上式,得方程 384 | \[\sqrt{(x+c)^2+y^2}-\sqrt{(x-c)^2+y^2}=\pm 2a\] 385 | 去根号,整理得 386 | \begin{equation*} 387 | (a^2-c^2)x^2+a^2y^2=a^2(a^2-c^2) 388 | \end{equation*} 389 | 这个式子和上节得到的椭圆方程(6.1)在外形上完全一样, 390 | 这里,由双曲线的定义$2c>2a$, 即 391 | \[c>a\] 392 | 所以$a^2-c^2<0$, 故设$a^2-c^2=-b^2\; (b>0)$, 代入(6.1) 393 | 式得 394 | \[-b^2x^2+a^2y^2=-a^2b^2\] 395 | 两边同除$-a^2b^2$得 396 | \begin{equation} 397 | \boxed{\frac{x^2}{a^2}-\frac{y^2}{b^2}=1} 398 | \end{equation} 399 | 400 | 由上述推导过程说明,凡在双曲线上的点,它的坐标一 401 | 定满足方程(6.5); 反过来, 402 | 设$P_1(x_1,y_1)$的坐标满足方程(6.5), 则 403 | \[\begin{split} 404 | \frac{x^2_1}{a^2}-\frac{y^2_1}{b^2}&=1\\ 405 | \overline{P_1F_1}&=\sqrt{(x_1+c)^2+y_1^2} 406 | \end{split}\] 407 | 但 408 | \[y^2_1=b^2\left(\frac{x_1^2}{a^2}-1\right)=(c^2-a^2)\left(\frac{x_1^2}{a^2}-1\right)\] 409 | 代入上式,化简可得 410 | \begin{equation} 411 | \overline{P_1F_1}=\left|\frac{c}{a}x_1+a\right| 412 | \end{equation} 413 | 同理可求 414 | \begin{equation} 415 | \overline{P_1F_2}=\left|\frac{c}{a}x_1-a\right| 416 | \end{equation} 417 | 418 | 因为$c>a$, $|x|\ge a$, 所以$\left|\frac{c}{a}x_1\right|>a$, 419 | $\frac{c}{a}x_1+a$及$\frac{c}{a}x_1-a$与$\frac{c}{a}x_1$同号. 420 | 421 | 当$x_1>0$时 422 | \[\overline{P_1F_1}=\frac{c}{a}x_1+a,\qquad \overline{P_1F_2}=\frac{c}{a}x_1-a\] 423 | 因此, 424 | \[\overline{P_1F_1}-\overline{P_1F_2}=2a\] 425 | 426 | 当$x_1<0$时, 427 | \[\overline{P_1F_1}=-\left(\frac{c}{a}x_1+a\right),\qquad \overline{P_1F_2}=-\left(\frac{c}{a}x_1-a\right)\] 428 | 因此, 429 | \[\overline{P_1F_1}-\overline{P_1F_2}=-2a\] 430 | 431 | 这就证明了,凡坐标适合方程(6.5)的点都在双曲线 432 | 上,由以上证明,所以方程(6.5)是所求的双曲线的方程, 433 | 并且我们把方程(6.5)叫做\textbf{双曲线的标准方程},它所表示的 434 | 双曲线的焦点在$X$轴上,焦点是$F_1(-c,0)$、$F_2(c,0)$. 435 | 这里$c^2=a^2+b^2$. 436 | 437 | 如果取$F_1$、$F_2$的连线作为$Y$轴,取$F_1F_2$的垂直平分 438 | 线作为$X$轴,在这一坐标系中,仿上面的方法可得双曲线的 439 | 方程为 440 | \begin{equation} 441 | \boxed{\frac{y^2}{a^2}-\frac{x^2}{b^2}=1} 442 | \end{equation} 443 | 只要将方程(6.5)中的$x$与$y$对调就可得到方程(6.8), 这一 444 | 方程也叫做双曲线的标准方程,它表示双曲线的焦点在$Y$轴 445 | 上,焦点是$F_1(0,-c)$, $F_2(0,c)$, $c^2=a^2+b^2$. 446 | 447 | 下面我们用双曲线的标准方程研究它的几何形状. 448 | 449 | 首先,与椭圆方程一样,方程(6.5)中只含有$x$、$y$的 450 | 平方,故把其中一个坐标变号,对于方程没有影响,这就表 451 | 明,如果点$M(x,y)$在双曲线上,那么$M_1(x,-y)$、$M_2(-x, 452 | -y)$、$M_3(-x,y)$等也都在双曲线上,所以,双曲线是以 453 | $X$轴或$Y$轴为对称轴的轴对称图形,又是以坐标原点为对称 454 | 中心的中心对称图形,对称中心又叫做双曲线的中心,其 455 | 次,由方程(6.5)得 456 | \[\frac{x^2}{a^2}\ge 1,\qquad x^2\ge a^2\] 457 | 则$x\ge a$或$x\le -a$.这说明双曲线在两条直线$x=a$, $x=-a$所夹平面区域的外侧,最后我们讨论双曲线在第I象限内的性态,在第I象 458 | 限,方程(6.5)可写为 459 | \[y=\frac{b}{a}\sqrt{x^2-a^2}\qquad (x\ge a)\] 460 | 461 | \begin{figure}[htp] 462 | \centering 463 | \begin{tikzpicture}[>=latex, yscale=.7] 464 | \draw[->](-3.5,0)--(3.5,0)node[right]{$X$}; 465 | \draw[->](0,-3.5)--(0,3.5)node[right]{$Y$}; 466 | \draw[domain=-2.8:2.8, samples=100, very thick]plot({sqrt(1+\x*\x)}, \x); 467 | \draw[domain=-2.8:2.8, samples=100, very thick]plot({-sqrt(1+\x*\x)}, \x); 468 | \draw(-3,3)--(3,-3); 469 | \draw(3,3)--(-3,-3); 470 | \draw(-1,-1) rectangle (1,1); 471 | \node at (0,0)[below left]{$O$}; 472 | \node at (-1,0)[below left]{$A_1$}; 473 | \node at (1,0)[below right]{$A_2$}; 474 | \node at (0,1)[above right]{$B_2$}; 475 | \node at (0,-1)[below right]{$B_1$}; 476 | \end{tikzpicture} 477 | \caption{} 478 | \end{figure} 479 | 480 | 当$x=a$时,$y=0$, 当$x$由$a$递增且趋向$\infty$, $y$也由0递 481 | 增趋向,方程的轨迹趋向无穷远(图6.7),但由于 482 | \[y=\frac{b}{a}\sqrt{x^2-a^2}=\frac{b}{a}x\sqrt{1-\left(\frac{a}{x}\right)^2}<\frac{b}{a}x\] 483 | 即 484 | \[\frac{b}{a}x-\frac{b}{a}\sqrt{x^2-a^2}>0\] 485 | 所以,当$x$由$a$趋向$\infty$时,相应的$y$值愈来愈接近$\frac{b}{a}x$, 而 486 | 又不会大于$\frac{b}{a}x$, 487 | 这说明,双曲线在第I象限的部分永远在 488 | 射线$y=\frac{b}{a}x\; (x\ge 0)$的下方并且逐渐接近于射线$y=\frac{b}{a}x\; (x\ge 0)$. 由对称性,可推知双曲线在其它象限的性态 489 | (图6.7). 490 | 491 | 直线$y=\frac{b}{a}x$和$y=-\frac{b}{a}x$叫做\textbf{双曲线的渐近线}. 492 | 493 | 令$y=0$, $x=\pm a$, 点$A_1(-a,0)$, $A_2(a,0)$叫 494 | 做双曲线的顶点,$\overline{A_1A_2}$叫做双曲线的\textbf{实轴},它的长等于 495 | $2a$, $a$叫做双曲线的实半轴长. 496 | 497 | 令$x=0$, $y=\pm b\sqrt{-1}$, 这说明双曲线和$Y$轴没有 498 | 交点.在$Y$轴上作$B_1(0,-b)$, $B_2(0,b)$, $\overline{B_1B_2}$叫做双 499 | 曲线的虚轴,它的长等于$2b$, $b$叫做双曲线虚半轴长(图6 500 | .7).实轴和虚轴等长的双曲线叫做\textbf{等轴双曲线}. 501 | 502 | 以已知双曲线的虚轴为实轴,实轴为虚轴所得到的双曲 503 | 线叫做原双曲线的\textbf{共轭双曲线}.由这个定义可知,双曲线 504 | $\frac{x^2}{a^2}-\frac{y^2}{b^2}=1$与$\frac{x^2}{a^2}-\frac{y^2}{b^2}=-1$ 505 | 是互为共轭双曲线(图6.8). 506 | 507 | 作为练习,请同学证明:双曲线和它的共轭双曲线有相 508 | 词的渐近线. 509 | 510 | \begin{figure}[htp] 511 | \centering 512 | \begin{tikzpicture}[>=latex, scale=.7] 513 | \draw[->](-3.5,0)--(3.5,0)node[right]{$X$}; 514 | \draw[->](0,-3.5)--(0,3.5)node[right]{$Y$}; 515 | \node at (0,0)[below left]{$O$}; 516 | \draw[domain=-2.8:2.8, samples=100, very thick]plot({sqrt(1+\x*\x)}, \x); 517 | \draw[domain=-2.8:2.8, samples=100, very thick]plot({-sqrt(1+\x*\x)}, \x); 518 | \draw(-3,3)--(3,-3); 519 | \draw(3,3)--(-3,-3); 520 | \draw[domain=-2.8:2.8, samples=100, dashed]plot(\x, {sqrt(1+\x*\x)}); 521 | \draw[domain=-2.8:2.8, samples=100, dashed]plot( \x, {-sqrt(1+\x*\x)}); 522 | 523 | 524 | \end{tikzpicture} 525 | \caption{} 526 | \end{figure} 527 | 528 | 和定义椭圆的离心率$e$一样,对于双曲线,比值 529 | \[e=\frac{c}{a}=\frac{\sqrt{a^2+b^2}}{a}=\sqrt{1+\left(\frac{b}{a}\right)^2}\] 530 | 也叫做双曲线的离心率,因为$c>a$, 所以双曲线的离心率 531 | $e>1$, 容易看出,$\frac{b}{a}$越大,$e$越大;反之,$e$越大, 532 | $\frac{b}{a}$也越大,渐近线$y=\pm \frac{b}{a}x$的斜率的绝对值也越大,这时双曲 533 | 线的开口增大越快. 534 | 535 | 536 | \begin{example} 537 | 设双曲线两焦点间的距离等于8, 顶点间的距离 538 | 等于6, 实轴在$X$轴上,求双曲线的标准方程,离心率,渐 539 | 近线方程并画草图. 540 | \end{example} 541 | 542 | \begin{solution} 543 | 依题意$2c=8$, $2a=6$, 所以$c=4$, $a=3$, 544 | \[ b^2=c^2-a^2=16-9=7,\qquad b=\sqrt{7}\] 545 | 所求双曲线方程为 546 | \[\frac{x^2}{9}-\frac{y^2}{7}=1\] 547 | 离心率$e=\frac{c}{a}=\frac{4}{3}$,渐近线方程为 548 | \[y=\frac{\sqrt{7}}{3}x,\qquad y=-\frac{\sqrt{7}}{3}x\] 549 | 550 | 作图:如图6.9所示 551 | \begin{enumerate} 552 | \item 在$X$轴上作$A_1(-3, 553 | 0)$, $A_2(3,0)$, 在$Y$轴上作$B_1(0,-\sqrt{7})$、$B_2(0, 554 | \sqrt{7})$, 过$A_1$、$A_2$、$B_1$、$B_2$作矩形$ABCD$, 直线$OA$, 555 | $OB$为双曲线的渐近线. 556 | \item 算出一些满足所求双曲线方程的点的坐标: 557 | \begin{center} 558 | \begin{tabular}{ccccc} 559 | \hline 560 | $x$ & $\pm 4$ & $\pm 5$ & $\pm 6$ & $\pm 7$\\ 561 | \hline 562 | $y$ & $\pm 2.3$ & $\pm 3.5$ & $\pm 4.6$ & $\pm 5.6$\\ 563 | \hline 564 | \end{tabular} 565 | \end{center} 566 | 描点连线,使曲线与渐近线逐渐接近,就可得到双曲线的草图. 567 | \end{enumerate} 568 | \end{solution} 569 | 570 | 571 | \begin{figure}[htp] 572 | \centering 573 | \begin{tikzpicture}[>=latex, scale=.4] 574 | \draw[->](-8.5,0)--(8.5,0)node[right]{$X$}; 575 | \draw[->](0,-8.5)--(0,8.5)node[right]{$Y$}; 576 | \node at (0,0)[below left]{$O$}; 577 | 578 | \draw[domain=-7:7, samples=100, very thick]plot({sqrt(9+9/7*\x*\x)}, \x); 579 | \draw[domain=-7:7, samples=100, very thick]plot({-sqrt(9+9/7*\x*\x)}, \x); 580 | \draw(-3*3,2.65*3)--(3*3,-2.65*3); 581 | \draw(3*3,2.65*3)--(-3*3,-2.65*3); 582 | \draw(-3,-2.65) rectangle(3,2.65); 583 | 584 | \tkzDefPoints{-3/-2.65/C, 3/-2.65/D, -3/2.65/B, 3/2.65/A} 585 | \tkzDefPoints{-3/0/A_1, 3/0/A_2, 0/2.65/B_2, 0/-2.65/B_1} 586 | \tkzLabelPoints[above](A,B) 587 | \tkzLabelPoints[below](C,D) 588 | \tkzLabelPoints[above left](B_2) 589 | \tkzLabelPoints[below left](B_1) 590 | \tkzLabelPoints[below left](A_2) 591 | \tkzLabelPoints[below right](A_1) 592 | 593 | \tkzDefPoints{4/0/F_2, -4/0/F_1} 594 | \tkzLabelPoints[below](F_1,F_2) 595 | \tkzDrawPoints(F_1,F_2,A_1,A_2,B_1,B_2) 596 | 597 | \tkzDefPoints{4/2.3/A1, 5/3.5/A2, 6/4.6/A3, 7/5.6/A4} 598 | \tkzDrawPoints(A1,A2,A3,A4) 599 | \tkzDefPoints{4/-2.3/A1, 5/-3.5/A2, 6/-4.6/A3, 7/-5.6/A4} 600 | \tkzDrawPoints(A1,A2,A3,A4) 601 | \tkzDefPoints{-4/2.3/A1, -5/3.5/A2, -6/4.6/A3, -7/5.6/A4} 602 | \tkzDrawPoints(A1,A2,A3,A4) 603 | \tkzDefPoints{-4/-2.3/A1, -5/-3.5/A2, -6/-4.6/A3, -7/-5.6/A4} 604 | \tkzDrawPoints(A1,A2,A3,A4) 605 | 606 | \foreach \x in {-7,-6,...,7} 607 | { 608 | \draw(\x,0)--(\x,.2); 609 | \draw(0,\x)--(.2,\x); 610 | } 611 | \end{tikzpicture} 612 | \caption{} 613 | \end{figure} 614 | 615 | 616 | 617 | \begin{example} 618 | 证明:双曲线上任一点到两条渐近线的距离的乘积等于常数$\frac{a^2b^2}{a^2+b^2}$ 619 | \end{example} 620 | 621 | \begin{proof} 622 | 已知双曲线$\frac{x^2}{a^2}-\frac{y^2}{b^2}=1$,它的两条渐近线方程为 623 | \[\ell_1:\; bx+ay=0,\qquad \ell_2:\; bx-ay=0\] 624 | 设$P(x_1,y_1)$为双曲线上任一点,$P$到$\ell_1$的距离记为$d_1$,$P$ 625 | 到$\ell_2$的距离记为$d_2$,则: 626 | \[d_1=\frac{|bx_1+ay_1|}{\sqrt{a^2+b^2}},\qquad d_2=\frac{|bx_1-ay_1|}{\sqrt{a^2+b^2}}\] 627 | \[d_1\cdot d_2=\frac{|bx_1+ay_1|}{\sqrt{a^2+b^2}}\cdot \frac{|bx_1-ay_1|}{\sqrt{a^2+b^2}}=\frac{|b^2x^2_1-a^2y^2_1|}{{a^2+b^2}}\] 628 | 但 629 | \[|b^2x^2_1-a^2y^2_1|=a^2b^2\] 630 | 所以 631 | \[d_1\cdot d_2=\frac{a^2b^2}{a^2+b^2}\] 632 | \end{proof} 633 | 634 | \begin{ex} 635 | \begin{enumerate} 636 | \item 求适合下列条件的双曲线的标准方程. 637 | \begin{enumerate} 638 | \item 实轴长为12, 虚轴长为$2\sqrt{13}$, 639 | 焦点在$X$轴上; 640 | \item 实轴长为5, 虚轴长为3, 焦点在$Y$轴上; 641 | \item 焦距是10, 两顶点间的距离是8; 642 | 实轴在$X$轴上; 643 | \item 实轴长等于4, 且经过点$A(2,4)$, 实轴在$X$ 644 | 轴上; 645 | \item 实轴长等于4, 且经过点$A(2,4)$, 实轴在$Y$轴 646 | 上. 647 | \end{enumerate} 648 | \item 求下列双曲线的实轴和虚轴长,顶点和焦点的坐标,离 649 | 心率和渐近线方程. 650 | \begin{multicols}{2} 651 | \begin{enumerate} 652 | \item $x^2-y^2=1$ 653 | \item $\frac{x^2}{5}-\frac{y^2}{9}=1$ 654 | \item $4x^2-y^2=16$ 655 | \item $x^2-y^2=-1$ 656 | \item $\frac{x^2}{25}-y^2=-1$ 657 | \end{enumerate} 658 | \end{multicols} 659 | 660 | \item 画出双曲线$\frac{x^2}{16}-\frac{y^2}{9}=1$的草图. 661 | \item 等轴双曲线的一个焦点$F_1(-4,0)$, 求它的标准方 662 | 程. 663 | \item 证明: 664 | \begin{enumerate} 665 | \item 双曲线和它的共轭双曲线有共同的渐近 666 | 线; 667 | \item 双曲线和它的共轭双曲线的四个焦点在同一个圆 668 | 上. 669 | \end{enumerate} 670 | 671 | \item 已知双曲线 672 | $\frac{x^2}{9}-\frac{y^2}{16}=1$, 求出它的共领双曲线方程并 673 | 求共轭双曲线的焦点的坐标,渐近线方程和离心率. 674 | \end{enumerate} 675 | \end{ex} 676 | 677 | \subsection{抛物线的标准方程和形状} 678 | \begin{blk} 679 | {定义} 680 | 平面内与一定点和一条定直线的距离相等的点的轨迹叫做抛物线.定点叫做抛物线的焦点,定直线叫做抛物 681 | 线的准线. 682 | \end{blk} 683 | 684 | 下面,我们根据抛物线的定义来建立抛物线的方程. 685 | 686 | \begin{figure}[htp] 687 | \centering 688 | \begin{tikzpicture}[>=latex, scale=.8] 689 | \draw[->](-1,0)--(4,0)node[right]{$X$}; 690 | \draw[->](0,-3)--(0,3)node[right]{$Y$}; 691 | \draw[thick](-.5,-3)--(-.5,0)node[above left]{$D$}--(-.5,3)node[left]{$\ell$}; 692 | \draw[domain=-2.5:2.5, samples=100, very thick]plot({0.5*\x*\x}, \x); 693 | \draw[thick](-.5,2)--(2,2)node[below right]{$P(x,y)$}--(.5,0)node[below]{$F$}; 694 | \node at (0,0)[below left]{$O$}; 695 | \end{tikzpicture} 696 | \caption{} 697 | \end{figure} 698 | 699 | 设焦点为$F$, 准线为$\ell$ (图 700 | 6.10),过$F$点作$\ell$的垂线与$\ell$相交于$D$点,取射线$DF$的方向 701 | 作为$X$轴的正方向,以$\overline{DF}$的垂直平分线为$Y$轴,设$F$到$\ell$的距离为$p$, 即$\overline{DF}=p$, 则 702 | \[F\left(\frac{p}{2},0\right),\qquad D\left(-\frac{p}{2},0\right)\] 703 | 准线$\ell$的方程为 704 | \[x=-\frac{p}{2}\] 705 | 706 | 设$P(x,y)$是抛物线上任一点,它到焦点$F$的距离等 707 | 于它到$\ell$的距离.即 708 | \[\sqrt{\left(x-\frac{p}{2}\right)^2+y^2}=\left|x+\frac{p}{2}\right|\] 709 | 将上式两边平方,并化简得 710 | \begin{equation} 711 | y^2=2px\qquad (p>0) 712 | \end{equation} 713 | 这就是说,凡是在抛物线上的点,它的坐标都适合这一方 714 | 程;反过来,设$P(x_1,y_1)$的坐标满足方程(6.9), 则 715 | $y_1^2=2px_1$. 点$P(x_1,y_1)$与焦点$F$的距离 716 | \[\begin{split} 717 | PF=\sqrt{\left(x_1-\frac{p}{2}\right)^2+y^2_1}&=\sqrt{\left(x_1-\frac{p}{2}\right)^2+2px_1}\\ 718 | &=\sqrt{\left(x_1+\frac{p}{2}\right)^2}=\left|x_1+\frac{p}{2}\right| 719 | \end{split}\] 720 | 这就是点$P$到准线的距离,这就证明了,凡坐标适合方程(6.9) 721 | 的点,都在抛物线上. 722 | 723 | 由以上证明,所以方程(6.9)是所求的抛物线方程,并 724 | 把方程(6.9)叫做\textbf{抛物线的标准方程}.它表示的抛物线的焦 725 | 点$F$在$X$轴的正半轴上且坐标是$\left(\frac{p}{2},0\right)$ 726 | 准线方程是$x=-\frac{p}{2}$. 727 | 728 | 如果抛物线的焦点和准线分别取 729 | \[\begin{split} 730 | F\left(-\frac{p}{2},0\right),\quad x=\frac{p}{2}&\qquad \text{(图6.11)}\\ 731 | F\left(0,\frac{p}{2}\right),\quad y=-\frac{p}{2}&\qquad \text{(图6.12)}\\ 732 | F\left(-\frac{p}{2},0\right),\quad y=\frac{p}{2}&\qquad \text{(图6.13)}\\ 733 | \end{split}\] 734 | 可分别类似地得到抛物线标准方程为 735 | \[y^2=-2px,\qquad x^2=2py,\qquad x^2=-2py\] 736 | 737 | \begin{figure}[htp]\centering 738 | \begin{minipage}[t]{0.3\textwidth} 739 | \centering 740 | \begin{tikzpicture}[>=latex, scale=.6] 741 | \draw[->](-3.5,0)--(2,0)node[right]{$X$}; 742 | \draw[->](0,-3)--(0,3)node[left]{$Y$}; 743 | \draw[thick](.5,-2.8)--(.5,2.8); 744 | \draw[domain=-2.5:2.5, samples=100, very thick]plot({-0.5*\x*\x}, \x); 745 | \node at (-.5,0) [below]{$F$}; 746 | \tkzDrawPoint (-.5,0) 747 | \node at (0,0)[below right]{$O$}; 748 | \end{tikzpicture} 749 | \caption{} 750 | \end{minipage} 751 | \begin{minipage}[t]{0.3\textwidth} 752 | \centering 753 | \begin{tikzpicture}[>=latex, scale=.6] 754 | \draw[->](-3,0)--(3,0)node[right]{$X$}; 755 | \draw[->](0,-1)--(0,4)node[left]{$Y$}; 756 | \draw[thick](-2.8,-.5)--(2.8,-.5); 757 | \draw[domain=-2.5:2.5, samples=100, very thick]plot(\x,{0.5*\x*\x}); 758 | \node at (0,.5) [left]{$F$}; 759 | \tkzDrawPoint (0,.5) 760 | \node at (0,0)[below right]{$O$}; 761 | \end{tikzpicture} 762 | \caption{} 763 | \end{minipage} 764 | \begin{minipage}[t]{0.3\textwidth} 765 | \centering 766 | \begin{tikzpicture}[>=latex, scale=.6] 767 | \draw[->](-3,0)--(3,0)node[right]{$X$}; 768 | \draw[->](0,-4)--(0,1.5)node[left]{$Y$}; 769 | \draw[thick](-2.8,.5)--(2.8,.5); 770 | \draw[domain=-2.5:2.5, samples=100, very thick]plot(\x,{-0.5*\x*\x}); 771 | \node at (0,-.5) [right]{$F$}; 772 | \tkzDrawPoint (0,-.5) 773 | \node at (0,0)[below left]{$O$}; 774 | \end{tikzpicture} 775 | \caption{} 776 | \end{minipage} 777 | \end{figure} 778 | 779 | 下面我们用抛物线的标准方程来研究抛物线的形状. 780 | 781 | 首先,在方程(6.9)中,含有$y$的平方,故把$-y$代替$y$对 782 | 方程没有影响,这表明曲线是\textbf{以$X$轴为对称轴的轴对称形}. 783 | 我们把这条轴叫做\textbf{抛物线的轴}.轴和抛物线的交点叫做\textbf{抛物 784 | 线的顶点}. 785 | 786 | 其次,由方程(6.9)可知$x\ge 0$, 当$x$增大时,$y$的绝 787 | 对值也跟着增大,因此抛物线在$Y$轴的右方,向上、向下无 788 | 限伸展.设$P(x,y)$是抛物线上任一点,直线$OP$的倾角为 789 | $\alpha$, 则 790 | \[\tan\alpha=\frac{y}{x}=\frac{y^2}{xy}=\frac{2px}{xy}=\frac{2p}{y}\] 791 | 其中$p$是常数,当$y$无限增大时,$\frac{2p}{y}$ 792 | 无限接近于零,这说 793 | 明,抛物线上动点$P(x,y)$无限远离原点时,$P(x,y)$点 794 | 到$X$轴的距离无限增大,而$\Vec{OP}$的方向与$X$轴正向之间的方 795 | 向差却趋于零,这是抛物线与双曲线的重要区别之一. 796 | 797 | \begin{example} 798 | 已知抛物线的焦点$F(2,0)$, 求它的标准方 799 | 程. 800 | \end{example} 801 | 802 | 803 | \begin{solution} 804 | 因为焦点$F(2,0)$在$X$轴上, 805 | $\frac{p}{2}=2$, $p=4$, 806 | 所以抛物线标准方程是 807 | \[y^2=8x\] 808 | \end{solution} 809 | 810 | 811 | 812 | \begin{example} 813 | 已知抛物线的标准方程是$y^2=-4x$, 求它的焦点 814 | 坐标和准线方程. 815 | \end{example} 816 | 817 | \begin{solution} 818 | 因为$p=-2$, 所以焦点的坐标是$(-1,0)$, 819 | 准线方程是$x=1$. 820 | \end{solution} 821 | 822 | \begin{ex} 823 | \begin{enumerate} 824 | \item 根据下列抛物线方程,求出焦点的坐标,准线方程,并 825 | 画出草图. 826 | \begin{multicols}{2} 827 | \begin{enumerate} 828 | \item $y^2=10x$ 829 | \item $y^2=-10x$ 830 | \item $x^2=12y$ 831 | \item $y^2=-8x$ 832 | \end{enumerate} 833 | \end{multicols} 834 | 835 | \item 证明:抛物线$y^2=2px$纵坐标中点的轨迹方程,是 836 | $y^2=\frac{p}{2}x$ 837 | \item 由下列已知条件,求抛物线的标准方程. 838 | \begin{enumerate} 839 | \item 焦点与顶点的距离等于3; 840 | \item 焦点$F$的坐标是$(5,0)$; 841 | \item $X$轴是对称轴,抛物线通过原点和点$(1,-4)$; 842 | \item $Y$轴是对称轴,焦点在$(0,2)$; 843 | \item $Y$轴是对称轴,通过原点和点$(6,-2)$. 844 | \end{enumerate} 845 | 846 | \item 在抛物线$y^2=8x$上,求到原点距离等于20的点的坐 847 | 标. 848 | \item 已知抛物线焦点$F(2,1)$, 准线方程是$x+y+1=0$,由抛物线的定义,求它的方程. 849 | \end{enumerate} 850 | \end{ex} 851 | 852 | \subsection{椭圆与双曲线的准线} 853 | 由抛物线的定义可知,抛物线上任一点到一定点(焦 854 | 点)的距离与它到一条定直线(准线)的距离之比等于常数 855 | 1, 这一节,我们将证明对椭圆和双曲线也存在着这样的定 856 | 直线,使椭圆和双曲线上任一点到焦点的距离与到定直线的 857 | 距离之比等于一常数,并且这个常数正好等于它们的离心率$e$. 858 | 859 | 设$P(x_1,y_1)$是椭圆$\frac{x^2}{a^2}+\frac{y^2}{b^2}=1$上任一点,我们在前面曾得到公式 860 | \begin{align} 861 | \overline{PF_1}=a+\frac{c}{a}x_1\\ 862 | \overline{PF_2}=a-\frac{c}{a}x_1 863 | \end{align} 864 | 把(6.10)式右边变形可得 865 | \[\overline{PF_1}=\frac{c}{a}\left(x_1+\frac{a^2}{c}\right)=e\left(x_1+\frac{a^2}{c}\right)\] 866 | 即:$\frac{\overline{PF_1}}{d_1}=e$,其中$d_1=x_1+\frac{a^2}{c}$.同样(6.11)式也可化为$\frac{\overline{PF_2}}{d_2}=e$,其中$d_2=-x_1+\frac{a^2}{c}$. 867 | 868 | 由计算点$P(x_1,y_1)$分别到直线$\ell_1:\; x+\frac{a^2}{c}=0$和 869 | $\ell_2:\; x-\frac{a^2}{c}=0$的距离可知,$d_1,d_2$正好分别是$P(x_1,y_1)$到$\ell_1$与$\ell_2$的距离,这说明\textbf{椭圆上任一点$P(x_1,y_1)$到焦点$F_1(-c,0)$($F_2(c,0)$)的距离与它到定直线 870 | $\ell_1$ ($\ell_2$) 的距离的比是一个常数}(等于离心率$e$),两条 871 | 直线 872 | \[\ell_1:\; x=-\frac{a^2}{c},\qquad \ell_2:\; x=\frac{a^2}{c}\] 873 | 分别叫做椭圆的左准线和右准线(图6.14). 874 | \begin{figure}[htp] 875 | \centering 876 | \begin{tikzpicture}[>=latex] 877 | \draw[->](-3.5,0)--(3.5,0)node[right]{$X$}; 878 | \draw[->](0,-1.5)--(0,2.5)node[right]{$Y$}; 879 | \node at (0,0)[below left]{$O$}; 880 | \draw[very thick](0,0) ellipse [x radius=2, y radius=1]; 881 | \tkzDefPoints{1.732/0/F_2, -1.732/0/F_1, 1.5/.66/P} 882 | \foreach \x/\xtext in {2.31/\ell_2,-2.31/\ell_1} 883 | { 884 | \draw(\x,-1.5)--(\x,2)node[right]{$\xtext$}; 885 | } 886 | \tkzDrawSegments[thick](P,F_1 P,F_2) 887 | \tkzLabelPoints[below](F_1,F_2) 888 | \tkzDrawPoints(F_1,F_2,P) 889 | \tkzLabelPoints[above](P) 890 | \draw(1.5,1.2)--(1.5,1.6); 891 | \draw[<->](-2.31,1.4)--node[above]{$d_1$}(1.5,1.4); 892 | \draw[<->](2.31,1.4)--node[above]{$d_2$}(1.5,1.4); 893 | \end{tikzpicture} 894 | \caption{} 895 | \end{figure} 896 | 897 | 反过来,我们也可证明:与定点$F_1(-c,0)$ ($F_2(c,0)$) 898 | 的距离和定直线$\ell_1:\; x=-\frac{a^2}{c}$ ($\ell_2:\; x=\frac{a^2}{c}$) 的距离的比等于 899 | 常数$e\; (0=latex] 904 | \draw[->](-2.5,0)--(2.5,0)node[right]{$X$}; 905 | \draw[->](0,-2.5)--(0,3)node[right]{$Y$}; 906 | \draw[domain=-2:2, samples=100, very thick]plot({sqrt(1+\x*\x)}, \x); 907 | \draw[domain=-2:2, samples=100, very thick]plot({-sqrt(1+\x*\x)}, \x); 908 | \tkzDefPoints{-1.414/0/F_1, 1.414/0/F_2, 2/1.732/P} 909 | \tkzDrawSegments[thick](F_1,P F_2,P) 910 | \tkzLabelPoints[below](F_1, F_2) 911 | \tkzDrawPoints(F_1, F_2, P) 912 | \tkzLabelPoints[above left](P) 913 | \node at (0,0)[below left]{$O$}; 914 | 915 | \foreach \x/\xtext in {0.707/\ell_2,-0.707/\ell_1} 916 | { 917 | \draw(\x,-2)--(\x,2.5)node[right]{$\xtext$}; 918 | } 919 | \draw[thick](P)--(-0.707,1.732); 920 | \end{tikzpicture} 921 | \caption{} 922 | \end{figure} 923 | 924 | 类比上述对椭圆的分析,同样也可证明,双曲线$\frac{x^2}{a^2}-\frac{y^2}{b^2}=1$也有两条准线(图6.15): 925 | 左准线$\ell_1:\; x=-\frac{a^2}{c}$;左准线$\ell_2:\; x=\frac{a^2}{c}$.并且具有如下特征性质. 926 | 927 | 双曲线$\frac{x^2}{a^2}-\frac{y^2}{b^2}=1$上任一点$P(x_1,y_1)$到焦点$F_1(-c, 928 | 0)$ ($F_2(c,0)$) 的距离和到定直线$\ell_1$ ($\ell_2$) 的距离的比 929 | 是一个常数(等于离心率$e$);反之也对. 930 | 931 | 总结以上讨论和抛物线的定义,我们可给圆锥曲线一个 932 | 统一的定义如下: 933 | 934 | \textbf{圆锥曲线是与一定点的距离和定直线的距离的比等于常 935 | 数$e$的点的轨迹,当$01$时是双曲线; 936 | $e=1$时是抛物线},定点叫做圆锥曲线的焦点,定直线叫做 937 | 圆锥曲线的准线.椭圆和双曲线有两个焦点和两条准线,抛 938 | 物线只有一个焦点和一条准线. 939 | 940 | \begin{example} 941 | 求椭圆$x+4y^2=100$的准线方程. 942 | \end{example} 943 | 944 | \begin{solution} 945 | 已知椭圆的标准方程为$\frac{x^2}{100}+\frac{y^2}{25}=1$,因此: 946 | \[a=10,\qquad b=5,\qquad c=\sqrt{c^2-b^2}=5\sqrt{3}\] 947 | 所以已知椭圆的准线方程为 948 | \[\ell_1:\; x=-\frac{a^2}{c}=-\frac{20\sqrt{3}}{3},\qquad \ell_2:\; x=\frac{20\sqrt{3}}{3}\] 949 | \end{solution} 950 | 951 | \begin{example} 952 | 求双曲线$x^2-y^2=1$的准线方程. 953 | \end{example} 954 | 955 | 956 | 957 | \begin{solution} 958 | 在已知双曲线方程中,$a=1$, $b=1$, 因此, 959 | \[c=\sqrt{a^2+b^2}=\sqrt{2}\] 960 | 所以已知双曲线的准线方程为 961 | \[\ell_1:\; x=-\frac{a^2}{c}=-\frac{\sqrt{2}}{2},\qquad \ell_2:\; x=\frac{\sqrt{2}}{2}\] 962 | \end{solution} 963 | 964 | \begin{ex} 965 | \begin{enumerate} 966 | \item 求椭圆$3x^2+4y^2=36$的焦点的坐标和准线方程并画出 967 | 草图. 968 | \item 求双曲线$2x^2-3y^2=6$的焦点的坐标和准线方程并画 969 | 出草图. 970 | \item 求下列每个椭圆或双曲线的准线方程 971 | \begin{multicols}{2} 972 | \begin{enumerate} 973 | \item $16x^2+25y^2=400$ 974 | \item $x^2+2y^2=4$ 975 | \item $x^2-5y^2=10$ 976 | \item $4x^2-3y^2=36$ 977 | \end{enumerate} 978 | \end{multicols} 979 | \end{enumerate} 980 | \end{ex} 981 | 982 | 983 | \subsection{圆锥曲线的切线} 984 | 我们知道,与圆只有一个公共点的直线叫做圆的切线, 985 | 但这个定义不能推广为一般曲线的切线的定义,如图6.16 986 | 所示,直线$\ell_1$虽然与曲线只有一个公共点,但它不是曲线的 987 | “切线”,直线$\ell_2$虽与曲线有两个公共点,但它与曲线“相 988 | 切”. 989 | 990 | \begin{figure}[htp]\centering 991 | \begin{minipage}[t]{0.48\textwidth} 992 | \centering 993 | \includegraphics[scale=.7]{fig/6-16.png} 994 | \caption{} 995 | \end{minipage} 996 | \begin{minipage}[t]{0.48\textwidth} 997 | \centering 998 | \includegraphics[scale=.7]{fig/6-17.png} 999 | \caption{} 1000 | \end{minipage} 1001 | \end{figure} 1002 | 1003 | 下面我们来阐述一般曲线的切线的定义,并由这个定义 1004 | 推导圆锥曲线的切线方程. 1005 | 1006 | \begin{blk} 1007 | {定义} 设$P_1$为曲线上一点,过$P_1$引割线$P_1P_2$交曲线 1008 | 于另一点$P_2$, 当$P_2$沿曲线无限趋近于点$P_1$时,割线$P_1P_2$ 1009 | 的极限位置$P_1T$叫做曲线在$P_1$点的切线(图6.17). 1010 | \end{blk} 1011 | 1012 | 已知椭圆$\frac{x^2}{a^2}+\frac{y^2}{b^2}=1$, 1013 | 设$P_1(x_1,y_1)$是椭圆上一定点,$P_2(x_2,y_2)$是椭圆上任 1014 | 一点,则椭圆的割线$P_1P_2$的方程为 1015 | \begin{equation} 1016 | y-y_1=\frac{y_2-y_1}{x_2-x_1}(x-x_1)=\frac{y^2_2-y^2_1}{(x_2-x_1)(y_2+y_1)}(x-x_1) 1017 | \end{equation} 1018 | 由于点$P_1(x_1,y_1)$, $P_2(x_2,y_2)$都在已给的椭圆上, 1019 | 所以 1020 | \[y^2_1=b^2\left(1-\frac{x^2_1}{a^2}\right),\qquad y^2_2=b^2\left(1-\frac{x^2_2}{a^2}\right)\] 1021 | 两式相减得 1022 | \[y_2^2-y_1^2=\frac{b^2}{a^2}(x^2_2-x^2_1)\] 1023 | 代入(6.12)化简即可得 1024 | \[y-y_1=\frac{b^2}{a^2}\cdot \frac{x_2+x_1}{y_2+y_1}(x-x_1)\] 1025 | 当$P_2$与$P_1$重合时,即$x_2=x_1$, $y_2=y_1$, 上式变为 1026 | \[y-y_1=\frac{b^2}{a^2}\cdot \frac{x_1}{y_1}(x-x_1)\] 1027 | 或 1028 | \[\frac{x_1x}{a^2}+\frac{y_1y}{b^2}=\frac{x_1^2}{a^2}+\frac{y_1^2}{b^2}\] 1029 | 即: 1030 | \begin{equation} 1031 | \boxed{\frac{x_1x}{a^2}+\frac{y_1y}{b^2}=1} 1032 | \end{equation} 1033 | (6.13)式就是椭圆$\frac{x^2}{a^2}+\frac{y^2}{b^2}=1$ 1034 | 在点$P_1(x_1,y_1)$的切线方程. 1035 | 1036 | 同理可证,双曲线 1037 | $\frac{x^2}{a^2}-\frac{y^2}{b^2}=1$ 1038 | 在点$P_1(x_1,y_1)$处的切线方程为 1039 | \begin{equation} 1040 | \boxed{\frac{x_1x}{a^2}-\frac{y_1y}{b^2}=1} 1041 | \end{equation} 1042 | 1043 | 抛物线$y^2=2px$在点$P_1(x_1,y_1)$处的切线方程为 1044 | \begin{equation} 1045 | \boxed{y_1y=p(x+x_1)} 1046 | \end{equation} 1047 | 1048 | 经过切点$P_1(x_1,y_1)$与切线垂直的直线叫做曲线在点 1049 | $P_1$的\textbf{法线}. 1050 | 1051 | 根据法线的定义可知,法线的方向向量可取切线的法向 1052 | 量,因此可得椭圆、双曲线、抛物线的在$P_1(x_1,y_1)$点的 1053 | 法线方程分别为 1054 | \begin{align} 1055 | \frac{x-x_1}{b^2x_1}&=\frac{y-y_1}{a^2y_1}\\ 1056 | \frac{x-x_1}{b^2x_1}&=\frac{y-y_1}{-a^2y_1}\\ 1057 | \frac{x-x_1}{p}&=\frac{y-y_1}{-y_1} 1058 | \end{align} 1059 | 1060 | 1061 | \begin{example} 1062 | 求椭圆:$2x^2+3y^2=35$在其上一点$P(2,3)$ 1063 | 的切线方程. 1064 | \end{example} 1065 | 1066 | \begin{solution} 1067 | 已知椭圆化为标准方程为 1068 | \[\frac{x^2}{\dfrac{35}{2}}+\frac{y^2}{\dfrac{35}{3}}=1\] 1069 | 所以,已知椭圆在点$P(2,3)$的切线方程为 1070 | \[\frac{2x}{\dfrac{35}{2}}+\frac{3y}{\dfrac{35}{3}}=1\] 1071 | 整理得:$4x+9y=35$ 1072 | \end{solution} 1073 | 1074 | \begin{example} 1075 | 设双曲线$\frac{x^2}{a^2}-\frac{y^2}{b^2}=1$ 1076 | 在其上任一点$P_0(x_0,y_0)$的切线与双曲线的两条渐近线分别相交于$A$、$B$两点(图6.18),求证$\triangle OAB$的面积等于常数$ab$. 1077 | \end{example} 1078 | 1079 | 1080 | 1081 | \begin{solution} 1082 | 已知双曲线在$P_0(x_0,y_0)$的切线方程为 1083 | \[\frac{x_0x}{a^2}-\frac{y_0y}{b^2}=1\] 1084 | 将它分别与渐近线方程$y=-\frac{b}{a}x$, 1085 | $y=\frac{b}{a}x$联立求解,就可分别得到 1086 | \[A\left(\frac{a^2b}{bx_0+ay_0},\frac{-ab^2}{bx_0+ay_0}\right),\qquad B\left(\frac{a^2b}{bx_0-ay_0},\frac{ab^2}{bx_0-ay_0}\right)\] 1087 | 所以 1088 | \[\begin{split} 1089 | S_{\triangle OAB}&=\frac{1}{2}\begin{vmatrix} 1090 | \frac{a^2b}{bx_0+ay_0}& \frac{-ab^2}{bx_0+ay_0}\\ 1091 | \frac{a^2b}{bx_0-ay_0}& \frac{ab^2}{bx_0-ay_0} 1092 | \end{vmatrix}\\ 1093 | &=\frac{1}{2}\left(\frac{a^3b^3}{b^2x^2_0-a^2y_0^2}+\frac{a^3b^3}{b^2x^2_0-a^2y_0^2}\right)\\ 1094 | &=\frac{a^3b^3}{b^2x^2_0-a^2y_0^2}=\frac{a^3b^3}{a^2b^2}=ab 1095 | \end{split}\] 1096 | \end{solution} 1097 | 1098 | \begin{figure}[htp]\centering 1099 | \begin{minipage}[t]{0.48\textwidth} 1100 | \centering 1101 | \begin{tikzpicture}[>=latex, scale=1] 1102 | \draw[->](-1,0)--(4,0)node[right]{$X$}; 1103 | \draw[->](0,-3)--(0,3)node[right]{$Y$}; 1104 | \node at (0,0)[below left]{$O$}; 1105 | \draw(3,3)--(0,0)--(3,-3); 1106 | \draw[domain=-2.5:2.5, samples=100, thick]plot({sqrt(1+\x*\x)},\x); 1107 | \draw[domain=-.5:3, samples=10, thick]plot(\x, {-0.89+1.34*\x}); 1108 | 1109 | \tkzDefPoints{1.5/1.12/P_0, .38/-.38/A, 2.62/2.62/B} 1110 | \tkzDrawPoints(P_0,A,B) 1111 | \tkzLabelPoints[right](P_0) 1112 | \tkzLabelPoints[left](B) 1113 | \tkzLabelPoints[below](A) 1114 | 1115 | \end{tikzpicture} 1116 | \caption{} 1117 | \end{minipage} 1118 | \begin{minipage}[t]{0.48\textwidth} 1119 | \centering 1120 | \begin{tikzpicture}[>=latex, scale=1] 1121 | \draw[->](-2,0)--(3,0)node[right]{$X$}; 1122 | \draw[->](0,-3)--(0,3)node[right]{$Y$}; 1123 | \node at (0,0)[below left]{$O$};\node at (-.5,0)[below left]{$T$}; 1124 | \draw(-.5,-2.5)--(-.5,2.5); 1125 | \draw[domain=-2.2:2.2, samples=100, thick]plot({0.5*\x*\x}, \x); 1126 | \draw(-1,1.414)--(2.5,1.414)node[right]{$E$}; 1127 | \tkzDefPoints{1/1.414/M, 0.5/0/F, 2/0/N, 2.5/1.414/E} 1128 | \tkzDrawSegments(M,F M,N) 1129 | \tkzDrawPoints(M,F,N) 1130 | \tkzLabelPoints[below](F,N)\tkzLabelPoints[above](M) 1131 | \draw[domain=-2:3, samples=100]plot(\x, {0.707*\x+0.707}); 1132 | \tkzMarkAngles[mark=none, size=.3](M,N,F F,M,N) 1133 | \tkzMarkAngles[mark=none, size=.4](N,M,E) 1134 | \end{tikzpicture} 1135 | \caption{} 1136 | \end{minipage} 1137 | \end{figure} 1138 | 1139 | \begin{example} 1140 | 设$F$是抛物线$y^2=2px$的焦点,$M$是抛物线上任 1141 | 一点,$MT$是抛物线在点$M$的切线,$MN$是法线并与$X$轴相 1142 | 交于$N$点,直线$ME$平行$X$轴(图6.19), 1143 | 1144 | 求证:$\angle FMN=\angle NME$. 1145 | \end{example} 1146 | 1147 | \begin{solution} 1148 | 设$M(x_1,y_1)$, 则由在$M$点的切线方程可得在 1149 | $M$点的法线方程为 1150 | \[y-y_1=-\frac{y_1}{p}(x-x_1)\] 1151 | 令$y=0$, 1152 | 得$N(x_1+p,0)$, 1153 | 所以 1154 | \[\overline{FN}=x_1+p-\frac{p}{2}= x_1+\frac{p}{2}\] 1155 | 又由抛物线的定义可知,$\overline{MF}$等于$M$点到准线$x=-\frac{p}{2}$的距离,即 1156 | \[\overline{FM}=x_1+\frac{p}{2}\] 1157 | 所以 1158 | $\overline{FN}=\overline{FM},\quad \angle FMN=\angle FNM$, 1159 | 但$\angle FNM=\angle NME$, 1160 | 所以 1161 | \[\angle FMN=\angle NME\] 1162 | \end{solution} 1163 | 1164 | 例6.12所证结论告诉我们,如果一族平行光线照射到抛物 1165 | 线上,经抛物线反射都通过焦点,抛物线这种光学性质有许 1166 | 多用途,例如太阳能灶的聚光镜,把太阳光线(看作平行) 1167 | 集中到焦点上,在焦点产生高温,探照灯把放在焦点处光 1168 | 源发出的光线经镜面反射后成为平行光线等. 1169 | 1170 | 我们同样可以证明,椭圆和双曲线具有如下性质. 1171 | \begin{itemize} 1172 | \item 椭圆的法线平分切点与两个焦点连线所成的角(图6.20). 1173 | \item 双曲线的法线平分切点与两个焦点连线所成角的邻补角 1174 | (图6.21). 1175 | \end{itemize} 1176 | 1177 | 我们把证明留给同学.作为练习. 1178 | 1179 | \begin{figure}[htp]\centering 1180 | \begin{minipage}[t]{0.48\textwidth} 1181 | \centering 1182 | \begin{tikzpicture}[>=latex, scale=1] 1183 | \draw[->](-2.5,0)--(2.75,0)node[right]{$X$}; 1184 | \draw[->](0,-2)--(0,2.5)node[right]{$Y$}; 1185 | \draw[thick](0,0) ellipse [x radius=2, y radius=1.25]; 1186 | \node at (0,0)[below left]{$O$}; 1187 | \tkzDefPoints{1.5/0/F_2, -1.5/0/F_1, 1.25/0.976/M} 1188 | \tkzDrawPoints(F_1,F_2,M)\tkzLabelPoints[below](F_1,F_2) 1189 | \tkzLabelPoints[above](M) 1190 | \tkzDrawSegments(F_1,M F_2,M) 1191 | \draw[domain=-1:2.5, samples=10]plot(\x, {1.6-0.5*\x}); 1192 | \draw[domain=.76:1.25, samples=10]plot(\x, {2*\x-1.524}); 1193 | 1194 | \tkzDefPoints{.76/0/S1} 1195 | \tkzMarkAngle[mark=none, size=.35](F_1,M,S1) 1196 | \tkzMarkAngle[mark=none, size=.45](S1,M,F_2) 1197 | 1198 | \end{tikzpicture} 1199 | \caption{} 1200 | \end{minipage} 1201 | \begin{minipage}[t]{0.48\textwidth} 1202 | \centering 1203 | \begin{tikzpicture}[>=latex, scale=.8] 1204 | \draw[->](-2.5,0)--(4.5,0)node[right]{$X$}; 1205 | \draw[->](0,-2.5)--(0,3)node[right]{$Y$}; 1206 | \draw[domain=-2:2, samples=100, thick]plot({sqrt(1+\x*\x)}, \x); 1207 | \draw[domain=-2:2, samples=100, thick]plot({-sqrt(1+\x*\x)}, \x); 1208 | \tkzDefPoints{-1.414/0/F_1, 1.414/0/F_2, 2/1.732/M} 1209 | \tkzDrawSegments(F_2,M) 1210 | \tkzDrawLines[add=.25 and .35](F_1,M) 1211 | \tkzLabelPoints[below](F_1, F_2) 1212 | \tkzDrawPoints(F_1, F_2, M) 1213 | \tkzLabelPoints[above left](M) 1214 | \node at (0,0)[below left]{$O$}; 1215 | \draw[domain=.2:2.5, samples=10]plot(\x, {1.155*\x-0.577}); 1216 | \draw(M)--+(-40.9:3); 1217 | \tkzDefPoints{4/0/S1, 2.828/2.15/S2} 1218 | \tkzMarkAngle[mark=none, size=.35](F_2,M,S1) 1219 | \tkzMarkAngle[mark=none, size=.5](S1,M,S2) 1220 | \end{tikzpicture} 1221 | \caption{} 1222 | \end{minipage} 1223 | \end{figure} 1224 | 1225 | 上述性质说明,椭圆和双曲线具有类似于抛物线的光学 1226 | 性质,由椭圆一个焦点射出的光线照射到椭圆上,经过反射 1227 | 后都通过另一焦点(图6.22),在双曲线一个焦点发出的 1228 | 光线,照射到双曲线上,经过反射,会使光线散开,如同光 1229 | 线从另一个焦点发出来的光线一样(图6.23). 1230 | 1231 | 1232 | \begin{figure}[htp]\centering 1233 | \begin{minipage}[t]{0.48\textwidth} 1234 | \centering 1235 | \begin{tikzpicture}[>=latex, scale=1] 1236 | \draw[thick](0,0) ellipse [x radius=2, y radius=1.25]; 1237 | \tkzDefPoints{1.5/0/F_2, -1.5/0/F_1} 1238 | \tkzLabelPoints[right](F_1, F_2) 1239 | \tkzDefPoints{-1/1.08/S1, -1/-1.08/S2, 0/1.25/S3, 0/-1.25/S4} 1240 | \tkzDrawSegments[->](F_1,S1 F_1,S2 F_1,S3 F_1,S4 S1,F_2 S2,F_2 S3,F_2 S4,F_2) 1241 | \end{tikzpicture} 1242 | \caption{} 1243 | \end{minipage} 1244 | \begin{minipage}[t]{0.48\textwidth} 1245 | \centering 1246 | \begin{tikzpicture}[>=latex, scale=1] 1247 | \draw[->](-2.5,0)--(2.5,0)node[right]{$X$}; 1248 | \draw[->](0,-2)--(0,2.5)node[right]{$Y$}; 1249 | \draw[domain=-2:2, samples=100, thick]plot({sqrt(1+\x*\x)}, \x); 1250 | \draw[domain=-2:2, samples=100, thick]plot({-sqrt(1+\x*\x)}, \x); 1251 | \tkzDefPoints{-1.414/0/F_1, 1.414/0/F_2} 1252 | \tkzDefPoints{-1.2/.66/S1, -1.4/.98/S2, -1.6/1.25/S3, -1.2/-.66/S4, -1.4/-.98/S5, -1.6/-1.25/S6} 1253 | \tkzDrawSegments[dashed](F_2,S1 F_2,S2 F_2,S3 F_2,S4 F_2,S5 F_2,S6) 1254 | \tkzDrawSegments[thick,->](F_1,S1 F_1,S2 F_1,S3 F_1,S4 F_1,S5 F_1,S6) 1255 | \tkzLabelPoints[below](F_2) 1256 | \tkzLabelPoints[below left](F_1) 1257 | \node at (0,0)[below left]{$O$}; 1258 | \foreach \x in {1,2,...,6} 1259 | { 1260 | \tkzDefPointWith[linear, K=1.5](F_2,S\x) \tkzGetPoint{D\x} 1261 | \tkzDrawSegments[thick,->](S\x,D\x) 1262 | } 1263 | \end{tikzpicture} 1264 | \caption{} 1265 | \end{minipage} 1266 | \end{figure} 1267 | 1268 | \begin{ex} 1269 | \begin{enumerate} 1270 | \item 证明本小节的双曲线的切线方程(6.14). 1271 | \item 证明本小节的抛物线的切线方程(6.15). 1272 | \item 已知如下各曲线上一点的坐标,求在这点的切线和法线 1273 | 方程. 1274 | \begin{enumerate} 1275 | \item $\frac{x^2}{225}+\frac{y^2}{25}=1,\qquad (9,4)$ 1276 | \item $2x^2+3y^2=14,\qquad (1,-2)$ 1277 | \item $4x^2-y^2=15,\qquad (2,-1)$ 1278 | \item $y^2=3x,\qquad (12,6)$ 1279 | \end{enumerate} 1280 | 1281 | \item 求抛物线$y^2=4x$在点$(36,12)$的切线在$X$轴上的截 1282 | 距. 1283 | \item 已知椭圆$\frac{x^2}{a^2}+\frac{y^2}{b^2}=1$ 1284 | 在$P_1(x_1,y_1)$的切线与过两顶 1285 | 点$A_1,A_2$且垂直于$X$轴的两条直线分别相交于$C$、$D$ 1286 | 两点,求证:$\overline{A_1C}\cdot \overline{A_2D}=b^2$. 1287 | \item 求证:双曲线两条渐近线之间的切线段被切点等分. 1288 | \end{enumerate} 1289 | \end{ex} 1290 | 1291 | \subsection{圆锥曲线的直径} 1292 | \begin{blk} 1293 | {定义} 通过椭圆(双曲线)中心的直线,叫做\textbf{椭圆(双 1294 | 曲线)的直径},与抛物线的轴平行的直线叫做\textbf{抛物线的直径}. 1295 | 1296 | 如果一条直线与圆锥曲线相交于两点,那么交点间的线 1297 | 段叫做\textbf{圆锥曲线的弦}(图6.24). 1298 | \end{blk} 1299 | 1300 | \begin{figure}[htp] 1301 | \centering 1302 | \begin{tikzpicture}[>=latex] 1303 | \begin{scope} 1304 | \draw[->](-2,0)--(2,0)node[right]{$X$}; 1305 | \draw[->](0,-2)--(0,2)node[right]{$Y$}; 1306 | \node at (0,0)[below left]{$O$}; 1307 | \draw [pattern=north east lines](0,0) ellipse [x radius=1.5, y radius=1]; 1308 | \draw(-2,1.5)--(2,-1.5); 1309 | \node at (0,-2.5){(1)}; 1310 | \end{scope} 1311 | 1312 | \begin{scope}[xshift=5cm] 1313 | \draw[->](-1,0)--(3,0)node[right]{$X$}; 1314 | \draw[->](0,-2)--(0,2)node[right]{$Y$}; 1315 | \draw(0,1)--(3,1); 1316 | \node at (0,0)[below left]{$O$}; 1317 | \draw[pattern=north east lines, domain=-1.5:1.5 ,samples=100, thick] plot({\x*\x},\x); 1318 | \node at (0,-2.5){(2)}; 1319 | \end{scope} 1320 | 1321 | \begin{scope}[yshift=-5cm] 1322 | \draw[->](-2,0)--(2,0)node[right]{$X$}; 1323 | \draw[->](0,-2)--(0,2)node[right]{$Y$}; 1324 | \node at (0,0)[below left]{$O$}; 1325 | \fill[pattern=north west lines, domain=-1.5:1.5 ,samples=100] plot({sqrt(1+\x*\x)},\x)--(0,1.5)--(0,-1.5)--(1.8,-1.5); 1326 | \fill[pattern=north west lines, domain=-1.5:1.5 ,samples=100] plot({-sqrt(1+\x*\x)},\x)--(0,1.5)--(0,-1.5)--(-1.8,-1.5); 1327 | \draw[domain=-1.5:1.5 ,samples=100, thick] plot({sqrt(1+\x*\x)},\x); 1328 | \draw[domain=-1.5:1.5 ,samples=100, thick] plot({-sqrt(1+\x*\x)},\x); 1329 | \node at (0,-2.5){(3)}; 1330 | \draw(-1,2)--(1,-2); 1331 | \end{scope} 1332 | 1333 | \begin{scope}[xshift=5cm, yshift=-5cm] 1334 | \draw[->](-2,0)--(2,0)node[right]{$X$}; 1335 | \draw[->](0,-2)--(0,2)node[right]{$Y$}; 1336 | \node at (0,0)[below left]{$O$}; 1337 | \node at (0,-2.5){(4)}; 1338 | \draw[pattern=north east lines, domain=-1.5:1.5 ,samples=100, thick] plot({sqrt(1+\x*\x)},\x); 1339 | \draw[pattern=north east lines, domain=-1.5:1.5 ,samples=100, thick] plot({-sqrt(1+\x*\x)},\x); 1340 | \draw(-2,1)--(2,-1); 1341 | 1342 | \end{scope} 1343 | \end{tikzpicture} 1344 | \caption{} 1345 | \end{figure} 1346 | 1347 | 1348 | \begin{blk} 1349 | {定理} 圆锥曲线的平行弦的中点在直径上. 1350 | \end{blk} 1351 | 1352 | 我们以椭圆为例加以证明,关于双曲线和抛物线的情况 1353 | 留给同学作为练习. 1354 | 1355 | 已知椭圆$\frac{x^2}{a^2}+\frac{y^2}{b^2}=1$, 求证它的一族平行弦的中点在它的一条直径上(图6.24(1)). 1356 | 1357 | \begin{proof} 1358 | 如果平行弦垂直于对称轴,那么,由椭圆的对称 1359 | 性,定理显然成立.我们来证明一般情况. 1360 | 设平行弦所在的平行线系方程为 1361 | $y=kx+c,\quad k\ne 0$. 1362 | 代入已知椭圆方程整理得 1363 | \[(b^2+a^2k^2)x^2+2a^2kcx+a^2(c^2-b^2)=0\] 1364 | 设这个二次方程的两个根为$x_1,x_2$, 则: 1365 | \[x_1+x_2=-\frac{2a^2kc}{b^2+a^2k^2}\] 1366 | 因此平行弦中点的横坐标 1367 | \[x=\frac{x_1+x_2}{2}=-\frac{a^2kc}{b^2+a^2k^2}\] 1368 | 代入直线系方程得中点的纵坐标 1369 | \[y=-\frac{b^2c}{b^2+a^2k^2}\] 1370 | 于是 1371 | \[\frac{y}{x}=-\frac{b^2}{a^2k}\] 1372 | 所以平行弦中点的坐标都在直线$y=-\frac{b^2}{a^2k}x$ 1373 | 上,这条直线通过椭圆中心,因此它是椭圆的直径. 1374 | \end{proof} 1375 | 1376 | 已知椭圆$\frac{x^2}{a^2}+\frac{y^2}{b^2}=1$的一族平行弦平行于直径$y=kx$,则直径$y=k'x,\quad \left(k'=-\frac{b^2}{a^2k}\right)$ 1377 | 叫做$y=kx$的\textbf{共轭直径}. 1378 | 1379 | 上述定理也就是说,与一条直径平行的弦的中点都在它 1380 | 的共轭直径上.显然直径的共轭性是相互的(图6.25). 1381 | 1382 | \begin{figure}[htp]\centering 1383 | \begin{minipage}[t]{0.48\textwidth} 1384 | \centering 1385 | \begin{tikzpicture}[>=latex, scale=1] 1386 | \draw[->](-2,0)--(2,0)node[right]{$X$}; 1387 | \draw[->](0,-2)--(0,2)node[right]{$Y$}; 1388 | \node at (0,0)[below left]{$O$}; 1389 | \draw (0,0) ellipse [x radius=1.5, y radius=1]; 1390 | \draw(-2,1)--(2,-1); 1391 | \draw(-2,-1)--(2,1); 1392 | \end{tikzpicture} 1393 | \caption{} 1394 | \end{minipage} 1395 | \begin{minipage}[t]{0.48\textwidth} 1396 | \centering 1397 | \begin{tikzpicture}[>=latex, scale=1] 1398 | \draw[->](-2,0)--(2,0)node[right]{$X$}; 1399 | \draw[->](0,-2)--(0,2)node[right]{$Y$}; 1400 | \node at (0,0)[below left]{$O$}; 1401 | \draw (0,0) ellipse [x radius=1.5, y radius=1]; 1402 | \draw[domain=-1.2:1.2, samples=10]plot(\x, {0.5*\x}); 1403 | \draw[domain=-1.2:1.2, samples=10]plot(\x, {-0.5*\x}); 1404 | \draw[domain=-1:2, samples=10]plot(\x, {-0.5*\x+1.25}); 1405 | \draw[domain=-2:1, samples=10]plot(\x, {-0.5*\x-1.25}); 1406 | \node at (.56,.93)[above]{$P_0$}; 1407 | \end{tikzpicture} 1408 | \caption{} 1409 | \end{minipage} 1410 | \end{figure} 1411 | 1412 | \begin{example} 1413 | $P_0(x_0,y_0)$是椭圆$\frac{x^2}{a^2}+\frac{y^2}{b^2}=1$与它的一条 1414 | 直径$y=kx$的交点,求证椭圆在$P_0(x_0,y_0)$的切线平行于 1415 | 这条直径的共轭直径(图6.26). 1416 | \end{example} 1417 | 1418 | \begin{proof} 1419 | 椭圆在$(x_0,y_0)$点的切线方程是$\frac{x_0x}{a^2}+\frac{y_0y}{b^2}=1$,它的斜率$k'=-\frac{x_0b^2}{a^2y_0}$, 1420 | 但$(x_0,y_0)$在已知直径上,所以 1421 | \[\frac{y_0}{x_0}=k,\qquad k'=-\frac{b^2}{a^2k}\] 1422 | 又已知直径$y=kx$的共轭直径是 1423 | $y=-\frac{b^2}{a^2k}x$,所以切线 1424 | 与共轭直径平行. 1425 | \end{proof} 1426 | 1427 | 1428 | \begin{ex} 1429 | \begin{enumerate} 1430 | \item 对双曲线和抛物线情况证明本节定理. 1431 | \item 已知椭圆$3x^2+4y^2=12$, 求倾角为$135^{\circ}$的椭圆平行弦 1432 | 中点所在的直线方程. 1433 | \item 已知双曲线$2x^2-y^2=6$, 它的一族平行弦的倾角是 1434 | $30^{\circ}$, 求这族平行弦中点所在的直线方程. 1435 | \item 在练习2、3中写出与弦平行的直径和它的共轭直径的 1436 | 方程. 1437 | \item 已知抛物线$y^2=6x$的一族平行弦的斜率是$1/2$, 1438 | 求平分这族平行弦的直径方程. 1439 | \item 设$P_0(x_0,y_0)$是双曲线 1440 | $\frac{x^2}{a^2}-\frac{y^2}{b^2}=1$ 1441 | 与它的一条直 1442 | 径$y=kx$的交点,求证:双曲线在$P_0(x_0,y_0)$的切线 1443 | 平行于这条直径的共轭直径. 1444 | \end{enumerate} 1445 | \end{ex} 1446 | 1447 | \section*{习题6.1} 1448 | 1449 | \addcontentsline{toc}{subsection}{习题6.1} 1450 | 1451 | \begin{enumerate} 1452 | \item 在椭圆$24x^2+30y^2=720$上,求与短轴相距为5的点 1453 | 的坐标. 1454 | \item 一椭圆以坐标轴为对称轴,坐标原点为对称中心且经过 1455 | 点$M(\sqrt{3},-2)$, $N(-2\sqrt{3},1)$, 求此椭圆的方 1456 | 程. 1457 | \item 点$P(x_1,y_1)$和点$Q(x_2,y_2)$分别位于椭圆的内部 1458 | 和外部,求证 1459 | \[\frac{x_1^2}{a^2}+\frac{y_1^2}{b^2}<1,\qquad \frac{x_2^2}{a^2}+\frac{y_2^2}{b^2}>1\] 1460 | \item 已知一椭圆的准线方程是$x=\pm 8$, 短轴长等于8, 求 1461 | 此椭圆的方程. 1462 | \item 已知椭圆$36x^2+100y^2=3600$, 在它上面求一点使这点 1463 | 到右焦点的距离是这点到左焦点距离的4倍. 1464 | \item 已知椭圆中心在原点,它的一个焦点是$F_2(3,0)$, 求 1465 | 其上一点$M(4,2.4)$到准线的距离. 1466 | \item 求下列各双曲线标准方程. 1467 | \begin{enumerate} 1468 | \item 两焦点间的距离是8, 两准线间的距离是6; 1469 | \item 已知两条准线方程是$x=\pm 3\sqrt{2}$, 两条渐近线的 1470 | 夹角是直角. 1471 | \item 已知渐近线方程是$y=\pm 2x$, 两个焦点距中心的距 1472 | 离是5. 1473 | \item 已知渐近线方程是$y=\pm \frac{5}{3}x$, 且双曲线通过点 1474 | $N(6,9)$. 1475 | \end{enumerate} 1476 | 1477 | \item 根据下列已知条件,求双曲线$\frac{x^2}{a^2}-\frac{y^2}{b^2}=1$的渐近线 1478 | 方程. 1479 | \begin{enumerate} 1480 | \item 离心率$e=2$; 1481 | \item 两焦点间的距离是二准线间距离的2倍. 1482 | \end{enumerate} 1483 | 1484 | \item 根据下列已知条件,求双曲线$\frac{x^2}{a^2}-\frac{y^2}{b^2}=1$的离心率. 1485 | \begin{enumerate} 1486 | \item 两渐近线之间的夹角是$60^{\circ}$; 1487 | \item 两渐近线之间的夹角是$90^{\circ}$. 1488 | \end{enumerate} 1489 | 1490 | \item 已知等轴双曲线$x^2-y^2=8$, 求一抛物线方程使它与 1491 | 已知双曲线有公共焦点且通过点$M(-5,3)$. 1492 | \item 通过点$A(2,-5)$引直线平行于双曲线$x^2-4y^2=4$的 1493 | 渐近线,求此直线的方程. 1494 | 1495 | \item 通过点$A(3,-1)$作双曲线$\frac{x^2}{4}-y^2=1$的弦且被$A$点 1496 | 平分,求此弦的方程. 1497 | \item 求下列抛物线方程,已知 1498 | \begin{enumerate} 1499 | \item 顶点在$(0,0)$, 焦点在$(2,0)$; 1500 | \item 顶点在$(0,0)$, 准线是$2x+5=0$; 1501 | \item 顶点在$(0,0)$, 准线是$2y-1=0$; 1502 | \item 顶点在$(0,0)$, 焦点在$(0,-3/5)$. 1503 | \end{enumerate} 1504 | 1505 | \item 一条抛物线顶点在原点,它的轴是$X$轴并且它通过点 1506 | $M(-1,1)$, 求它的方程. 1507 | 1508 | \item 求椭圆$\frac{x^2}{6}+\frac{y^2}{3}=1$的内接正方形每边所在直线的方程. 1509 | \item 求直线$Ax+By+C=0$与椭圆$\frac{x^2}{a^2}+\frac{y^2}{b^2}=1$相切的条 1510 | 件. 1511 | 1512 | \item 已知椭圆$\frac{x^2}{25}-\frac{y^2}{9}=1$ 1513 | 的两个焦点到它的某条切线的距离之比是9, 求此切线方程. 1514 | \item 求双曲线$\frac{x^2}{8}-\frac{y^2}{9}=1$在下列各点的切线,$(2\sqrt{2},0)$, $(-4,3)$. 1515 | \item 一条双曲线在点$M(4,2)$与直线$x-y-2=0$相切. 1516 | 求此双曲线的方程. 1517 | \item 求直线:$Ax+By+C=0$与双曲线 1518 | $\frac{x^2}{a^2}-\frac{y^2}{b^2}=1$相切 1519 | 的条件. 1520 | \item 已知抛物线$y^2=12x$, 根据下列各条件,求它的切线方 1521 | 程. 1522 | \begin{enumerate} 1523 | \item 切点的横坐标$x=3$; 1524 | \item 平行于直线$3x-y+5=0$; 1525 | \item 垂直于直线$2x+y-7=0$; 1526 | \item 与直线$4x-2y+9=0$交成$\pi/4$角. 1527 | \end{enumerate} 1528 | 1529 | \item 求直线$y=kx+b$与抛物线$y^2=2px$相切的条件. 1530 | \item 直线$x+y=1$与椭圆相交于$C$和$D$两点,求弦$\overline{CD}$的 1531 | 中点的坐标. 1532 | \item 已知椭圆$\frac{x^2}{a^2}+\frac{y^2}{b^2}=1$, 在其上一点$P$的切线和法线分 1533 | 别与$X$轴相交于$T$点和$G$点,过焦点$F_1$、$F_2$和原点分 1534 | 别作切线的垂线,设垂足分别为$V$、$U$、$K$, 过$P$点作 1535 | $X$轴的垂线,设垂足为$N$, 求证: 1536 | \begin{enumerate} 1537 | \item $\overline{ON}\cdot \overline{OT}=a^2$ 1538 | \item $\overline{PG}\cdot \overline{OK}=b^2$ 1539 | \item $\overline{OG}=e^2\cdot \overline{ON}$ 1540 | \item $\overline{F_1V}\cdot \overline{F_2U}=b^2$ 1541 | \end{enumerate} 1542 | \end{enumerate} 1543 | 1544 | 1545 | \section{坐标变换} 1546 | \subsection{坐标轴的平移} 1547 | 不改变坐标轴的方向和长度单位,只变换原点的位置, 1548 | 这种坐标系的变换叫做\textbf{坐标轴的平移},简称\textbf{移轴}. 1549 | 1550 | 给定一坐标系$OXY$, 平移 1551 | 坐标轴得到新坐标系$O'X'Y'$, 1552 | 下面我们来确定平面上任意一点 1553 | $P$的新坐标$(x',y')$与原坐标 1554 | $(x,y)$之间的关系(图6.27). 1555 | 1556 | \begin{figure}[htp] 1557 | \centering 1558 | \begin{tikzpicture}[>=latex] 1559 | \draw[->](-1,0)--(3,0)node[right]{$X$}; 1560 | \draw[->](-1,1)--(3,1)node[right]{$X'$}; 1561 | \draw[->](0,-1)--(0,4)node[right]{$Y$}; 1562 | \draw[->](1,-1)--(1,4)node[right]{$Y'$}; 1563 | \node at (0,0)[below left]{$O$}; 1564 | \node at (1,1)[below right]{$O'$}; 1565 | \draw[->, thick](0,0)--(1,1); 1566 | \draw[->, thick](0,0)--(2.1,3.3)node[right]{$P$}; 1567 | \draw[->, thick](1,1)--(2.1,3.3); 1568 | \end{tikzpicture} 1569 | \caption{} 1570 | \end{figure} 1571 | 1572 | 设$O'$在坐标系$OXY$中的 1573 | 坐标为$(h,k)$, 则在坐标系$OXY$中 1574 | \[\Vec{OO'}=(h,k),\qquad \Vec{OP}=(x,y),\qquad \Vec{O'P}=(x',y')\] 1575 | 因为$\Vec{OP}=\Vec{OO'}+\Vec{O'P}$,所以 1576 | \[(x,y)=(h,k)+(x',y')\] 1577 | 即: 1578 | \begin{equation} 1579 | x=x'+h,\qquad y=y'+k 1580 | \end{equation} 1581 | 或 1582 | \begin{equation} 1583 | x'=x-h,\qquad y'=y-k 1584 | \end{equation} 1585 | 公式(6.19), (6.20)叫做\textbf{平移公式}或\textbf{移轴公式}. 1586 | 1587 | 1588 | \begin{example} 1589 | 给定一坐标系$OXY$, 平移坐标轴,原点移到 1590 | $O'(3,2)$, 求$A(5,6)$的新坐标. 1591 | \end{example} 1592 | 1593 | \begin{solution} 1594 | 把$A$、$O'$点的坐标代入平移公式(6.20), 得 1595 | \[x'=5-3=2,\qquad y'=6-2=4\] 1596 | 即点$A$在新坐标系$O'X'Y'$中的坐标为$(2,4)$. 1597 | \end{solution} 1598 | 1599 | 1600 | 1601 | \begin{example} 1602 | 平移坐标轴,化简圆的方程$x^2+y^2+2x-6y+6=0$ 1603 | \end{example} 1604 | 1605 | \begin{solution} 1606 | 把已知圆的方程配方得 1607 | \begin{equation} 1608 | (x+1)^2+(y-3)^2=4 1609 | \end{equation} 1610 | 设它上面任一点的新坐标为 1611 | $(x',y')$, 平移坐标轴使 1612 | \[x'=x+1,\qquad y'=y-3\] 1613 | 即:$x=x'-1,\qquad y=y'+3$,代入(6.21),得到新方程为(图6.28) 1614 | \[{x'}^2+{y'}^2=4\] 1615 | \end{solution} 1616 | 1617 | \begin{figure}[htp] 1618 | \centering 1619 | \begin{tikzpicture}[>=latex, scale=.5] 1620 | \draw[->](-6,0)--(4,0)node[right]{$X$}; 1621 | \draw[->](0,-2)--(0,7)node[right]{$Y$}; 1622 | \draw[->, thick](-5.5,3)--(3.5,3)node[right]{$X'$}; 1623 | \draw[->, thick](-1,-2)--(-1,7)node[left]{$Y'$}; 1624 | \node at (0,0) [below right]{$O$}; 1625 | \node at (-1,3) [below left]{$O'$}; 1626 | \draw[thick](-1,3) circle (2); 1627 | \end{tikzpicture} 1628 | \caption{} 1629 | \end{figure} 1630 | 1631 | 1632 | 从例6.15可以看出,适当地 1633 | 变换坐标系,可以使曲线的方程简化.由于曲线的几何性质 1634 | 与我们选取的坐标系无关.所以,我们研究曲线时,总是想 1635 | 法选择能使曲线方程最为简单的坐标系,以便于我们研究曲 1636 | 线的性质. 1637 | 1638 | \begin{ex} 1639 | \begin{enumerate} 1640 | \item 平移坐标轴,使原点移至$O'(-2,.3)$, 求下列各点的 1641 | 新坐标,并画图. 1642 | \[ A(1,2),\quad B(0,-2),\quad C(-3,-2),\quad D(-3,-5)\] 1643 | \item 平移坐标轴,化简下列圆的方程,并画图. 1644 | \begin{enumerate} 1645 | \item $(x-1)^2+(y-2)^2=1$ 1646 | \item $(x+2)^2+(y-5)^2=9$ 1647 | \item $x^2+y^2+2x-6y+1=0$ 1648 | \end{enumerate} 1649 | 1650 | \item 平移坐标轴,使原点移至$O'(2,-1)$, 求曲线 1651 | $y^2+2y-4x+9=0$在新坐标系的方程. 1652 | \item 已知直线$\ell:\; 2x+3y-5=0$, 平移坐标轴,使原点移至 1653 | $O'\left(\frac{5}{2},0\right)$, 1654 | 求$\ell$在新坐标系中的方程. 1655 | \item 平移坐标轴,化简下列曲线方程,并画出曲线的草图. 1656 | \begin{enumerate} 1657 | \item $\frac{(x-3)^2}{25}+\frac{(y+3)^2}{9}=1$ 1658 | \item $(x+1)^2-(y-1)^2=1$ 1659 | \item $y=4(x+3)^2-1$ 1660 | \end{enumerate} 1661 | \end{enumerate} 1662 | \end{ex} 1663 | 1664 | \subsection{坐标轴的旋转} 1665 | 不改变坐标轴的原点和长 1666 | 度单位,只是坐标轴绕原点转 1667 | 一角度,这种坐标系的变换叫 1668 | 做\textbf{坐标轴的旋转},简称\textbf{转轴}. 1669 | 1670 | 给定一坐标系,坐标轴绕 1671 | 原点$O$转$\theta$角,得到一新坐标 1672 | 系$OX'Y'$(图6.29). 下 1673 | 面我们来确定平面上任一点$P$ 1674 | 的新坐标$(x',y')$与原坐标$(x,y)$之间的关系. 1675 | 1676 | \begin{figure}[htp] 1677 | \centering 1678 | \begin{tikzpicture}[>=latex, scale=.8] 1679 | \draw[->](-2,0)--(4,0)node[right]{$X$}; 1680 | \draw[->](0,-2)--(0,4)node[right]{$Y$}; 1681 | \draw[->](180+30:2)--(30:4)node[right]{$X'$}; 1682 | \draw[->](-60:2)--(90+30:4)node[right]{$Y'$}; 1683 | \draw[very thick,->](0,0)--(60:4)node[above]{$P$}; 1684 | \draw[dashed](2,0)--(60:4)--(0,2*1.732); 1685 | \draw[dashed](120:2)--(60:4); 1686 | \draw[<-, very thick](60:4)--+(-60:2); 1687 | \node at (0,0)[below left]{$O$}; 1688 | \draw[->](.8,0)arc (0:30:.8)node[right]{$\theta$}; 1689 | \draw[->](0,.8)arc (90:120:.8)node[above]{$\theta$}; 1690 | \foreach \x in {0,30,90,120} 1691 | { 1692 | \draw[->](0,0)--(\x:1.6); 1693 | } 1694 | \end{tikzpicture} 1695 | \caption{} 1696 | \end{figure} 1697 | 1698 | 设$\eX,\eY$和$\vec{e}_{x'},\vec{e}_{y'}$分别是两个坐标系中的基向 1699 | 量,则 1700 | \[\begin{split} 1701 | \Vec{OP}&=x\eX+y\eY=x'\vec{e}_{x'}+y'\vec{e}_{y'}\\ 1702 | \vec{e}_{x'}&=\cos\theta \eX+\sin\theta\eY\\ 1703 | \vec{e}_{y'}&=\cos\left(\frac{\pi}{2}+\theta\right)\eX+\sin\left(\frac{\pi}{2}+\theta\right)\eY=-\sin\theta\eX+\cos\theta\eY 1704 | \end{split}\] 1705 | 代入上式,得 1706 | \[x\eX+y\eY=(x'\cos\theta-y'\sin\theta)\eX+(x'\sin\theta+y'\cos\theta)\eY\] 1707 | 所以: 1708 | \begin{equation} 1709 | \boxed{\begin{cases} 1710 | x=x'\cos\theta-y'\sin\theta\\ 1711 | y=x'\sin\theta+y'\cos\theta 1712 | \end{cases}} 1713 | \end{equation} 1714 | 1715 | 由(6.22)解出$x',y'$得 1716 | \begin{equation} 1717 | \boxed{\begin{cases} 1718 | x'=x\cos\theta+y\sin\theta\\ 1719 | y'=-x\sin\theta+y\cos\theta 1720 | \end{cases}} 1721 | \end{equation} 1722 | 1723 | (6.22)式是用新坐标来表示原坐标的公式,(6.23)式是用原 1724 | 坐标来表示新坐标的公式,它们统称为\textbf{旋转公式}或转轴公 1725 | 式. 1726 | 1727 | \begin{example} 1728 | 把坐标轴旋转$\frac{\pi}{3}$, 1729 | 求点$P(-2,3)$在新坐标系中 1730 | 的坐标. 1731 | \end{example} 1732 | 1733 | \begin{solution} 1734 | 把已知量代入旋转公式(6.23), 得 1735 | \[\begin{split} 1736 | x'&=(-2)\cdot \cos\frac{\pi}{3}+3\cdot \sin\frac{\pi}{3}=\frac{3\sqrt{3}-2}{2},\\ 1737 | y'&=-(-2)\cdot\sin\frac{\pi}{3}+3\cdot \cos\frac{\pi}{3}=\frac{2\sqrt{3}+3}{2} 1738 | \end{split}\] 1739 | 所以,$P$点的新坐标是 1740 | $\left(\frac{3\sqrt{3}-2}{2}, \; \frac{2\sqrt{3}+3}{2}\right)$ 1741 | \end{solution} 1742 | 1743 | \begin{example} 1744 | 把坐标轴旋转$\frac{\pi}{4}$, 1745 | 求曲线$xy=1$ 1746 | 在新坐标系中的方程. 1747 | \end{example} 1748 | 1749 | 1750 | \begin{solution} 1751 | $\theta=\frac{\pi}{4}$,代入旋转公式(6.22), 得 1752 | \[\begin{split} 1753 | x&=x'\cos\frac{\pi}{4}-y'\sin \frac{\pi}{4}=\frac{\sqrt{2}}{2}(x'-y')\\ 1754 | y&=x'\sin\frac{\pi}{4}+y'\cos\frac{\pi}{4}=\frac{\sqrt{2}}{2}(x'+y') 1755 | \end{split}\] 1756 | 代入$xy=1$, 得 1757 | \[\frac{1}{2}(x'-y')(x'+y')=1\] 1758 | 即: 1759 | \[\frac{{x'}^2}{\left(\sqrt{2}\right)^2}-\frac{{y'}^2}{\left(\sqrt{2}\right)^2}=1\] 1760 | 这就是曲线在新坐标系中的方 1761 | 程,容易看出,它是一条等轴 1762 | 双曲线(图6.30). 1763 | \end{solution} 1764 | 1765 | \begin{figure}[htp] 1766 | \centering 1767 | \begin{tikzpicture}[>=latex, scale=.7] 1768 | \draw[->](-4,0)--(4,0)node[right]{$X$}; 1769 | \draw[->](0,-4)--(0,4)node[left]{$Y$}; 1770 | \draw[->](180+45:4.5)--(45:4.5)node[right]{$X'$}; 1771 | \draw[->](-45:4.5)--(90+45:4.5)node[right]{$Y'$}; 1772 | \draw[domain=.25:3.5, samples=100, very thick]plot(\x,{1/\x}); 1773 | \draw[domain=-3.5:-.25, samples=100, very thick]plot(\x,{1/\x}); 1774 | \node at (0,0)[below left]{$O$}; 1775 | \end{tikzpicture} 1776 | \caption{} 1777 | \end{figure} 1778 | 1779 | \begin{ex} 1780 | \begin{enumerate} 1781 | \item 写出旋转角$\theta$为下列各值时的旋转公式(6.22)和(6.23): 1782 | \begin{multicols}{3} 1783 | \begin{enumerate} 1784 | \item $\theta=30^{\circ}$ 1785 | \item $\theta=-30^{\circ}$ 1786 | \item $\theta = 120^{\circ}$ 1787 | \item $\theta = 90^{\circ}$ 1788 | \item $\theta = -90^{\circ}$ 1789 | \item $\theta = 180^{\circ}$ 1790 | \end{enumerate} 1791 | \end{multicols} 1792 | 1793 | \item 已知$A(2,0)$, $B(3,1)$, $C(-2,4)$. 设旋转角 1794 | $\theta = \frac{\pi}{4}$, 1795 | 求它们在新坐标系中的坐标. 1796 | \item 设旋转角$\theta=45^{\circ}$, 写出下列曲线在新坐标系中的方程: 1797 | \begin{multicols}{2} 1798 | \begin{enumerate} 1799 | \item $x+y=0$ 1800 | \item $x^2+y^2=4$ 1801 | \item $3x^2-10xy+3y^2+8=0$ 1802 | \end{enumerate} 1803 | \end{multicols} 1804 | \end{enumerate} 1805 | \end{ex} 1806 | 1807 | \subsection{一般的坐标变换公式} 1808 | 设$OXY$, $O'X'Y'$是两个坐标系(图6.31),$O'$在 1809 | 坐标系$OXY$中的坐标是$(h,k)$, 容易看出,把坐标系 1810 | $OXY$作移轴变换,把原点 1811 | $O$移到$O'(h,k)$得到坐 1812 | 标系$O'XY$然后再绕 1813 | $O'$旋转$\theta$角就可得到坐标系 1814 | $O'X'Y'$, 这就说,上述的 1815 | 一般的坐标变换是平移与旋 1816 | 转的合成.下面我们来确 1817 | 定,平面上任意一点$P$的新 1818 | 坐标$(x',y')$与原坐标$(x,y)$之间的关系. 1819 | 1820 | \begin{figure}[htp] 1821 | \centering 1822 | \begin{tikzpicture}[>=latex] 1823 | \draw[->, thick](-3,0)--(4,0)node[right]{$X''$}; 1824 | \draw[->, thick](0,-2)--(0,4)node[right]{$Y''$}; 1825 | \draw[->, very thick](180+30:1)--(30:4)node[right]{$X'$}; 1826 | \draw[->, very thick](-60:1)--(90+30:3)node[right]{$Y'$}; 1827 | \draw[->](-3,-1)--(4,-1)node[right]{$X$}; 1828 | \draw[->](-2,-2)--(-2,4)node[right]{$Y$}; 1829 | 1830 | \node at (0,0)[below right]{$O'$}; 1831 | \node at (-2,-1)[below left]{$O$}; 1832 | \draw[->](.75,0)arc (0:30:.75)node[right]{$\theta$}; 1833 | \end{tikzpicture} 1834 | 1835 | \caption{} 1836 | \end{figure} 1837 | 1838 | 1839 | 设 $OXY$ 经过移轴后得到的坐标系为$O'XY$ 1840 | (图6.31),则由平移公式,得 1841 | \begin{equation} 1842 | \begin{cases} 1843 | x=x''+h\\ 1844 | y=y''+k 1845 | \end{cases} 1846 | \end{equation} 1847 | 再由旋转公式,得 1848 | \begin{equation} 1849 | \begin{cases} 1850 | x''=x'\cos\theta-y'\sin\theta\\ 1851 | y''=x'\sin\theta+y'\cos\theta 1852 | \end{cases} 1853 | \end{equation} 1854 | 把(6.25)代入(6.24),得 1855 | \begin{equation} 1856 | \boxed{\begin{cases} 1857 | x=x'\cos\theta-y'\sin\theta+h\\ 1858 | y=x'\sin\theta+y'\cos\theta+k 1859 | \end{cases} } 1860 | \end{equation} 1861 | 1862 | 由(6.26)式解出$x',y'$又可得 1863 | \begin{equation} 1864 | \boxed{\begin{cases} 1865 | x'=(x-h)\cos\theta+(y-k)\sin\theta\\ 1866 | y'=-(x-h)\sin\theta+(y-k)\cos\theta 1867 | \end{cases} } 1868 | \end{equation} 1869 | (6.26), (6.27)两个公式就是\textbf{一般的坐标变换公式}.公式(6.26)是 1870 | 通过新坐标来表示原坐标,公式(6.27)是通过原坐标来表示新 1871 | 坐标. 1872 | 1873 | \begin{example} 1874 | 已知直线$x+y-2=0$, 1875 | 平移坐标轴,使原点移到 1876 | $O'(1,1)$, 再旋转$\left(-\frac{\pi}{4}\right)$角, 1877 | 求直线$\ell$在新坐标系$O'X'Y'$中 1878 | 的方程(图6.32). 1879 | \end{example} 1880 | 1881 | \begin{figure}[htp] 1882 | \centering 1883 | \begin{tikzpicture}[>=latex] 1884 | \draw[->, thick](-2,0)--(3,0)node[right]{$X''$}; 1885 | \draw[->, thick](0,-2)--(0,3)node[right]{$Y''$}; 1886 | \draw[->, very thick](180+45:2.5)--(45:3)node[right]{$Y'$}; 1887 | \draw[very thick](-45:2.5)--(90+45:3)node[right]{$\ell$}; 1888 | \draw[->](0,0)--(-2.5,2.5)node[right]{$X'$}; 1889 | \draw[->](-2,-1)--(3,-1)node[right]{$X$}; 1890 | \draw[->](-1,-2)--(-1,3)node[right]{$Y$}; 1891 | 1892 | \node at (0,0)[below right]{$O'$}; 1893 | \node at (-1,-1)[below right]{$O$}; 1894 | \end{tikzpicture} 1895 | 1896 | \caption{} 1897 | \end{figure} 1898 | 1899 | \begin{solution} 1900 | 把已知量代入变换公式(6.27), 得 1901 | \[\begin{split} 1902 | x&=x'\cos\left(-\frac{\pi}{4}\right)-y'\sin\left(-\frac{\pi}{4}\right)+1=\frac{\sqrt{2}}{2}(x'+y')+1\\ 1903 | y&=x'\sin\left(-\frac{\pi}{4}\right)+y'\cos\left(-\frac{\pi}{4}\right)+1=\frac{\sqrt{2}}{2}(-x'+y')+1 1904 | \end{split}\] 1905 | 代入方程$x+y=2$得:$$y'=0$$ 1906 | 这就是直线$\ell$在新坐标系中的方程. 1907 | \end{solution} 1908 | 1909 | \begin{example} 1910 | 讨论线性分式函数$y=\frac{2x+1}{x-1}$ 1911 | 的图象. 1912 | \end{example} 1913 | 1914 | \begin{solution} 1915 | 原式可化为 $xy-2x-y-1=0$, 为了求得这个方程的图象,我们希望选择一个新坐标系,使 1916 | 图象在新坐标系中有较简单的方程.我们考虑移轴变换, 1917 | \[x=x'+h,\qquad y=y'+k\] 1918 | 代入原方程,得 1919 | \[x'y'+(k-2)x'+(h-1)y'+hk-2h-k-1=0\] 1920 | 如果取$h=1$, $k=2$, 上述方程变为 1921 | \[x'y'=3\] 1922 | 这就是图象在新坐标系 1923 | $O'X'Y'$中的方程,由例6.17可知它是以新坐标系 1924 | 的坐标轴为渐近线的等轴双 1925 | 曲线(图6.33). 1926 | \end{solution} 1927 | 1928 | \begin{figure}[htp] 1929 | \centering 1930 | \begin{tikzpicture}[>=latex, scale=.4] 1931 | \draw[->, thick](-6,0)--(6,0)node[right]{$X'$}; 1932 | \draw[->](-6,-2)--(6,-2)node[right]{$X$}; 1933 | \draw[->, thick](0,-6)--(0,6)node[right]{$Y'$}; 1934 | \draw[->](-1,-6)--(-1,6)node[right]{$Y$}; 1935 | \draw[dashed](-5,-5)--(5,5); 1936 | \draw[domain=.6:5.5, samples=100, very thick]plot(\x,{3/\x}); 1937 | \draw[domain=-5.5:-.6, samples=100, very thick]plot(\x,{3/\x}); 1938 | \node at (0,0)[below right]{$O'$}; 1939 | \node at (-1,-2)[below right]{$O$}; 1940 | \end{tikzpicture} 1941 | \caption{} 1942 | \end{figure} 1943 | 1944 | 1945 | 1946 | \begin{ex} 1947 | \begin{enumerate} 1948 | \item 写出下列平移坐标轴,使原点移到$(h,k)$, 再旋转$\theta$角的一般坐标变换公式: 1949 | \begin{enumerate} 1950 | \item $(h,k)=(1,1),\qquad \theta=\frac{\pi}{4}$ 1951 | \item $(h,k)=(-5,-3),\qquad \theta=120^{\circ}$ 1952 | \item $(h,k)=(0,3),\qquad \theta=-\frac{\pi}{2}$ 1953 | \end{enumerate} 1954 | 1955 | \item 给定坐标系$OXY$, 移轴使原点移到$O'(2,1)$, 再旋 1956 | 转$\frac{\pi}{4}$ 1957 | 角,写出曲线在新坐标系中的方程: 1958 | \begin{enumerate} 1959 | \item $2x-y-3=0$ 1960 | \item $x^2+6xy+y^2-10x-14y+9=0$ 1961 | \end{enumerate} 1962 | \item 讨论线性分式函数$y=\frac{x+1}{x-1}$ 1963 | 的图象. 1964 | \item 取两条互相垂直的直线$x+2y+1=0$, $2x-y-3=0$, 1965 | 作新坐标轴,写出坐标变换公式. 1966 | \end{enumerate} 1967 | \end{ex} 1968 | 1969 | \section*{习题6.2} 1970 | \addcontentsline{toc}{subsection}{习题6.2} 1971 | 1972 | 1973 | \begin{enumerate} 1974 | \item 给定坐标系$OXY$, 已知$P(2,1)$, 移轴将原点分别移 1975 | 到下列各点,求$P$点的新坐标. 1976 | \begin{multicols}{3} 1977 | \begin{enumerate} 1978 | \item $(0,3)$ 1979 | \item $(3,4)$ 1980 | \item $(-2,-6)$ 1981 | \end{enumerate} 1982 | \end{multicols} 1983 | 1984 | \item 一定点$P$在坐系$OXY$与$O'X'Y'$中的坐标分别是 1985 | $(2,5)$, $(-4,3)$且两坐标系的坐标轴方向相同, 1986 | 试求每一坐标系的原点相对于另一坐标系中的坐标. 1987 | \item 利用配方因式分解证明下列各方程都表示一对直线,并 1988 | 作平移变换,化简这些方程: 1989 | \begin{enumerate} 1990 | \item $x^2-y^2-4x+6y-5=0$ 1991 | \item $4x^2-9y^2+8x+18y-5=0$ 1992 | \item $4x^2-16y^2-12x+9=0$ 1993 | \end{enumerate} 1994 | 1995 | \item 转轴到怎样的角度后,才可使点$(2,0)$的两个坐标分 1996 | 量相等. 1997 | \item 利用移变换消去方程$xy-x-y=1$中的一次项. 1998 | \item 证明:方程$x^2+y^2=r^2$不因旋转坐标轴而变形. 1999 | \item 取两条互相垂直的直线 2000 | \[ ax+by+c_1=0,\qquad -bx+ay+c_2=0\] 2001 | 作新坐标系的坐标轴,建立新旧坐标系的变换公式. 2002 | \item 取直线$x+y=0$, $x-y=0$, 分别为新坐标系的 2003 | $X'$轴和$Y'$轴,求在新坐标系下曲线$x^2-y^2=a^2$的方程. 2004 | \end{enumerate} 2005 | 2006 | \section{一般二元二次方程的讨论} 2007 | \subsection{在坐标变换下二元二次方程系数的变换} 2008 | 一般二元二次方程可写为如下形式 2009 | \begin{equation} 2010 | Ax^2+2Bxy+Cy^2+2Dx+2Ey+F=0 2011 | \end{equation} 2012 | 方程的系数$A$、$B$、$C$中至少有一个不等于零,系数中析出 2013 | 因子2, 是为了以后演算的方便.凡坐标满足方程(6.28)的点 2014 | 的轨迹叫做\textbf{二次曲线}. 2015 | 2016 | 对方程(6.28), 我们进行平移和旋转变换,令 2017 | \[\begin{split} 2018 | x&=x'\cos\theta-y'\sin\theta+h\\ 2019 | y&=x'\sin\theta+y'\cos\theta+k 2020 | \end{split}\] 2021 | 代入(6.28)式,展开合并同类项,就得到同一个二次曲线在 2022 | $O'X'Y'$坐标系中的方程为 2023 | \begin{equation} 2024 | A'{x'}^2+2B'x'y'+C'{y'}^2+2D'x'+2E'y'+F'=0 2025 | \end{equation} 2026 | 其中 2027 | \begin{equation} 2028 | \begin{split} 2029 | A'&=A\cos^2\theta +2B\sin\theta \cos\theta +C\sin^2\theta \\ 2030 | 2B'&=-2A\sin\theta \cos\theta +2B(\cos^2\theta -\sin^2\theta)+2C\sin\theta \cos\theta\\ 2031 | &=2B\cos2\theta-(A-C)\sin2\theta \\ 2032 | C'&=A\sin^2\theta -2B\cos\theta \sin\theta +C\cos^2\theta \\ 2033 | D'&=2Ah\cos\theta +2B(k\cos\theta +h\sin\theta )+2Ck\sin\theta\\ 2034 | &\qquad +2D\cos\theta +2E\sin\theta \\ 2035 | E'&=-2Ah\sin\theta +2B(h\cos\theta -k\sin\theta)+2Ck\cos\theta\\ 2036 | &\qquad -2D\sin\theta +2E\cos\theta \\ 2037 | F'&=Ah^2+2Bhk+Ck^2+2Dh+2Ek+F 2038 | \end{split} 2039 | \end{equation} 2040 | 2041 | 上述关系式(6.30), 骤看起来是有些繁琐的,但稍加分析 2042 | 我们不难看出以下几点: 2043 | \begin{enumerate} 2044 | \item 一般二元二次方程,经坐标变换后,仍是二元二 2045 | 次方程,这就是说,上述二次曲线的定义与坐标系的选取无 2046 | 关. 2047 | \item 在(6.30)中,令$h=k=0$, 也就是坐标系只作旋 2048 | 转变换,这时二次项系数和一次项系数一般都发生改变,而 2049 | 常数项不变.新方程(6.29)中的二次项系数$A',B',C'$只与 2050 | 原方程(6.28)中二次项系数和转角$\theta$有关,而与原方程中 2051 | 的一次项系数和常数无关;新方程(6.29)中一次项系数只与原 2052 | 方程(6.28)中一次项系数及转角有关,而与二次项系数和常数 2053 | 无关. 2054 | \item 在(6.30)中,令$\theta=0$, 也就是坐标系只作平移变 2055 | 换,这时二次项系数不变,即 2056 | \[A=A',\qquad B=B',\qquad C=C'\] 2057 | 而一次项系数和常数项一般都要改变,且 2058 | \begin{equation} 2059 | \begin{split} 2060 | D'&=2Ah+2Bk+2D\\ 2061 | E'&=2Bh+2Ck+2E 2062 | \end{split} 2063 | \end{equation} 2064 | \end{enumerate} 2065 | 2066 | 最后让我们来证明,在一般坐标变换下,新方程(6.29)与 2067 | 原方程(6.28)的系数有如下关系: 2068 | \begin{enumerate} 2069 | \item $A+C=A'+C'$ 2070 | \item $B^2-AC={B'}^2-A'C'$ 2071 | \end{enumerate} 2072 | 2073 | \begin{proof} 2074 | \begin{enumerate} 2075 | \item \[\begin{split} 2076 | A'+C'&=A\cos^2\theta +2B\sin\theta \cos\theta 2077 | +C \sin^2\theta +A\sin^2\theta 2078 | \\ 2079 | &\qquad -2B\sin\theta \cos\theta +C\cos^2\theta\\ 2080 | &=A(\cos^2\theta+\sin^2\theta)+C(\sin^2\theta+\cos^2\theta)\\ 2081 | &=A+C 2082 | \end{split}\] 2083 | \item \[\begin{split} 2084 | A'-C'&=(A-C)\cos2\theta+2B\sin2\theta \\ 2085 | (2B')^2+(A'-C')^2&=(2B)^2+(A-C)^2 2086 | \end{split}\] 2087 | 即 2088 | \[\begin{split} 2089 | {B'}^2-A'C'&=\frac{1}{4}\left[(2B)^2+(A-C)^2-(A'+C')^2\right]\\ 2090 | &=\frac{1}{4}\left[(2B)^2+(A-C)^2-(A+C)^2\right]\\ 2091 | &=B^2-AC 2092 | \end{split}\] 2093 | \end{enumerate} 2094 | \end{proof} 2095 | 2096 | 由以上证明可知,\textbf{$A+C$和$B^2-AC$都是二元二次方 2097 | 程在一般坐标变换下不变的量}. 2098 | 2099 | \begin{ex} 2100 | \begin{enumerate} 2101 | \item 对方程(6.28)作旋转变换,如果使方程(6.29)中$B'=0$, 2102 | 求证:$(A'-C')^2=(A-C)^2+4B^2$. 2103 | \item 对方程(6.28)作平移变换,如果使方程(6.29)中,消去各一 2104 | 次项,求证: 2105 | \[F=\frac{-\begin{vmatrix} 2106 | A&B&D\\B&C&E\\D&E&F 2107 | \end{vmatrix}}{B^2-AC}\] 2108 | \end{enumerate} 2109 | \end{ex} 2110 | 2111 | \subsection{一般二元二次方程的化简} 2112 | 这节,我们来研究,如何选取适当的坐标系,使二次曲 2113 | 线的方程有较简单的形式. 2114 | 2115 | \subsubsection{用平移变换消去二元二次方程中各一次项} 2116 | 由(6.31)式可知,对一般二元二次方程,若要在新 2117 | 坐标系中消去各一次项,只要作平移变换选取$(h,k)$使 2118 | \[\begin{cases} 2119 | Ah+Bk+D=0\\ 2120 | Bh+Ck+E=0 2121 | \end{cases}\] 2122 | 在$B^2-AC\ne 0$时,此方程组有唯一一组解$(h,k)$, 我们 2123 | 将坐标原点移到$(h,k)$, 就可使二次曲线在新坐标系中的 2124 | 方程中$D'=E'=0$. 在$B^2-AC=0$时,若$A:B=B:C 2125 | =D:E$, 方程组有无穷多解,若$A:B=B:C\ne D:E$, 2126 | 方程组无解,在后一种情况出现时,我们可先用下面(二)中 2127 | 介绍的方法去化简方程. 2128 | 2129 | \begin{example} 2130 | 平移坐标轴,化简方程 2131 | $2x^2+3y^2-8x+6y-7=0$ 2132 | 并画出新坐标系和方程的曲线. 2133 | \end{example} 2134 | 2135 | \begin{solution} 2136 | 令$x=x'+h$, $y=y'+k$, 代入已知方程,得 2137 | \[2(x'+h)^2+3(y'+k)^2-8(x'+h)+6(y'+k)-7=0\] 2138 | 就是, 2139 | \[2{x'}^2+3{y'}^2+(4h-8)x'+(6k+6)y'+2h^2+3k^2 2140 | -8h+6k-7=0\] 2141 | 令$4h-8=0$, $6k+6=0$,解得 2142 | $h=2$, $k=-1$, 代入方程(6.28),得 2143 | \[2{x'}^2+3{y'}^2=18\quad \Rightarrow\quad \frac{{x'}^2}{9}+\frac{{y'}^2}{6}=1\] 2144 | 这是椭圆的标准方程,对原 2145 | 坐标系来说,它的中心在$O'(2,-1)$, 它的长轴和 2146 | 短轴分别在直线$y=-1$, 2147 | $x=2$上,它的长轴长是6, 短轴长是$2\sqrt{6}$. 新坐标和曲线如图6.34所示. 2148 | \end{solution} 2149 | 2150 | \begin{figure}[htp]\centering 2151 | \begin{minipage}[t]{0.48\textwidth} 2152 | \centering 2153 | \begin{tikzpicture}[>=latex, scale=.6] 2154 | \draw[->, thick](-4,0)--(4,0)node[right]{$X'$}; 2155 | \draw[->](-4,1)--(4,1)node[right]{$X$}; 2156 | \draw[->, thick](0,-4)--(0,4)node[right]{$Y'$}; 2157 | \draw[->](-2,-4)--(-2,4)node[right]{$Y$}; 2158 | \node at (0,0) [below left]{$O'$};\node at (-2,1) [below left]{$O$}; 2159 | \draw[thick](0,0)ellipse[x radius= 3 , y radius=2.45 ]; 2160 | 2161 | \end{tikzpicture} 2162 | \caption{} 2163 | \end{minipage} 2164 | \begin{minipage}[t]{0.48\textwidth} 2165 | \centering 2166 | \begin{tikzpicture}[>=latex, scale=.4] 2167 | \draw[->, thick](-6,0)--(6,0)node[right]{$X'$}; 2168 | \draw[->](-6,-3)--(6,-3)node[right]{$X$}; 2169 | \draw[->, thick](0,-6)--(0,6)node[left]{$Y'$}; 2170 | \draw[->](1,-6)--(1,6)node[right]{$Y$}; 2171 | 2172 | \draw[domain=-4:4, samples=100, thick]plot({sqrt(8+1.33*\x*\x)},\x); 2173 | \draw[domain=-4:4, samples=100, thick]plot({-sqrt(8+1.33*\x*\x)},\x); 2174 | 2175 | \draw[domain=-6:6, samples=10]plot(\x,{0.866*\x}); 2176 | \draw[domain=-6:6, samples=10]plot(\x,{-0.866*\x}); 2177 | \draw(-2.828,-2.45) rectangle(2.828,2.45); 2178 | \node at (0,0) [below left]{$O'$};\node at (1,-3) [below left]{$O$}; 2179 | 2180 | \end{tikzpicture} 2181 | \caption{} 2182 | \end{minipage} 2183 | \end{figure} 2184 | 2185 | 2186 | \begin{example} 2187 | 平移坐标轴,化简方程 2188 | $3x^2-4y^2+6x+24y-57=0$ 2189 | 并画出新坐标系和方程的曲线. 2190 | \end{example} 2191 | 2192 | \begin{solution} 2193 | 把已知方程按$x,y$配方,得 2194 | \[3(x+1)^2-4(y-3)^2=24\] 2195 | 令$x'=x+1$, $y'=y-3$,代入上面方程, 2196 | 得: 2197 | \[3{x'}^2-4{y'}^2=24 \quad \Rightarrow\quad 2198 | \frac{{x'}^2}{8}-\frac{{y'}^2}{6}=1\] 2199 | 这是双曲线的标准方程, 2200 | 新坐标系和曲线如图6.35所示. 2201 | \end{solution} 2202 | 2203 | \subsubsection{用旋转变换,消去(6.28)中的$xy$项} 2204 | 2205 | 由关系式(6.30), 对一般二元二次方程,若要在 2206 | 新坐标系中使得方程不含$x'y'$项,只要选取$\theta$角,使 2207 | \[2B=2Bcos2\theta -(A-C)\sin\theta=0\] 2208 | 即 2209 | \[\cot 2\theta=\frac{A-C}{2B},\qquad \left(0<\theta<\frac{\pi}{2}\right)\] 2210 | 把坐标轴旋转由上式所决定的$\theta$角,就可使二次曲线在 2211 | 新坐标系中的方程不含$x'y'$项. 2212 | 2213 | \begin{example} 2214 | 利用坐标轴旋转化简二次方程 2215 | $8x^2+4xy+5y^2-36=0$ 2216 | 并画出它的图形. 2217 | \end{example} 2218 | 2219 | \begin{solution} 2220 | \[\cot 2\theta=\frac{A-C}{2B}=\frac{8-5}{4}=\frac{3}{4}\] 2221 | 由于$\cos2\theta$与$\cot2\theta$同号,所以 2222 | \[\begin{split} 2223 | \cos2\theta&=\frac{\frac{3}{4}}{\sqrt{1+\left(\frac{3}{4}\right)^2}}=\frac{3}{5}\\ 2224 | \sin\theta&=\sqrt{\frac{1-\frac{3}{5}}{2}}=\frac{1}{\sqrt{5}}\\ 2225 | \cos\theta&=\sqrt{\frac{1+\frac{3}{5}}{2}}=\frac{2}{\sqrt{5}}\\ 2226 | \end{split}\] 2227 | 因此,可令旋转变换为 2228 | \[x=\frac{2}{\sqrt{5}}x'-\frac{1}{\sqrt{5}}y',\qquad y=\frac{1}{\sqrt{5}}x'+\frac{2}{\sqrt{5}}y'\] 2229 | 代入原方程化简,得 2230 | \[9{x'}^2+4{y'}^2=36\] 2231 | 这是一个椭圆,长轴在$Y'$轴上. 2232 | 2233 | 根据$\sin\theta=\frac{1}{\sqrt{5}}$, 2234 | 得旋转角$\theta\approx 26^{\circ}34'$, 它的图形如 2235 | 图6.36所示. 2236 | \end{solution} 2237 | 2238 | \begin{figure}[htp] 2239 | \centering 2240 | \begin{tikzpicture}[>=latex, scale=.7] 2241 | \draw[->, thick](-180+26.56:4)--(26.56:4)node[right]{$X'$}; 2242 | \draw[->](-4,0)--(4,0)node[right]{$X$}; 2243 | \draw[->, thick](-90+26.56:4)--(26.56+90:4)node[left]{$Y'$}; 2244 | \draw[->](0,-4)--(0,4)node[right]{$Y$}; 2245 | 2246 | \draw[very thick, rotate=26.56](0,0) ellipse[x radius=2, y radius=3]; 2247 | \node at (0,0) [below left]{$O$}; 2248 | \draw[->](.85,0) arc (0:26.56:.85)node[right]{$\theta$}; 2249 | \end{tikzpicture} 2250 | \caption{} 2251 | \end{figure} 2252 | 2253 | \begin{ex} 2254 | \begin{enumerate} 2255 | \item 平移坐标轴,化简下列各二次方程 2256 | \begin{enumerate} 2257 | \item $x^2+y^2-2x+2y+3=0$ 2258 | \item $9x^2+4y^2-36x+16y+16=0$ 2259 | \item $2x^2-4y^2+4x+4y-1=0$ 2260 | \item $xy-6x-8y+20=0$ 2261 | \end{enumerate} 2262 | 2263 | \item 旋转坐标轴,化简下列各二次方程 2264 | \begin{enumerate} 2265 | \item $x^2+12xy+9y^2-16=0$ 2266 | \item $x^2-2xy+y^2+2x-4y+3=0$ 2267 | \end{enumerate} 2268 | 2269 | \item 化简下列各二次方程 2270 | \begin{enumerate} 2271 | \item $5x^2+6xy+5y^2-4x+4y-4=0$ 2272 | \item $9x^2+4xy+6y^2+12x+36y+44=0$ 2273 | \end{enumerate} 2274 | \end{enumerate} 2275 | \end{ex} 2276 | 2277 | \subsection{一般二元二次方程的讨论} 2278 | 由前节可知,一些二元二次方程,经一般坐标变换 2279 | 后可化为圆锥曲线的标准方程,这节我们对二元二次方程作 2280 | 一般性的讨论,看看如何根据二次曲线的方程来判断它的形 2281 | 状和位置. 2282 | 2283 | 给定二次曲线 2284 | \begin{equation} 2285 | Ax^2+2Bxy+Cy^2+2Dx+2Ey+F=0 2286 | \end{equation} 2287 | 我们总可通过转轴,选取适当的坐标系$OX'Y'$, 使二次曲 2288 | 线的方程在这个新系中不含$x'y'$项,由于转轴后方程(6.32) 2289 | 中的常数项不变,新方程可写为 2290 | \begin{equation} 2291 | A'{x'}^2+C'{y'}^2+2D'x'+2E'y'+F=0 2292 | \end{equation} 2293 | 下面分两种情况讨论: 2294 | \begin{enumerate} 2295 | \item $A'$、$C'$都不等于零(即$A'C'\ne 0$).再作平 2296 | 移变换,消去一次项,由于移轴后方程(6.33)中的二次项系数 2297 | 不变,所以新方程可写为 2298 | \begin{equation} 2299 | A'{x''}^2+C'{y''}^2+F'=0 2300 | \end{equation} 2301 | 我们再分两种情况: 2302 | \begin{enumerate} 2303 | \item $A'$、$C'$同号(即$A'C'>0$).当$F'\ne 0$,且 2304 | $A'$、$C'$与$F'$异号时,方程的图象是椭圆;当$F'\ne 0$且 2305 | $A'$、$C'$与$F'$同号时,显然没有点的坐标满足方程(6.34), 2306 | 因此,方程的图象不存在;当$F'= 0$, 且$A'$、$C'$同号时, 2307 | 显然方程的图象只有一点. 2308 | \item $A'$、$C'$异号(即$A'C'<0$).当$F'\ne 0$时, 2309 | 方程(6.32)的轨迹是双曲线;当$F'= 0$时,方程(6.34)可分解 2310 | 为 2311 | \[\left(x''+\sqrt{-\frac{C'}{A'}}y''\right)\cdot \left(x''-\sqrt{-\frac{C'}{A'}}y''\right)=0\] 2312 | 因此,方程的轨迹是两条相交直线. 2313 | \end{enumerate} 2314 | \item $A'$、$C'$中有一个为零(即$A'C'=0$). 2315 | 设$A'=0$, 则方程(6.33)变为 2316 | \begin{equation} 2317 | C'{y'}^2+2D'x'+2E'y'+F=0 2318 | \end{equation} 2319 | 作平移变换:令$x''=x'$, $y=y'+\frac{E'}{C'}$, 2320 | 方程(6.35)在新坐标 2321 | 系中的方程可写为 2322 | \begin{equation} 2323 | C'{y''}^2+2D'x''+F'=0 2324 | \end{equation} 2325 | 我们再分两种情况: 2326 | \begin{enumerate} 2327 | \item $D'\ne 0$, 这时方程的图象是抛物线. 2328 | \item $D'=0$, 当$F'\ne 0$且$C'$与$F'$异号,方程(6.36) 2329 | 变为 2330 | \[y''\pm \sqrt{-\frac{F'}{C'}}=0\] 2331 | 这时方程的图象两条平行直线;当$F'\ne 0$且$C'$与$F'$同 2332 | 号,显然没有点的坐标满足方程,这时方程(6.32)没有轨迹,当 2333 | $F'=0$时,方程(6.36)化为$y''=0$, 2334 | 这时方程(6.32)表示两条重合的直线. 2335 | \end{enumerate} 2336 | \end{enumerate} 2337 | 2338 | 2339 | 由以上讨论可知,\textbf{一般二次曲线或者是圆锥曲线(椭圆、 2340 | 双曲线、抛物线),或者是两条直线(包括重合情况), 2341 | 或者是一个点,或者不存在}. 2342 | 2343 | 一般我们把情况1中的类型(a)叫做\textbf{椭圆型方程}, 2344 | 类型(b)叫做\textbf{双曲线型方程};情况2中的类型叫做 2345 | \textbf{抛物线型方程}.由于在方程(6.33)中$B'=0$, 所以 2346 | \[B^2-AC={B'}^2-A'C'=-A'C'\] 2347 | 这样,上面情况1中的类型(a)的条件,$A'$、$C'$同 2348 | 号相当于$B^2-AC<0$; $A'$、$C'$异号相当于$B^2-AC 2349 | >0$;情况2中的条件$A'C'$中有一个为零相当于$B^2- 2350 | AC=0$. 因此,我们可不作坐标变换,直接根据$B^2-AC$ 2351 | 来判别二次曲线的类型.$B^2-AC$叫做一般二元二次方 2352 | 程的\textbf{判别式}. 2353 | 2354 | 由判别式判别二次曲线的类型,我们归纳为下表. 2355 | \begin{center} 2356 | 方程$Ax^2+2Bxy+Cy^2+2Dx+2Ey+F=0$ 2357 | \begin{tabular}{cccc} 2358 | \hline 2359 | 判别式&类型&一般情形&特殊情形(退化二次曲线)\\ 2360 | \hline 2361 | $B^2-4C<0$ &椭圆型&椭圆&一点或没有图象\\ 2362 | $B^2-4C>0$ &双曲线型&双曲线&两条相交直线\\ 2363 | $B^2-AC=0$&抛物线型&抛物线&两条平行或重合直线或没有图象\\ 2364 | \hline 2365 | \end{tabular} 2366 | \end{center} 2367 | 2368 | 2369 | 2370 | \begin{example} 2371 | 试判别下列方程的类型 2372 | \begin{enumerate} 2373 | \item $x^2-3xy+2y^2-x-5y+3=0$ 2374 | \item $9x^2-6xy+y^2-4=0$ 2375 | \item $3x^2-2xy+y^2-5x-2y+34=0$ 2376 | \end{enumerate} 2377 | \end{example} 2378 | 2379 | \begin{solution} 2380 | \begin{enumerate} 2381 | \item $B^2-AC=\left(\frac{3}{2}\right)^2-1\x 2=\frac{9}{4}-1>0$ 2382 | 因此方程是双曲线型. 2383 | \item $B^2-AC=\left(-3\right)^2-9\x 1=0$ 2384 | 因此方程是抛物线型. 2385 | \item $B^2-AC=\left(-1\right)^2-3\x 1=-2<0$ 2386 | 因此方程是椭圆型. 2387 | \end{enumerate} 2388 | \end{solution} 2389 | 2390 | \begin{example} 2391 | 判别方程 2392 | $3x^2+12xy+12y^2+10x+10y-3=0$ 2393 | 的类型,并画出它的图形. 2394 | \end{example} 2395 | 2396 | \begin{solution} 2397 | $B^2-AC=\left(6\right)^2-3\x 12=0$, 2398 | 因此方程是抛物线型. 2399 | 作旋转变换 2400 | \[\begin{split} 2401 | \cot2\theta&=\frac{3-12}{12}=-\frac{3}{4},\qquad 2402 | \cos2\theta=\frac{-\frac{3}{4}}{\sqrt{1+\left(\frac{3}{4}\right)^2}}=-\frac{3}{5}\\ 2403 | \sin\theta&=\frac{2}{\sqrt{5}},\qquad \cos\theta=\frac{1}{\sqrt{5}},\qquad 2404 | \theta \approx 63^{\circ}26' 2405 | \end{split}\] 2406 | 旋转公式是 2407 | \[x=\frac{1}{\sqrt{5}}(x'-2y'),\qquad y=\frac{1}{\sqrt{5}}(2x'+y')\] 2408 | 代入原方程,化简得 2409 | \[15{x'}^2+6\sqrt{5}x'-2\sqrt{5}y'-3=0\] 2410 | 对$x'$配方,方程可写为 2411 | \[15\left(x'+\frac{1}{\sqrt{5}}\right)^2=2\sqrt{5}\left(y'+\frac{3}{\sqrt{5}}\right)\] 2412 | 作平移变换,令$x''=x'+\frac{1}{\sqrt{5}}$, $y''=y'+\frac{3}{\sqrt{5}}$,最后方程变为 2413 | \[15{x''}^2=2\sqrt{5}y''\] 2414 | 这是一条抛物线(图6.37),在$OX'Y'$坐标系中,它的顶点 2415 | 是$O'\left(-\frac{1}{\sqrt{5}},-\frac{3}{\sqrt{5}}\right)$ 2416 | ,由旋转公式容易求得,在原坐标 2417 | 系$OXY$中,它的顶点是$O'(1,-1)$. $x''$轴是直线 2418 | $2x-y-3=0$, $y''$轴是直线$x+2y+1=0$. 2419 | \end{solution} 2420 | 2421 | \begin{figure}[htp] 2422 | \centering 2423 | \begin{tikzpicture}[>=latex, rotate=63.43] 2424 | 2425 | \draw[->, thick](-3,0)--(3,0)node[right]{$X''$}; 2426 | \draw[->](-3,1.34)--(3,1.34)node[right]{$X'$}; 2427 | \draw[->, thick](0,-2)--(0,5)node[left]{$Y''$}; 2428 | \draw[->](0.45,-2)--(0.45,5)node[above]{$Y'$}; 2429 | \draw[domain=-1.1:1.1, samples=100, very thick]plot(\x, {3.35*\x*\x}); 2430 | \draw[->](0.45,1.34)--+(90-63.43:3)node[right]{$Y$}; 2431 | \draw(0.45,1.34)--+(90-63.43:-4); 2432 | \draw[->](0.45,1.34)--+(-63.43:4)node[right]{$X$}; 2433 | \draw(0.45,1.34)--+(-63.43:-4); 2434 | \node at (0,0)[below]{$O'$}; 2435 | \node at (0.45,1.34)[below left]{$O$}; 2436 | \draw[<-](0.45+.5,1.34) arc (0:-63.43:.5); 2437 | \end{tikzpicture} 2438 | \caption{} 2439 | \end{figure} 2440 | 2441 | 2442 | 2443 | \begin{ex} 2444 | \begin{enumerate} 2445 | \item 试判别下列方程的类型 2446 | \begin{enumerate} 2447 | \item $16x^2-24xy+9y^2-38x-34y+71=0$ 2448 | \item $x^2-5xy+13y^2-3x+21y=0$ 2449 | \item $8x^2+8xy-7y^2+36y+36=0$ 2450 | \item $4x^2+9y^2-16x-18y-11=0$ 2451 | \item $2x^2+5xy-3y^2+3x+16y-5=0$ 2452 | \end{enumerate} 2453 | 2454 | \item 判别下列方程的类型,并画出它们的图形 2455 | \begin{enumerate} 2456 | \item $5x^2-6xy+5y^2-4x-4y-4=0$ 2457 | \item $7x^2-8xy+y^2+14x-8y-2=0$ 2458 | \item $x^2-2xy+y^2+3x-y-4=0$ 2459 | \item $3x^2-xy+5y^2-6x+y+3=0$ 2460 | \item $4x^2+12xy+9y^2+2x+3y+2=0$ 2461 | \end{enumerate} 2462 | 2463 | \end{enumerate} 2464 | \end{ex} 2465 | 2466 | \section*{习题6.3} 2467 | \addcontentsline{toc}{subsection}{习题6.3} 2468 | 2469 | \begin{enumerate} 2470 | \item 化简下列方程,求对称轴方程,并画出方程的图象. 2471 | \begin{enumerate} 2472 | \item $11x^2+6xy+3y^2-12x+2y-12=0$ 2473 | \item $7x^2-8xy+y^2+14x-8y+16=0$ 2474 | \item $8x^2+8xy+2y^2-6x-3y-5=0$ 2475 | \item $x^2-2xy-6x+4y+4=0$ 2476 | \end{enumerate} 2477 | 2478 | \item 证明二元二次方程表示等轴双曲线或两条互相垂直的直 2479 | 线的充要条件是$A+C=0$. 2480 | \item 证明抛物线$y=ax^2+bx+c\; (a\ne 0)$的对称轴平行于原 2481 | 坐标轴. 2482 | \item 方程$2x^2+\lambda xy+4y^2-7x+\lambda^2y+3=0$中,$\lambda$取什么 2483 | 值时,方程是:椭圆型;双曲线型;抛物 2484 | 线型. 2485 | \item 设一二次曲线过点$(2,3)$, $(4,2)$, $(-1,-3)$, 且以 2486 | $(0,1)$为对称中心,求这曲线方程. 2487 | \end{enumerate} 2488 | 2489 | \section*{复习题六} 2490 | \addcontentsline{toc}{section}{复习题六} 2491 | 2492 | \begin{enumerate} 2493 | \item 已知椭圆的两个焦点分别是$F_1(2,4)$、$F_2(8,4)$并经 2494 | 点$A(5,0)$, 求此椭圆方程. 2495 | \item 两条直线$3x\pm 4y=0$都是适合下列各条件的双曲线的渐 2496 | 近线,求各双曲线方程. 2497 | \begin{enumerate} 2498 | \item 焦点在点$(0,10)$; 2499 | \item 焦点在点$(5,0)$; 2500 | \item 经过点$(7,2)$. 2501 | \end{enumerate} 2502 | 2503 | \item 求适合下列条件的抛物线的方程式. 2504 | \begin{enumerate} 2505 | \item 顶点在点$(2,4)$, 焦点在点$(3,4)$; 2506 | \item 经过$(0,1)$, $(2,3)$, $(5,-1)$三点且它的轴 2507 | 平行于$Y$轴; 2508 | \item 顶点在原点,准线是$x=3$. 2509 | \end{enumerate} 2510 | 2511 | \item 已知椭圆$\frac{x^2}{a^2}+\frac{y^2}{b^2}=1$,直线$\overline{OP}$与$\overline{OQ}$互相垂直并与 2512 | 椭圆分别相交于$P$、$Q$两点,求证: 2513 | \[\frac{1}{\overline{OP}^2}+\frac{1}{\overline{OQ}^2}=\frac{1}{a^2}+\frac{1}{b^2}\] 2514 | \item 已知$P(x_1,y_1)$和$Q(x_2,y_2)$是椭圆$b^2x^2+a^2y^2=a^2b^2$ 2515 | 上任意两点,又知点$L(e_{x_1},0)$, 点$M(e_{x_2},0)$; 2516 | 求证:$\overline{PM}=\overline{QL}$. 2517 | \item 已知双曲线$\frac{x^2}{a^2}-\frac{y^2}{b^2}=1$, 2518 | 求证:通过点$M(h,k)$且被 2519 | $M$点平分的弦的方程是 2520 | \[\frac{hx}{a^2}-\frac{ky}{b^2}=\frac{h^2}{a^2}-\frac{k^2}{b^2}\] 2521 | \item 证明方程 2522 | \[\frac{x^2}{9+\lambda}+\frac{y^2}{5+\lambda}=1\] 2523 | 当$\lambda>-5$时,表示椭圆,当$-9<\lambda<-5$时,表示双 2524 | 曲线,并证明所有这些椭圆和双曲线具有公共的焦点 2525 | $(\pm 2,0)$. 2526 | \item 已知方程 2527 | \[\frac{x^2}{a^2+\lambda}+\frac{y^2}{b^2+\lambda}=1,\qquad a>b>0\] 2528 | 问$\lambda$为何值时,表示椭圆;表示双曲线.并证明 2529 | 所有这些椭圆和双曲线有公共焦点. 2530 | 2531 | \item 已知双曲线的轴是坐标轴,且通过点$(1,4)$和点$(-2, 2532 | 7)$, 求这双曲线的方程. 2533 | \item 证明由方程$4x^2-5y^2=c$($c$为非零常数)所确定的 2534 | 双曲线具有公共的渐近线. 2535 | \item 设$\alpha$是双曲线 2536 | $\frac{x^2}{a^2}-\frac{y^2}{b^2}=1$的两条渐近线的夹角,证明 2537 | $\cos\alpha=2e^{-2}-1$. 2538 | \item 已知双曲线$\frac{x^2}{a^2}-\frac{y^2}{b^2}=1$, 如果与双曲线在$P$点的切线 2539 | 与两条渐近线分别相交于$E$、$F$, 求证: 2540 | \begin{enumerate} 2541 | \item $P$点是$EF$的中点; 2542 | \item $\overline{OE}\cdot \overline{OF}=a^2+b^2$. 2543 | \end{enumerate} 2544 | 2545 | \item 双曲线$x^2-y^2=a^2$在$P$点的法线与坐标轴相交于$C$、 2546 | $D$两点,求证:$P$点是通过$O$、$C$、$D$三点圆的中心. 2547 | \item 已知双曲线 2548 | $\frac{x^2}{a^2}-\frac{y^2}{b^2}=1$在$P$点的法线分别与$X$轴,$Y$轴 2549 | 相交于$C$、$D$两点,求证$\overline{CD}$中点的轨迹是 2550 | \[4(a^2x^2-b^2y^2)=(a^2+b^2)^2\] 2551 | \item 求证椭圆只有一个内接正方形和一个外切正方形. 2552 | \item 证明通过点$M(a,b)$的椭圆$b^2x^2+a^2y^2=a^2b^2$的弦的 2553 | 中点的轨迹是 2554 | \[\frac{x^2}{a^2}+\frac{y^2}{b^2}=\frac{x}{a}+\frac{y}{b}\] 2555 | \item 从椭圆外一点$P(x_1,y_1)$引椭圆的两条切线,求证: 2556 | 通过两个切点的直线方程为 2557 | \[\frac{xx_1}{a^2}+\frac{yy_1}{b^2}=1\] 2558 | 2559 | \item 证明:在过椭圆焦点弦的两个端点处的切线相交在椭圆 2560 | 的准线上. 2561 | \item 求抛物线$y^2=8ax$和$x^2=ay$在公共点切线之间的交 2562 | 角. 2563 | \item 求椭圆$\frac{x^2}{6}+\frac{y^2}{3}=1$ 2564 | 的外切正方形的边长. 2565 | \item 已知椭圆的轴平行于坐标轴且与$X$轴相切于点$(7,0)$, 2566 | 与$Y$轴相切于点$(0,4)$. 求这椭圆的方程. 2567 | \item 在抛物线$x^2=ay\; (a>0)$上求一点$N$, 使它到$M(0,ka)$ 2568 | ($k>0$且为定值)的距离最小;又当$a$变化时,求$N$点的 2569 | 轨迹. 2570 | \item 求抛物线$4x^2+4x+3y-2=0$的顶点和焦点的坐标及其 2571 | 对称轴和准线方程. 2572 | \item 证明:任何一个以椭圆$\frac{x^2}{a^2}+\frac{y^2}{b^2}=1$ 2573 | 的互为共轭直径的端点为顶点的平行四边形的面积都等 2574 | 于常数$2ab$. 2575 | \item 证明:外切椭圆的矩形,其对角线之长等于定量, 2576 | \item 试证明在抛物线上三点$P_1$、$P_2$、$P_3$各引切线,这三 2577 | 条切线所围成的三角形面积等于$\triangle P_1P_2P_3$面积的一 2578 | 半. 2579 | \item 判定下列二次曲线的类型,并把它们化为标准方程. 2580 | \begin{enumerate} 2581 | \item $8x^2+4xy+5y^2+8x-16y-16=0$ 2582 | \item $x^2-4xy-2y^2+10x+4y=0$ 2583 | \item $4x^2-4xy+y^2+4x-2y=0$ 2584 | \end{enumerate} 2585 | \end{enumerate} 2586 | 2587 | 2588 | --------------------------------------------------------------------------------